You are on page 1of 204

केन्द्रीय विद्यालय संगठन

KENDRIYA VIDYALAYA SANGATHAN

केन्द्रीय विद्यालय संगठन, कोलकाता सम्भाग


KENDRIYA VIDYALAYA SANGATHAN, KOLKATA REGION

अध्ययन सामग्री/ STUDY MATERIAL

Academic Session–2023-24

Class –TWELVE(XII)

Subject – MATHEMATICS

Subject Code - 041

1
Chief Patron
Shri Y Arun Kumar
Deputy Commissioner, KVS RO KOLKATA

Patron
Shri Sanjib Sinha, Assistant Commissioner
Shri Chintapalli Vijaya Ratnam, Assistant Commissioner
Shri Dibakar Bhoi, Assistant Commissioner
Shri Amit Baidya, Assistant Commissioner

Advisors
Shri Dibyendu Datta - Principal, KV Garden Reach
Shri Abhijit Saha – Principal KV Burdwan

Review team
Shri Pintu Banerjee, PGT(Maths), KV Fort William
Shri Pinaki Chakraborty PGT(Maths), KV Of Dumdum
Shri Biswajit Patra, PGT(Maths), KV Kharagpur No 2

2
A note to the students….
Dear students,

This material is prepared for you, for your needs while making revision, practice questions or
preparing for CBSE examinations. We tried to prepare a handy material which can be used by
all students especially for the late bloomers who struggle a bit. This material provides all the
valuable and precise information and it contains all information for a quick revision.

Main features of the material

• Latest CBSE curriculum and sample question paper has been provided so that latest trend
of the question paper can clearly be understood.
• All the necessary synopsis and essential formula are included in the beginning of material
for quick revision.
• A good number of practice questions from each chapter are included according to the
latest pattern of CBSE.

All care has been taken to include those information which are relevant and in support with the
text book, but in no way it is the replacement for NCERT book and NCERT Exemplar rather it
is supplement to those books. Your teacher will help you to choose the key areas of scoring
during revision after completion of syllabus.

Best of luck for the exams.

From All the teachers

3
4
5
6
Latest CBSE SAMPLE PAPER

7
8
9
10
11
12
13
14
15
16
Quick revision concepts and important formula

17
18
19
20
21
22
23
24
25
PROBABILITY

26
PRACTICE MATERIAL

CHAPTER-1: RELATIONS AND FUNCTIONS


Q. QUESTION MARK
NO
1 Let R be the relation in the set N given by R = {(a, b) : a = b – 2, b > 6}, then 1
(a) (2, 4)∈ R (b) (3, 8) ∈ R (c) (6, 8) ∈ R (d) (8, 7) ∈ R

2 A relation R in the set A = [1, 2, 3} is defined as R = {(1, 1), (1, 2), (2, 2), (3, 3)}. Which of the 1
following ordered pair in R shall be removed to make it an equivalence relation in A
(a) (1, 1) (b) (1, 2) (c) (2, 2) (d) (3, 3)

3 The maximum number of equivalence relation on the set A ={ 2, 4, 6} is/are 1


(a) 1 (b) 2 (c) 3 (d)

4 Let 𝑓∶R → R defined by f(x) = 1+ x2. Choose the correct answer 1


(a) both one -one and onto (b)one-one but not onto
(c) onto but not one-one (d) Neither one- one nor onto

27
5 Let set S = {1, 2, 3} and a relation R is defined in X as : R = {(1, 2), (2, 2), (3, 2)}, then minimum 1
ordered pair which should be added in R to make it reflexive and symmetric are
(a) {(1, 1), (2, 3), (1, 2)} (b) {(3, 3), (3, 1), (1, 2)}
(c) {(1, 1), (3, 3), (3,1), (2, 3)} (d) {(1, 1), (3, 3), (3,1), (1, 2)}

6 Let A = {1, 2}. Then number of reflexive relation in A is 1


(a) 2 (b) 4 (c) 0 (d) 8

7 Let the relation R in the set A = { 𝑥𝜖 𝑍: 0 ≤ 𝑥 ≤ 12}, given by R = { (a,b): |a-b| is multiple of 1
4 }. Then [1], the equivalence class containing 1 is
(a) {1, 5, 9} (b) { 0, 1, 2, 5} (c)∅ (d) A

8 Consider the non-empty set consisting of children in a family and relation r is defined asaRb 1
if a is brother of b. Then R is
(a ) Symmetric but not transitive (b) transitive but nit symmetric
(c ) neither symmetric nor transitive (d) both symmetric and transitive

9 Let f : R → R be defined by f (x) = 1/x ∀ x ∈ R. Then f is 1


(A) one-one (B) onto (C) bijective (D) f is not defined

10 A relation R is defined on Z as 1
aRb if and only if 𝑎2 − 7𝑎𝑏 + 6𝑏 2 = 0. Then R is
(a) reflexive and symmetric (b) symmetric but not reflexive
(c ) transitive but not reflexive (d) reflexive but not symmetric

11 Let A = {1, 3, 5}. Then the number of equivalence relation in A containing (1, 3) is 1
(a) 1 (b) 2 (c) 3 (d) 4

12 The number of functions defined from {1, 2, 3, 4, 5} → {𝑎, 𝑏}, which are one –one is 1
(a) 5 (b) 3 (c) 2 (d) 0

13 The function f: R → R defined as f(x) = x3 is 1


(a) one-one but not onto (b) not one-one but onto
(c) neither one-one nor onto (d) one-one and onto

14 The function f: Z → Z defined as f(x) = x3 is 1


(a) one-one but not onto (b) not one-one but onto
(c) neither one-one nor onto (d) one-one and onto

28
15 For real numbers x and y , define xRy if and only if 𝑥 − 𝑦 + √2 is an irrational number. Then 1
the relation R is
(a) reflexive (b) symmetric (c) transitive (d) none of these

16 If the set A contains 5 elements and the set B contains 6 elements, then the number of one- 1
one and onto mappings from A to B is
(a) 720 (b) 120 (c) 0 (d) none of these

17 If the set A contains 3 elements and the set B contains 4 elements, then the number of one- 1
one mappings from A to B is
(a) 144 (b) 81 (c) 24 (d) 64

Direction (Q. 18 - Q. 20) -


In the following questions, a statement of assertion (A) is followed by a statement of

Reason (R). Choose the correct answer out of the following choices:

(a) Both A and R are true and R is the correct explanation of A.

(b) Both A and R are true but R is not the correct explanation of A.

(c) A is true but R is false.

(d) A is false but R is true.

18 Assertion (A): A function 𝑓: 𝑅 → 𝑅, given by 𝑓(𝑥) = 𝑥 2 . Then function 𝑓 is onto 1


Reason (R): A function 𝑔: 𝐴 → 𝐵 is said to be onto if 𝑔(𝐴) = 𝐵 i.e. range of g = B.

19 Assertion (A):If A = {1, 2, 3}, B = {2, 4}, then the number of relation from A to B is equal to 1
64.

Reason (R): The total number of relation from set A to set B is equal to 2 n(A).n(B)
|𝑥|
20 Assertion (A): A function 𝑓: 𝑅 → 𝑅, given by 𝑓(𝑥) = = {1𝑖𝑓 𝑥 > 00𝑖𝑓 𝑥 = 0 − 1𝑖𝑓 𝑥 < 0 1
𝑥
is a bijection
Reason (R): A function 𝑔: 𝐴 → 𝐵 is said to be bijection if it is one-one and onto.

21 Verify whether the relation 𝑅 = {(𝑥, 𝑦): 2𝑥 − 𝑦 = 0} on the set 𝐴 = {1,2,3,4,5,6} is 2


reflexive, symmetric and transitive.

22 Prove that the Greatest Integer Function 𝑓: 𝑅 → 𝑅, 𝑔𝑖𝑣𝑒𝑛 𝑏𝑦 𝑓(𝑥) = [𝑥] is neither one-one 2
nor onto, where [𝑥] denotes greatest integer less than or equal to x.

23 Show that the modulus function 𝑓: 𝑅 → 𝑅, given by 𝑓(𝑥) = |𝑥|, is neither one-one nor onto, 2
where |𝑥| = {𝑥, 𝑖𝑓 𝑥 > 00 𝑖𝑓 𝑥 = 0 − 𝑥 𝑖𝑓 𝑥 < 0.

29
24 Show that the function 𝑓: 𝑅 → 𝑅 defined by 𝑓(𝑥) = 2𝑥 3 + 3 is bijective. 2
25 Let A be the set of all 50 students of class XII in a Kendriya Vidyalaya. Let 𝑓: 𝐴 → 𝑁 be a 2
function defined by
𝑓(𝑥) = 𝑅𝑜𝑙𝑙 𝑛𝑢𝑚𝑏𝑒𝑟 𝑜𝑓 𝑠𝑡𝑢𝑑𝑒𝑛𝑡 𝑥
Show that f is one-one but not onto.

26 A Relation R is defined on a set of real numbers R as R = {(x, y) = x.y is an irrational number}. 3


Check whether R is reflexive, symmetric and transitive or not.

27 State whether the function is one one, onto or bijective. Justify your answer. 3
𝑓: 𝑅 → 𝑅 defined by 𝑓(𝑥) = 3 − 4𝑥.

28 Show that the relation R on the set Z of all integers given by 𝑅 = {(𝑥, 𝑦): |𝑥 − 3
𝑦| 𝑖𝑠 𝑒𝑣𝑒𝑛, 𝑥, 𝑦 𝜖 𝑍 } is an equivalence relation.

29 Show that the function 𝑓: 𝑅 → 𝑅 given by 𝑓(𝑥) = 𝑎𝑥 + 𝑏 where a, b ∈ 𝑅, 𝑎 ≠ 0 is a 3


bijection.

30 Let R be a relation defined on the set of natural numbers N as 3


𝑅 = {(𝑥, 𝑦): 𝑥, 𝑦 𝜖 𝑁, 2𝑥 + 𝑦 = 41}
Verify whether R is (i) reflexive (ii) symmetric (iii) transitive.

31 Students of Grade 11, planned to plant saplings along straight lines, parallel to each other to 4
one side of the playground ensuring that they had enough play area. Let us assume that they
planted one of the rows of the saplings along the line 𝑦 = 𝑥 − 4. Let L be the set of all lines
which are parallel on the ground and R be a relation on L.

Answer the following using the above information.


(i) Let 𝑓: 𝑅 → 𝑅 be defined by 𝑓 (𝑥) = 𝑥 − 4. Find the range of 𝑓(𝑥)
(ii) Let R = {(L 1 , L 2 ) : L 1 is parallel to L 2 and L 1 : y = x – 4} then write of the equation L 2
?
(iii) Let relation R be defined by R = {(𝐿1, 𝐿2): 𝐿1║𝐿2 where L1,L2 € L} then show that R is
equivalence relation.

30
32 Kriti and Kirat are two friends studying in class XII in a school at Chandigarh. While doing 4
their mathematics project on Relations and Functions they have to collect the name of five
metro cities and four cities other than metro cities of India; and present the name of cities in
the form of sets. They have collected the name of cities and write in the form of sets given
as follows:
A={ five metro cities of India}= { Delhi, Mumbai, Bangalore, Calcutta, Pune}
and B = {four non metro cities of India} = { Patiala, Agra, Jaipur, Ahmedabad}

Answer the following questions using the above information.


(i) How many functions exist from A to B.
(ii) Riya wants to know how many relation are possible from A to B.
(iii) Karan wants to know how many reflexive relation on set B.
(iv) How many symmetric relation on set A.
(v) Let R : A → A defined by R = { (x, y) : Total number of vehicles in Delhi(x) is greater
than total number of vehicles in Mumbai(y)}. Show that R is neither reflexive nor symmetric.
33 Aryan visited the Exhibition along with her family. The Exhibition had a huge swing, which 4
attracted many children. Aryan found that the swing traced the path of a Parabola as given
by 𝑦 = 𝑥2.

Answer the following questions using the above information.


(i) Let f: {1,2,3,….}→{1,4,9,….} be defined by 𝑓(𝑥) = 𝑥2. Prove it is bijective.
(ii) Check the function f: Z→Z defined by (𝑥) = 𝑥2 is injective , surjective.
(iii) Let :𝑁 → 𝑅 be defined by 𝑓(𝑥) = 𝑥 2 . Write Range of the function.
(iv) Show that :𝑅 → 𝑅 be defined by 𝑓(𝑥) = 𝑥2 is not one-one.
Let 𝑓: 𝑁 → 𝑁 be defined by (𝑥) = 𝑥2 . Write the domain of f.

31
34 The math teacher of class XII dictates a math problem as follows: 4
' ' Draw the graph of the function, f of x is equal to modulus of x plus three minus one in
the closed interval -3 to +3' '
Three students Rakesh, Sravya and Navya have interpreted the same dictation in three
different ways and they have noted the function as f(x)= |x +3 -1|, f(x)=|x|+3 -1 and
f(x)=|x+3|-1 respectively.
Based on the above information answer the followings:
i Sravya ' s graph is in ' V shape ' with vertex
(a) (-3,1) (b) (3,-1) (c) (-2,0) (d) (0,2)

ii (ii) The function 𝑓(𝑥)= {−𝑥 − 4, 𝑖𝑓𝑥 ≤ −3 }


𝑥 + 2, 𝑖𝑓 𝑥 > −3
is another form of the function of…?
(a) Rakesh (b) Sravya (c) Navya (d) None of them

(iii) Find the domain and range of the function interpreted by Sravya.
(iv) Find the distance between the vertices of the graphs of Rakesh and Navys graphs

35 A function f :X→ Y is said to be one-one if for every x1, x2∈ X, f(x1) = f(x2) 4
⇒ x1 =x2 and f is said to be onto if range of f = Y (codomain of f).
Based on above definition answer the following questions:
(i) The function f:N→N given by f(1)=f(2)=1 and f(x)=x−1 for every x>2 𝑖𝑠
(a) One-one but not onto (b) Onto but not one-one
(c) One-one and onto d)Neither one-one nor onto
(ii) The function f∶R→R defined by f(x)=3x is
(a) One-one but not onto (b) Onto but not one-one
(c) One-one and onto (d) Neither one-one nor onto
The function f:N→N given by f(x)= x2 is
(iii)
(a) One-one but not onto (b) Onto but not one-one
(c) One-one and onto (d) Neither one-one nor onto
(iv) The function f:R→R given by f(x)= x2 is
(a) One-one but not onto (b) Onto but not one-one
(c) One-one and onto (d) Neither one-one nor onto
(v) The function f : N →[3,∞) defined by 𝑓(𝑥)=𝑥2+𝑥+1 is
(a) One-one but not onto (b) Onto but not one-one
(c) One-one and onto (d) Neither one-one nor onto
36 Q19. Prove that a function f: [0,∞) → [−𝟓, ∞) be defined by f(x)= 4x2+4x-5 is bijective.

32
𝒙
37 Q20. Show that the function f:R→ {𝒙 ∈ 𝑹 ∶ −𝟏 < 𝑥 < 1} defined by f(x) = ,x∈ 𝑹 is
𝟏+|𝒙|
one-one onto function.

38 Q21. Show that the relation R on the set A={ 𝒙𝝐𝒁 , 𝟎 ≤ 𝒙 ≤ 𝟏𝟐} ,given by R = {(a,b) , |a-
b|is a multiple of 4} is an equivalence relation. Find the set of all elements related to 1 i.e.
equivalence class [1].

39 Q22. A function f:[-4,4] →[0,4] is given by f(x) =√𝟏𝟔 − 𝒙𝟐 , show that f is a onto function
but not one-one. Find all possible values of “a” for which f(x)=√𝟕

ANSWERS:

1 c
2 b
3 c
4 d
5 c
6 b
7 a
8 b
9 d
10 d
11 b
12 d
13 d
14 a
15 a

33
16 c
17 c
18 d
19 a
20 d
21 Here, 𝑅 = {(1,2), (2,4), (3,6)}
As (1, 1) does not belongs to R, So R is not reflexive.
Again, (1, 2)∈ 𝑅 𝑏𝑢𝑡 (2, 1) does not belongs to R.
So, R is not Symmetric.
Also, (1,2) ∈ 𝑅 𝑎𝑛𝑑 (2, 4) ∈ 𝑅 𝑏𝑢𝑡 (1,4)does not belongs to R.
Therefore R is not transitive.

22 Here, 𝑓(1.2) = [1.2] = 1, 𝑓(1.9) = [1.9] = 1


𝑓(1.2) = 𝑓(1.9), 𝑏𝑢𝑡 1.2 ≠ 1.9
So, f is not one one.
Now, consider 0.7 ∈ 𝑅.
It is known that 𝑓(𝑥) = [𝑥] is always an integer. Thus, there does not exist any element 𝑥 ∈ 𝑅 such
that 𝑓(𝑥) = 0.7.
Therefore, f is not onto.
Hence, the greatest integer function 𝑓(𝑥) = [𝑥] is neither one-one nor onto.

23 Here, 𝑓(−1) = 1 𝑎𝑛𝑑 𝑓(1) = 1.


𝑓(−1) = 𝑓(1), 𝑏𝑢𝑡 − 1 ≠ 1.
So, f is not one one.
Now, consider, −1 ∈ 𝑅.
It is known that 𝑓(𝑥) = |𝑥| is always non-negative.
Thus, there does not exist any element x in domain R such that 𝑓(𝑥) = |𝑥| = −1.
Therefore, f is not onto.
Hence, the modulus function is neither one one nor onto.

34
24 Let, 𝑥1 , 𝑥2 ∈ 𝑅 and 𝑓(𝑥1 ) = 𝑓(𝑥2)
Or, 2𝑥1 3 + 3 = 2𝑥2 3 + 3
Or, 𝑥1 = 𝑥2 .
Hence, the function is one one.
Let, 𝑦 ∈ 𝑅(𝐶𝑜 − 𝑑𝑜𝑚𝑎𝑖𝑛)such that
𝑦 = 𝑓(𝑥)
Or, 𝑦 = 2𝑥 3 + 3
𝑦−3 1
Or, 𝑥 = ( )3 𝜖 𝑅, for all x ∈ 𝑅(𝑑𝑜𝑚𝑎𝑖𝑛).
2

Hence, the function is onto.


Therefore, f is bijective.

25 Here, f associates each students to his (her) roll number. Since no two different students of the class
can have the same roll number. Therefore, ƒ is one-one.

We observe that ƒ (A) = Range of f = {1, 2, 3... 50} ≠N i.e. range of ƒ is not same as its co-domain. So, f is
not onto.

26
Since, (√3, √3) ∉ R
R is not reflexive.

Also, (√3, √2) ∈ R and (√2, 3√3) ∈ R does not implies that (√3, 3√3) ∈ R.

So, R is not transitive.

So, R is Symmetric but neither reflexive nor transitive.

35
27 Let, 𝑥1 , 𝑥2 ∈ 𝑅 and 𝑓(𝑥1 ) = 𝑓(𝑥2)
Or, 3 − 4𝑥1 = 3 − 4𝑥2
Or, 𝑥1 = 𝑥2 .
Hence, the function is one one.
Let, 𝑦 ∈ 𝑅(𝐶𝑜 − 𝑑𝑜𝑚𝑎𝑖𝑛), such that
𝑦 = 𝑓(𝑥)
Or, 𝑦 = 3 − 4𝑥
3−𝑦
Or, 𝑥 = 𝜖 𝑅(𝑑𝑜𝑚𝑎𝑖𝑛)
4

Hence, the function is onto. Hence f is bijective.

28 For all 𝑥 ∈ 𝑍, |𝑥 − 𝑥| = 0, which is even.


Therefore (𝑥, 𝑥) ∈ 𝑅
So, R is reflexive on Z.
Again, let (𝑥, 𝑦) ∈ 𝑅.
Then, |𝑥 − 𝑦| is even.
So, |𝑦 − 𝑥| is even.
So, (𝑦, 𝑥) ∈ 𝑅.
Therefore, R is symmetric on Z.
Let (𝑥, 𝑦)&(𝑦, 𝑧) ∈ 𝑅.
Then, |𝑥 − 𝑦| is even &|𝑦 − 𝑧| is even.
So, |𝑥 − 𝑧| is even.
So, (𝑥, 𝑧) ∈ 𝑅.
Therefore, R is transitive.
So R is an equivalence relation.

36
29 Let, 𝑥1 , 𝑥2 ∈ 𝑅 and 𝑓(𝑥1 ) = 𝑓(𝑥2)
𝑂𝑟, 𝑎𝑥1 + 𝑏 = 𝑎𝑥2 + 𝑏
Or, 𝑥1 = 𝑥2 .
Hence, the function is one one.
Let, 𝑦 ∈ 𝑅(𝐶𝑜 − 𝑑𝑜𝑚𝑎𝑖𝑛), such that
𝑦 = 𝑓(𝑥)
Or, 𝑦 = 𝑎𝑥 + 𝑏
𝑦−𝑏
Or, 𝑥 = ∈ 𝑅(𝑑𝑜𝑚𝑎𝑖𝑛)
𝑎

Hence, the function is onto.


Hence f is bijective.

30 Here,
𝑅 = {(1,39), (2,37) … … … (20,1)}
Here (1, 1) does not belongs to R , so R is not reflexive.
As (1, 39) belongs to R but (39, 1) does not belongs to R.
So R is not symmetric.

31 (i) R, (ii) y= x +c.


32 .(i)1024, (ii)220, (iii) 212, (iv)215.
33 (i) For x1, x2∈ 𝐷𝑜𝑚𝑎𝑖𝑛 and x1≠ x2 implies f(x1) ≠ f(x2)
∴ f is one-one
Also range of f = {1,4,9,….} = co-domaim
∴ f is onto
Hence f is bijective
(ii) f(- 2) = 4 = f(2) implies f is not injective
since range of f = [0, ∞) ≠ co domain Z implies f is not surjective
Hence f is neither injective nor surjective,
(iii) {1,4,9,16…},
(iv) Sa,e as (ii)
(v) N

37
34 The graph of all the three functions are:

By Rakesh By Sravya By Navya

(i) d (ii) c (iii) Domain f = R, Range f = [2,∞)


(iv) Distance =distance between ( -2, 0) and (-3, -1) = √2
35 One-One
Let x1 ,x2 ∈[0,∞) such that x1 ≠ x2

4x12+4x1-5 =4x22+4x2-5

x1 =x2 therefore f is one one

onto :

x ∈[0,∞)
4x2+4x-5≥-5

F(x)≥-5

R(f) =[−5, ∞)

F is onto, f is bijective.

38
36 𝑥
𝑥
𝑖𝑓 𝑥 ≥ 0
f(x) = = {1+𝑥
𝑥
1+|𝑥| 𝑖𝑓 𝑥 < 0
1−𝑥
two cases arise:

(i) x≥0
𝑥
𝑦=
1+𝑥

𝑥 = 𝑦 ; 𝑓 𝑖𝑠 𝑜𝑛𝑒 𝑜𝑛𝑒
𝑥
≥0
1+𝑥
𝑦
𝑥 = 1−𝑦 ≥0 such that f(x)=y
𝑓 𝑖𝑠 𝑜𝑛𝑡𝑜
(ii) x<0
Now we will prove it similarly as above.

37 Reflexivity:
For a ∈ 𝐴, we have
|a-a| =0, which is a multiple of 4
R is reflexive.
Symmetric
Let (a,b) ∈ 𝑅
|a-b| is a multiple of 4
|b-a| will also multiple of 4
(b,a) ∈ 𝑅
R is symmetric.
Transitive:
Let (a,b) ,(b,c) ∈ 𝑅
|a-b| is a multiple of 4
|a-b| =4ʎ , a-b= ±4ʎ
|b-c| is a multiple of 4
|b-c| =4μ , b-c= ±4μ
Therefore a-c= ±4 ± 4μ
(a,c) ∈ 𝑅
R is transitive.
For equivalence class:
|x-1|= 0,4,8,12
X=1,5,9

39
38 y =√16 − 𝑥 2
y2= 16 − 𝑥 2
x =√16 − 𝑦 2clearly for x to be x∈ [−4,4]
16 − 𝑦 2 ≥0
(y-4) (y+4 )≤ 0
0≤y≤4
Therefore it is onto
When x=4 , y=0
When x=-4 ,y=0
So it is not one one
Also—
F(a)= √7
√16 − 𝑎2 = √7
16 − 𝑎2 = 7
a∈ [−3,3]
39 One-One
Let x1 ,x2 ∈[0,∞) such that x1 ≠ x2
4x12+4x1-5 =4x22+4x2-5
x1 =x2 therefore f is one one
onto :
x ∈[0,∞)
4x2+4x-5≥-5
F(x)≥-5
R(f) =[−5, ∞)
F is onto, f is bijective.

CHAPTER-2: INVERSE TRIGONOMETIC FUNCTIONS

Q. QUESTION MARK
NO

1 Which of the following corresponds to the principal value branch of tan –1x? 1
𝜋 𝜋 𝜋 𝜋 𝜋 𝜋
(a)(− , ) (b)[− , ] (c)(− , ) -{0} (d) (0, 𝜋)
2 2 2 2 2 2
2 Which of the following corresponds to the principal value branch of sin–1x? 1
𝜋 𝜋 𝜋 𝜋 𝜋 𝜋
(a)(− , ) (b)[− , ] (c)(− , ) -{0} (d) (0, 𝜋)
2 2 2 2 2 2
3 Which of the following corresponds to the principal value branch of cos –1x? 1
𝜋 𝜋 𝜋 𝜋 𝜋 𝜋
(a)(− , ) (b)[− , ] (c)(− , ) -{0} (d) [0, 𝜋]
2 2 2 2 2 2
4 Which of the following corresponds to the principal value branch of cot –1x? 1
𝜋 𝜋 𝜋 𝜋 𝜋 𝜋
(a)(− , ) (b)[− , ] (c)(− , ) -{0} (d) (0, 𝜋)
2 2 2 2 2 2
5 Which of the following corresponds to the principal value branch of sec –1x? 1
𝜋 𝜋 𝜋 𝜋 𝜋 𝜋
(a)(− , ) (b)[− , ] (c)(− , ) -{0} (d) [0, 𝜋] -
2 2 2 2 2 2
6 Which of the following corresponds to the principal value branch of cosec –1x? 1
𝜋 𝜋 𝜋 𝜋 𝜋 𝜋
(a)(− , ) (b)[− , ] -{0} (c)(− , ) (d) [0, 𝜋]
2 2 2 2 2 2
7 Write the value of cot (tan–1a + cot–1a). 1
(a)1 (b)2 (c) 0 (d) -1

40
8 The domain of the function cos–1 (2x – 1) is 1
(a) [0, 1] (b) [–1, 1] (c) ( –1, 1 ) (d) [0, π]
9 The domain of the function defined by f (x) = sin–1 x - 1 is 1

[1,2] (b) [−1,1] (c) [0,1] (d) None of these

10 The domain of sin–1 2x is 1


1 1
(a)[0,1] (b) [−1,1] (c) [− 2 , 2] (d) [−2 ,2]

11 If 3tan–1 x + cot–1 x = π, then x equals


(a) 0 (b) 1 (c) –1 (d) 1

12 The value of cos–1( cos3π/2 ) is 1


(a) π/2 (b) 3π/2 (c) 5π/2 (d) 7π/2
13 Domain of cos-1x is 1
(a) [0,1] (b) [−1,1] (c) [−1 ,1] (d) None of these

14 Domain of sec-1x is 1
(a) [−1,1] (b) R -{0} (c) R -{−1,1} (d) R- [−1,1]

15 If x≠ 0 then cos (tan-1x + cot-1x) =? 1


(a) -1 (b) 1 (c) 0 (d) None of these

16 Find the principal value of cosec-12 ? 2

17 Find the principal value of sin-1(-1/2)? 2

18 Find the value of tan-1(1) + sin-1(-1/2). 2

19 Find the value of tan-1(√3) + sin-1(-1/2). 2

20 Find the value of cos-1(cos 2π/3). 2

21 Find the value of cosec( sin-1x + cos -1x ) . 2

22 Write the principal value branch of sin-1x.Also write the other branches? 2

23 Draw the graph of sin-1x ? 2

24 Draw the graph of cos-1x ? 2

25 Find the value of cos-1(cos 7π/6). 2

Q. ANSWER MARK
NO S

1 (a) 1

2 (b) 1

3 (d) 1

4 (d) 1

5 (d) 1

41
6 (b) 1

7 (c) As cot (π/2) =0, we know that cot-1a + tan-1a = π/2 1

8 (a) -1≤2x-1 ≤1,0≤2x≤2 1


i.e 0≤x≤1
therefore xϵ[0,1]

9 (a) 0≤x-1 ≤1 and -1 ≤ x - 1≤1 1

Therefore 1≤ x≤2

10 (c) 1

11 (d) 2tan-1x + tan-1x +cot-1x =π


2tan-1x = π – π /2
2tan-1x = π/2
tan-1x = π /4 , x= tan π /4
x=1

12 (a) Cos-1(cos(2π-π/2 )) = cos-1(cosπ/2) =π/2 1

13 (b) 1

14 (c) 1

15 (c) 1

16 Let cosec-12 = α 1
Cosec α =2 = cosec π/6 1
α= π/6 ϵ [−𝜋/2, 𝜋/2] -{0}

17 Let sin-1(-1/2) = α 1
Sinα= -1/2= sin( -π/6) 1
𝜋 𝜋
α= -π/6 ϵ [− 2 , 2 ]

18 As we already know tan-1(1) = π/4ϵ [0, 𝜋] and sin-1(-1/2)= -π/6 1


Therefore 1
Value= π/4 + (-π/6) =π/12

42
19 𝜋 𝜋 1/2
As we know that tan-1(√3) = π/3 ϵ (− 2 , 2 )

andLet sin-1(-1/2) = α
1/2
Sinα= -1/2= sin( -π/6)
𝜋 𝜋
α= -π/6 ϵ [− 2 , 2 ]
1
therefore value= π/3+(-π/6) = π/6

20 cos-1(cos 2π/3) = 2π/3 1+1


since 2π/3 ϵ[0, 𝜋]
𝜋
21 As we know that sin-1x + cos -1x = 2 1+1

Coesc π/2 = 1

22 𝜋 𝜋 1
The principal value branch of sin-1x =[− 2 , 2 ]
𝜋 3𝜋 3𝜋 5𝜋 3𝜋 𝜋 5𝜋 1
And other branches are [2 , ] ,[ 2 , ] ,--------- and [− , − 2 ] ,[− , −3]
2 2 2 2

23 2

43
24 2

25 cos-1(cos 7π/6) =cos-1( cos (2π -5π/6)) 1+1


cos-1( cos 5π/6) = 5π /6

CHAPTER-3: MATRICES

Q. NO QUESTION MARK

1. 0 2 −3 1
𝐼𝑓𝐴 [−2 0 −1] then A is a
3 1 0
(a) skew - symmetric matrix (b) symmetric matrix
(c) none of these (d) diagonal matrix

44
2. 1 2 𝑥 1 −2 𝑦 1
If A =[0 1 0] and B = [0 1 0] and AB = l 3 , then x + y equals
0 0 1 0 0 1
(a) – 1 (b) 0 (c) none of these (d) 2
3. If A =[0 1] , then A 2 is equal to 1
1 0

(a) [1 1] (b) [0 1] (c) [1 0] (d) [1 0]


1 0 1 0 1 0 0 1
4. 2
0 −3 1
If A =[ 43 1 ] is expressed as the sum of a symmetric and skew - symmetric matrix,
−5 7 2
then the symmetric matrix is
2 2−4 2 4 −5
(a) [ 2 34 ] (b) [ 0 3 7]
−4 4 2 −3 1 2
4 4 −8 1 0 0
(c) [ 4 6 8 ] (d) [0 1 0]
−8 8 4 0 0 1

5. If A =[1 −1] , B = [𝑎 1
] and (A + B) 2 = A 2 + B 2 , then values of a and b are 1
2 −1 𝑏 −1
(a) a = 0, b = 4 (b) a = 1, b = 4 (c) a = 2, b = 4 (d) a = 4, b = 1

6. If A and B are matrices of same order, then (AB’ – BA’) is a 1


(a) null matrix (b) unit matrix
(c) symmetric matrix (d) skew - symmetric matrix

7. The number of all possible matrices of order3 × 3 with each entry 0 or1 is 1
(a) 81 (b) none of these (c) 512 (d) 18

8. If A and B are two matrices of the order 3× m and 3 × n, respectively, and m = n, then 1
the order of matrix (5A – 2B) is
(a) 3× 3 (b) m× n (c) 3× n (d) m× 3

9. Matrices A and B will be inverse of each other only if 1


(a) AB = BA (b) AB = BA = 0 (c) AB = 0, BA = I (d) AB = BA = I

10. If A and B are symmetric matrices, then ABA is


(a) symmetric matrix (b) diagonal matrix 1
(c) skew - symmetric matrix (d) scalar matrix

11. 𝑥 𝑦 8 𝑤 2
If[𝑧 + 6 𝑥 + 𝑦] = [ ] , then find values of x, y, z and w.
0 6
12. 1 2 4 0 2 0 2
Let A =[ ], 𝐵=[ ], 𝐶=[ ] Show that (AB) ′ = B ′ A ′
−1 3 1 5 1 −2

45
13. 8 0 2−2
If A =[4 −2] and B = [ 4 2 ],
3 6 −5 1 2
then find the matrix X of order 3 × 2 such that 2A + 3X = 5B.

14. 3 4 2
If 𝐴 = [ ] , 𝑓𝑖𝑛𝑑 𝐴 + 𝐴′ , where 𝐴′ is the transpose of matrix A.
2 3

15. 1 2 0 0 2
For what values of x:[121] [2 0 1] [2] = 0.
1 0 2 𝑥
16. A manufacture produces three stationery products Pencil, Eraser and Sharpener which 4
he sells in two markets. Annual sales are indicated below

Market Products (in numbers)

Pencil Eraser Sharpener

A 10000 2000 1800

B 6000 20000 8000

If the unit Sale price of Pencil, Eraser and Sharpener are Rs. 2.50, Rs. 1.50 and Rs. 1.00
respectively, and unit cost of the above three commodities are Rs. 2.00, Rs. 1.00 and Rs.
0.50 respectively, then using matrix multiplication answer the following:
(1)
1. Find Total revenue of market A
(1)
2. Find Total revenue of market B
(2)
3. Find Gross profit in both market

46
17. One Kendriya Vidyalaya decided to organize a fair for collecting money for helping the 4
flood victims. For this the Vidyalaya gave task to the students of three Classes X, XI & XII
They sold handmade fans, mats and plates from recycled material at a cost of Rs. 25,
Rs.100 and Rs. 50 each respectively.

The numbers of articles sold are given as

Classes/ Class X Class XI Class XII


Articles

Fans 40 25 35

Mates 50 40 50

Plates 20 30 40

Based on the information given above and using matrix multiplication answer the
following questions: (1)
1. What is the total money (in Rupees) collected by the class X? (1)
2. What is the total amount of money (in Rs.) collected by Class XI? (2)
3. What is the total amount of money collected by the Kendriya Vidyalaya ?

47
ANSWERS:
Q. NO ANSWER MARKS
1 (a) skew - symmetric matrix 1
2 (b) 0 1
3 (d) [1 0] 1
0 1

4 2
2 −4 1
(a) [ 23 4]
−4 4 2

5 (b) a = 1, b = 4. 1
6 (d) skew - symmetric matrix. 1
7 (c) 512 1
8 (c) 3× n 1
9 (d) AB = BA = I 1
10 (a) symmetric matrix 1
11 x = 4, y = 2, z = - 6, w =4. 2
12 (AB)’ = [ 6 −1
] 2
10 15

B’A’ = [4 1] [1 −1]= [ 6 −1
]
5 0 2 3 10 15

13 X = [−2 − 10/3414/3 − 31/3 − 7/3] 2


14 𝐴 + 𝐴′ = [6 6] 2
6 6

15 x = - 1. 2
16 1. Rs. 46,000 1
2. Rs. 53,000 1
3. Rs. 32,000. 2
17 1. (b) 7000 1
2. (a) 625. 1
3. (c) Rs.21000 2

48
CHAPTER-4: DETERMINANTS
Q. QUESTION MARK
NO
1 If A is square matrix of order 3 such that|𝑎𝑑𝑗 𝐴| = 64, find|𝐴|. 2

2 If 𝐴 = [3425], then find the determinant of the matrix 𝐴2 − 𝐴. 2

3 Find the value of p, so that the area of a triangle is 16 sq. units and vertices are(0, 2
𝑝), (0, 4) 𝑎𝑛𝑑 (−2, 0).

4 Find the value of 𝜆 so that the points (1, −5), (−4, 5) 𝑎𝑛𝑑 (𝜆, 7) are collinear 2
5 5 3 2
If 𝐴 = [ ], then show that |5𝐴| = 25|𝐴|.
10 −2

6 For what value of x, the matrix [5 − 𝑥 𝑥 + 1] is singular? 2


2 4

7 Find the value of x if |2 4| = |2𝑥 4| 2


5 1 6 𝑥

8 Find the equation of line joining (1,2) and (3,6) using Determinants. 2
9 2 −3 2
Given A=[−4 ], then compute 𝐴−1 .
7

10 4 −6 1 2
Find the co factors of the matrix A=[−1 −1 −1]
−4 11 −1

11 A school wants to awards its students for the values of Honesty, Regularity and Hard 4
work with a total cash award of Rs. 6000. Three times the award money for Hard work
added to that given for Honesty amounts to Rs. 11000. The award money given for
Honesty and Hard work together is double the one given for Regularity. Represent the
above situation algebraically and find the award money for each value, using matrix
method.

49
12 On her birth day, Seema decided to donate some money to children of an orphanage home. If 4
there were 8 children less, everyone would have got Rs.10 more. However, if there were 16
children more, everyone would have got Rs. 10 less. Let the number of children be x and the
amount distributed by Seema for one child be y (in Rs.)

Based on the information given above, answer the following questions:

(i). write above situation in equations in terms x and y

(ii) Which of the following matrix equations represent the information given above?

(iii)find number of children who got some money from Seema.

(iv) How much amount is given to each child by Seema?

13 Manjit wants to donate a rectangular plot of land for a school in his village. When he 4
was asked to give dimensions of the plot, he told that if its length is decreased by 50 m
and breadth is increased by 50m, then its area will remain same, but if length is
decreased by 10m and breadth is decreased by 20m, then its area will decrease by 5300
m2

Based on the information given above, answer the following questions(any four ):
1. The equations in terms of X and Y are
a. x-y=50, 2x-y=550 b. x-y=50, 2x+y=550
c. x + y = 50, 2x + y=550 d. x +y = 50, 2x + y=550
2.write correct equation of step 1 in matrix form.

3. The value of x (length of rectangular field) is ______

4. The value of y (breadth of rectangular field) is ________

5. How much is the area of rectangular field?

50
14 In a survey of 20 richest persons of three residential society A,B,C it is found that in 4
society a 5 believes in honesty ,10 believed in hard work ,5 in unfair means while in B 5
in believes in honesty ,8 in hard work ,7 in unfair means and in C, 6 in honesty ,8 in
hard work ,6 in unfair means. If the per day income of 20 richest persons of society
A,B,C are RS 32,500, Rs 30,500 ,Rs 31,000 respectively . on the basis of this information
answer the following questions
(i) Represent this information in Matrix form
(ii)What is the adjoint of the given matrix found in above question
(iii) Find the inverse of Matrix
(iv) Find the day income of people who believe in honesty by matrix

15 Two schools A and B decided to award prizes to their student for three values honesty 4
(x) ,punctuality (y) and obedience (z) . School A decided to award a total of Rs 11000 for
these three values to 5,4and 3 students respectively while school B decided to award a
total of Rs 10,700 for these three values to 4,3and 5 students respectively. If all the
three prizes together amount to Rs 2700. Answer the following questions on this
information.
(i) Represent the above situation by matrix equation and form form linear equation by
using matrix multiplication.
(ii) What is determinant value of the matrix obtained in que (i)
(iii) Is it possible to solve the system of equation so obtained using matrices?
(iv) If a determinant value of a matrix is zero then such matrix is called :

16 Solve the system of equations by matrix method: 5


𝑥 − 𝑦 + 𝑧 = 4 ; 2𝑥 + 𝑦 − 3𝑧 = 0; 𝑥 + 𝑦 + 𝑧 = 2.

17 2 2 −4 1 −1 0 5
Given A=[−4 2 −4] and B=[2 3 4] ,then find BA and use this to solve the system
2 −1 5 0 1 2
of equations

𝑥 − 𝑦 = 3 ; 2𝑥 + 3𝑦 + 4𝑧 = 17 ; 𝑦 + 2𝑧 = 7
18 1 2 1 5
If A=[−1 1 1], find 𝐴−1 .Hence solve the system of equation.
1 −3 1
x-y+2z=7 ,3x+4y-5z=-5,2x-y+3z=12.

19 If A= [ 31
] , show that 𝐴2 − 5𝐴 + 7𝐼 = 0. Hence find𝐴−1 . 5
−1 2

20 Using matrix ,solve the following system of linear equations: 5


𝑥 − 𝑦 + 2𝑧 = 7; 3𝑥 + 4𝑦 − 5𝑧 = −5; 2𝑥 − 𝑦 + 3𝑧 = 12

51
21 Solve the following equations by using matrix method
1 1 1 3 3 2 1 1 2
+ + =12, 𝑣+𝑤+𝑢=33, 𝑢+𝑤=𝑣
𝑢 𝑣 𝑤

Q. ANSWER MARK
NO S
1 Since order of matrix is 3 so |𝑎𝑑𝑗 𝑎| = |𝐴|2 =64 1
Thus |𝐴| = ±8 1
2 𝐴 = [3425] so 𝐴2 = [3425][3425] = [17321633]
Now 𝐴2 − 𝐴 = [17321633] − [3425] = [14281428] 1
|𝐴2 − 𝐴| = (14 × 28 − 14 × 28)=0

1
3 Are of triangle of vertices (0,p),(0,4)and (2,0) =16 sq unit
1
So 2 |0𝑝1041 − 201| = 16 or -2(p-4)=±32 1
P=∓12

1
4 Since points (1, −5), (−4, 5) 𝑎𝑛𝑑 (𝜆, 7) are collinear
So area of trigle form by these points are zero 1
1
So 2 [1 − 51 − 451 𝜆 71] = 0

1(5-7)+5(-4- 𝜆)+(-28-5 𝜆)=0 or 𝜆 = −5

1
5 𝐴 = [5 310 − 2], 5A=[25 1550 − 10]
LHS=|5𝐴|=(-250-750)=-1000 1
RHS= 25IAI=25× |5 310 − 2|=25× (-10-30)=-1000

Hence |5𝐴| = 25|𝐴|

52
6 Since [5 − 𝑥𝑥 + 124] is singular
So |5 − 𝑥𝑥 + 124| = 0 or 4(5 − 𝑥) − 2(𝑥 + 1) = 0 1
x=3 1
7 |2451| = |2𝑥46𝑥|= (2-20)=2𝑥 2 − 24 1
𝑥2 = 3 or 𝑥 = ±√3 1
8 Equation of line passing through (𝑥1 , 𝑦1 ) and (𝑥2 , 𝑦2 ) is
|𝑥𝑦1𝑥1 𝑦1 1𝑥2 𝑦2 1| = 0

Or |𝑥𝑦1121361| = 0
Or 𝑥(2 − 6) − 𝑦(1 − 3) + 1(6 − 6) = 0
−4𝑥 + 2𝑦 = 0 𝑜𝑟 2𝑥 − 𝑦 = 0 is required line.
1

1
9 A=[2 − 3 − 47] then |𝐴| = (14 − 12) = 2
adj A =[2 − 3 − 47] = [7 342] 1
𝑎𝑑𝑗 𝐴 1
𝐴−1 = = [7342]=[7/23/221]
|𝐴| 2

1
10 𝐴11 = (−1)1+1 |−1111 − 1| = −10 , 𝐴12 = (−1)1+2 |−11 − 4 − 1| = −5 2
𝐴13 = (−1)1+3 |−1 − 1 − 411| = −15, 𝐴21 = (−1)2+1 |−6111 − 1| = 5,𝐴22 =
(−1)2+2 |41 − 4 − 1| = 0 , 𝐴23 = (−1)2+3 |4 − 6 − 411| = −20
𝐴31 = (−1)3+1 |−61 − 11| = −5 ,𝐴32 = (−1)3+2 |41 − 11| = −5
𝐴33 = (−1)3+3 |4 − 6 − 1 − 1| = −10

11 x=500 ,y= 2000 and z= 3500 1+1+1


+1

12 (i). (a) 5x-4y = 40 5x-8y = -80 1


(ii). (c). [5 − 45 − 8][𝑥𝑦] =[40 − 80]
1
(iii). (d) 32
1
(iv) . (b) Rs.30
1

53
13 1. b) x-y=50, 2x+y=550 1
2.(a). [1 − 121][𝑥𝑦] =[50550] 1
3. c) 200m
1
4. a) 150m
1
5. b) 30000Sq.m

14 (i) (a) 1
(ii) (d) 1
(iii) (c)
1
(iv) (c)
1
15 1
(i) (c)
1
(ii) (b) -3
1
(ii (a) yes
1
(iv) (a) singular

16 x=2 ,y= -1 and z= 1

17 x=12 ,y=-1 and z=4

18 x=2 ,y=0 ,z=2

19 Find 𝐴2 1
Put all values in LHS =𝐴2 − 5𝐴 + 7𝐼 to prove. 2
Hence 𝐴2 − 5𝐴 + 7𝐼 = 0 .Now multiplying by 𝐴−1 on both side we get
(𝐴−1 𝐴)𝐴 − 5𝐴−1 − 7𝐼𝐴−1 = 0
1
−1 −1
𝐼𝐴 − 5𝐼 + 7𝐴 = 0 or 7𝐴 = 5𝐼 − 𝐴
1 2 −1
𝐴−1 = [2 − 113] [ ] 1
7 1 3

20 x=2 ,y=1 ,z= 3

21 x=3, y=4, z=5

CHAPTER-5:

CONTINUITY AND DIFFERENTIABILITY

54
Q. QUESTION MARK
NO
Choose the correct answers from the given four options in each of the
questions from 1 to 10.
1 𝑥2 1
If 𝑓(𝑥) = 2𝑥 and 𝑔(𝑥) = + 1, then which of the following can be a
2
discontinuous function :
(a) 𝑓(𝑥) + 𝑔(𝑥) (b) 𝑓(𝑥) − 𝑔(𝑥)
(c) 𝑓(𝑥). 𝑔(𝑥) 𝑓(𝑥)
(d)
𝑔(𝑥)

2 The set of points where the function 𝑓 given by 𝑓(𝑥) = |2𝑥 + 1| is 1


differentiable is :
1
(a) R (b) R – { }
2
1
(c) R – {− } (d) (0, ∞)
2

3 The function 𝑓(𝑥) =𝑐𝑜𝑡 𝑐𝑜𝑡𝑥 is discontinuous on the set 1


(a) {𝑥 = 𝑛𝜋 ∶ 𝑛 ∈ 𝑍} (b) {𝑥 = 2𝑛𝜋 ∶ 𝑛 ∈ 𝑍}
𝜋 𝑛𝜋
(c) {𝑥 = (2𝑛 + 1) ∶ 𝑛 ∈ 𝑍} (d) {𝑥 = ∶ 𝑛 ∈ 𝑍}
2 2

4 The function 𝑓(𝑥) = 𝑒 |𝑥| is : 1

(a) continuous everywhere but not differentiable at 𝑥 = 0.


(b) continuous and differentiable everywhere.
(c) not continuous at 𝑥 = 0.
(d) none of these.
5 𝑑𝑦 1
If 𝑥 = 𝑡 3 , 𝑦 = 𝑡 2 , then is
𝑑𝑥
2 3
(a) (b)
3𝑡 2𝑡
3𝑡
(c) (d) can not be determined
2

6 If = 𝑎 𝑠𝑖𝑛 𝑠𝑖𝑛𝑥 + 𝑏 𝑐𝑜𝑠 𝑐𝑜𝑠𝑥 , then out of the following which one is correct? 1
𝑑𝑦 𝑑𝑦
(a) −𝑦 =0 (b) +𝑦=0
𝑑𝑥 𝑑𝑥

𝑑2𝑦 𝑑2𝑦
(c) 2
−𝑦 =0 (d) +𝑦 =0
𝑑𝑥 𝑑𝑥 2

55
7 For the function 𝑓(𝑥) = {1, 𝑤ℎ𝑒𝑛 𝑥 > 0 0, 𝑤ℎ𝑒𝑛 𝑥 = 1 − 1, 𝑤ℎ𝑒𝑛 𝑥 < 0 1
point/points of discontinuity is/are :
(a) All 𝑥 ∈ {R – {0}} (b) All 𝑥 ∈ 𝑅+
(c) {0} (d) All 𝑥 ∈ 𝑅−
8 𝜋
For the function 𝑓(𝑥) = |𝑠𝑖𝑛 𝑠𝑖𝑛𝑥 −𝑐𝑜𝑠 𝑐𝑜𝑠𝑥| , ∀ 𝑥 ∈ [0, ] the point where it 1
2
is not derivable is/are :
𝜋
(a) 𝑥 = (b) 𝑥 = 0
2
𝜋 𝜋
(c) 𝑥 = (d) 𝑥 =
6 4

9 Find 𝑓 ′ (𝑥) , for the function :𝑓(𝑥) = 𝑒 𝑥 1


𝑥 𝑥
(a) √ 𝑒 (b) √ 𝑒2
1+𝑥2 𝑥 −1
𝑒𝑥
(c) √ (d) none of these
1−𝑥2

10 𝑑2𝑦 1
If 𝑥 = 𝑡 3 , 𝑦 = 𝑡 2 , then is
𝑑𝑥 2
2 9𝑡 4
(a) (b)
9𝑡 4 2
−2
(c) (d) none of these
9𝑡 4

11 Check the continuity at point 𝑥 = 0 for the function given by 2


|𝑥|
𝑓(𝑥) = { , 𝑖𝑓 𝑥 ≠ 00, 𝑖𝑓 𝑥 = 0.
𝑥
12 Find the relationship between a and b so that the function f defined by 2
𝑓(𝑥) = {𝑎𝑥 + 1, 𝑖𝑓 𝑥 ≤ 3𝑏𝑥 + 3, 𝑖𝑓 𝑥 > 3 is continuous at 𝑥 = 3.
13 Find the values of k so that the function f given by 𝑓(𝑥) = {𝑘𝑥 2 , 𝑖𝑓 𝑥 ≤ 2
23, 𝑖𝑓 𝑥 > 2 at 𝑥 = 2.
14 Differentiate 𝑓(𝑥) =𝑐𝑜𝑠 𝑐𝑜𝑠(𝑠𝑖𝑛 𝑠𝑖𝑛𝑥) with respect to 𝑥. 2
15 𝑑𝑦 2
Find, , if 𝑦 +𝑠𝑖𝑛 𝑠𝑖𝑛𝑦 =𝑐𝑜𝑠 𝑐𝑜𝑠𝑥.
𝑑𝑥

16 𝑑𝑦 2
Find, , if 𝑥 = 𝑎 𝑐𝑜𝑠 𝑐𝑜𝑠𝑡, 𝑦 = 𝑎 𝑠𝑖𝑛 𝑠𝑖𝑛𝑡.
𝑑𝑥
√1+𝑥 − √1−𝑥 𝑑𝑦
17 If y = sin-1{ } , then show that = 1/2(1-x2)1/2 . 3
2 𝑑𝑥

56
𝜋
18 For what value of k is the following function continuous at x = 4 ? 3
𝑠𝑖𝑛 𝑥 − 𝑐𝑜𝑠 𝑥 𝜋 𝜋
f(x ) = { 𝑥−4
𝜋 ,𝑥 ≠ 4
𝑘 , 𝑥 = 4

𝑑𝑦
19 If y = ae2x – be-x ,then show that d2y/dx2 – 𝑑𝑥 – 2y = 0. 3

𝑑𝑦
20 If (cos y)x = ( sin x )y then find 𝑑𝑥 . 3
𝑑𝑦 𝜋
21 Find 𝑑𝑥 at x = 2, y = 8 , if sin2 y + cos xy = k. 3
𝑐𝑜𝑠 𝑥 − 𝑠𝑖𝑛 𝑥
22 Differentiate cot-1 (𝑐𝑜𝑠 𝑥 + 𝑠𝑖𝑛 𝑥 ) with respect to x. 3

23 Prove that the greatest integer function defined by f(x) = [x], 0 < x< 2 is not differentiable at 3
x = 1.
𝑑𝑥
24 If x √1 + 𝑦 - y √1 + 𝑥 = 0 and x ≠ y then prove that = -1/(1+y)2 . 3
𝑑𝑦

25 If y = (cos-1x)2 , prove that (1 – x2 )y’’- x y’ – 2 = 0 3


𝑑𝑦 𝑥+𝑦
26 If x2 + y2 = 2 cot-1x , then show that 𝑑𝑥 = 𝑦 − 𝑥. 3

27 A potter made a mud vessel , where the shape of the pot is based on f(x) = I x – 1 I + I x – 4 I
, where f(x) represents the height of the pot.

1
(i) When x < 1 what will be the height in terms of x ?
(a ) 5 -2x (b) 2x – 3 ( c ) 2x – 4 ( d) 2x – 5
1
𝑑𝑦
(ii) What is 𝑑𝑥 at x = 1?

(a) 1 (b) 2 (c ) -2 (d) f(x) is not differentiable at x = 1. 2


(iii)What is the value of f’(x) if1 ≤ 𝑥 < 4 ?

Or, What is the value of f’(x) if𝑖𝑓 𝑥 ≥ 4 ?

57
28 Read the following text and answer the following questions on the basis of the same. 1+1+2
Mr. Punit of a School teacher is teaching chain rule to his students with the help of flow-
chart. The chain rule says that if h and g are functions and f(x) = g(h(x)))’, then f’(x) =
(g(h(x)))’ = g’(h(x)) h’(x)
(i) Derivative of sinx5 is
(a) x4 cos x5 (b) 5x4 cos x5 (c)-5 x4 cos x5 (d)4 x4 cos x5
(ii) Derivative of cos (sin x) is
(a) cos x cos (sin x) (b) – cos x sin(sin x)
( c) sin x cos (sin x) (d ) - sin x cos (sin x)
𝑑 𝜋
(iii) Find 𝑑𝑥 (sin 2x ) at x= 4 .
𝑑
Or, Find 𝑑𝑥 (sin e2x )

29 Read the following text and answer the following questions on the basis of the same.
Manisha started to read the topic ‘differentiability’ which she has prepared in the class of 2+2
mathematics . She wanted to solve the questions based on this topic , which teacher gave
as home work . She has written following matter in her notes:
Let f(x) be a real valued function, then its Left Hand Derivative is
𝑓(𝑎−ℎ) − 𝑓(𝑎)
Lf’(a) = −ℎ

𝑓(𝑎+ℎ) − 𝑓(𝑎)
Rf’(a) = ℎ

Also a function f(x) is said to be differentiable at x = a , if its LHD and RRHD at x = a exist and
one equal.
13 3𝑥
For the function , f(x) = {𝐼 𝑥 − 3 𝐼, 𝑥 ≥ 1 4
− 2
, 𝑥 < 1

(i) Find the value of f’(-2).


(ii) Find the value of f’(3).

58
30 The function f (x) will be discontinuous at x = a if f(x) has
● Discontinuity of first kind :𝑓(𝑎 − ℎ) and 𝑓(𝑎 + ℎ) both exist but are not equal. It is
also known as irremovable discontinuity.
● Discontinuity of second kind : If none of the limits 𝑓(𝑎 − ℎ) and 𝑓(𝑎 + ℎ) exist.
● Removable discontinuity ::𝑓(𝑎 − ℎ) and 𝑓(𝑎 + ℎ) both exist and equal but not
equal to f(a).
Based on the above information , answer the following questions.
(i) If f(x) = {𝑥 + 3 , 𝑓𝑜𝑟 𝑥 ≠ 34 , 𝑓𝑜𝑟 𝑥 = 3 , then at x = 3
(a)f has removable discontinuity (b)f is continuous.
(c ) f has irremovable discontinuity (d)None of these
(ii) If f(x) = {𝑥 + 2 , 𝑓𝑜𝑟 𝑥 ≤ 4𝑥 + 4 , 𝑓𝑜𝑟 𝑥 > 4 , then at x = 4
(a)f has removable discontinuity (b)f is continuous.
(c ) f has irremovable discontinuity (d)None of these
(iii) If f(x) = {𝑥 + 3 , 𝑓𝑜𝑟 𝑥 ≠ 3𝑘 , 𝑓𝑜𝑟 𝑥 = 3
defined as continuous at x = 3 then find the value of k.
𝑘𝑥
Or, If f(x) = {𝐼 𝑥 𝐼 , 𝑓𝑜𝑟 𝑥 < 03 , 𝑓𝑜𝑟 𝑥 ≥ 0

defined as continuous at x = 0 then find the value of k.

31 If 𝑦 = 𝑒 𝑎𝑥 , −1 ≤ 𝑥 ≤ 1, 5

𝑑2𝑦 𝑑𝑦
Show that (1 − 𝑥 2 ) − 𝑥. − 𝑎2 𝑦 = 0.
𝑑𝑥 2 𝑑𝑥

32 If 𝑐𝑜𝑠 𝑐𝑜𝑠𝑦 = 𝑥 𝑐𝑜𝑠 𝑐𝑜𝑠(𝑎 + 𝑦), with 𝑐𝑜𝑠 𝑐𝑜𝑠𝑎 ≠ ±1, prove that 5

𝑑𝑦 (𝑎 + 𝑦)
=
𝑑𝑥 𝑠𝑖𝑛 𝑠𝑖𝑛𝑎
𝑑𝑣 5
33 If ( 1- u 2 )1/2 + ( 1- v 2 )1/2 = a (u – v), then prove that 𝑑𝑢 = ( 1- v 2 )1/2 / ( 1- u 2 )1/2 .
𝜋
34 If x = a tan 3 θ , y = a sec3 θ, then find d2y/dx2 at θ = . 5
3

Q. NO ANSWER MA
RKS
1 𝑓(𝑥) 1
(d)
𝑔(𝑥)

1
2 (c) R – {− } 1
2

3 (a) {𝑥 = 𝑛𝜋 ∶ 𝑛 ∈ 𝑍} 1

59
4 (a) continuous everywhere but not differentiable at 𝑥 = 0. 1
2
5 (a) 1
3𝑡

6 𝑑2𝑦 1
(d) +𝑦 =0
𝑑𝑥 2

7 (c) {0} 1
𝜋
8 (d) 𝑥 = 1
4
𝑥
9 (c) √ 𝑒 1
1−𝑥2
−2
10 (c) 1
9𝑡 4
−𝑥
11 The function f can be defined as 𝑓(𝑥) = { = −1, 𝑤ℎ𝑒𝑛 𝑥 < 00, 𝑤ℎ𝑒𝑛 𝑥 =
𝑥
𝑥
0 = 1, 𝑤ℎ𝑒𝑛 𝑥 > 0
𝑥

Clearly, LHL of 𝑓(𝑥) at 𝑥 = 0 is given by, 𝑓(𝑥) = (−1) = −1 1


2
Also, RHL of 𝑓(𝑥) at 𝑥 = 0 is given by, 𝑓(𝑥) = (1) = 1.
Clearly, LHL of 𝑓(𝑥) at (𝑥 = 0) ≠ 𝑅𝐻𝐿 𝑜𝑓 𝑓(𝑥) 𝑎𝑡 (𝑥 = 0)
1
So that, 𝑓(𝑥) does not exist.
hence, f is not continuous at 𝑥 = 0.
1
2
12 Clearly, LHL of 𝑓(𝑥) at 𝑥 = 3 is given by, 𝑓(𝑥) = (𝑎𝑥 + 1)
= 3𝑎 + 1
Also, RHL of 𝑓(𝑥) at 𝑥 = 3 is given by, 𝑓(𝑥) = (𝑏𝑥 + 3) = 3𝑏 + 3. 1
As, f is continuous at 𝑥 = 3 hence,
LHL of 𝑓(𝑥) at (𝑥 = 3) = 𝑅𝐻𝐿 𝑜𝑓 𝑓(𝑥) 𝑎𝑡 (𝑥 = 3) = 𝑓(3)
2
⟹ 3𝑎 + 1 = 3𝑏 + 3 ⟹ 𝑎 − 𝑏 = .
3
1

60
13 Clearly, LHL of 𝑓(𝑥) at 𝑥 = 2 is given by, 𝑓(𝑥) = (𝑘𝑥 2 ) 1
= 4𝑘
Also, RHL of 𝑓(𝑥) at 𝑥 = 2 is given by, 𝑓(𝑥) = (3) = 3.
As, f is continuous at 𝑥 = 2 hence,
LHL of 𝑓(𝑥) at (𝑥 = 2) = 𝑅𝐻𝐿 𝑜𝑓 𝑓(𝑥) 𝑎𝑡 (𝑥 = 2) = 𝑓(2)
3
⟹ 4𝑘 = 3 ⟹ 𝑘 = . 1
4

14 Here, 𝑓(𝑥) =𝑐𝑜𝑠 𝑐𝑜𝑠(𝑠𝑖𝑛 𝑠𝑖𝑛𝑥)


differentiating with respect to 𝑥 and applying chain rule we get,
𝑑𝑓(𝑥) 𝑑𝑐𝑜𝑠𝑐𝑜𝑠(𝑠𝑖𝑛𝑠𝑖𝑛𝑥) 𝑑𝑐𝑜𝑠𝑐𝑜𝑠(𝑠𝑖𝑛𝑠𝑖𝑛𝑥) 𝑑𝑠𝑖𝑛𝑠𝑖𝑛𝑥 2
= = . = {− 𝑠𝑖𝑛 𝑠𝑖𝑛(𝑠𝑖𝑛 𝑠𝑖𝑛𝑥)}. (
𝑑𝑥 𝑑𝑥 𝑑𝑠𝑖𝑛𝑠𝑖𝑛𝑥 𝑑𝑥
𝑐𝑜𝑠 𝑐𝑜𝑠𝑥).
15 Here, 𝑦 +𝑠𝑖𝑛 𝑠𝑖𝑛𝑦 =𝑐𝑜𝑠 𝑐𝑜𝑠𝑥
differentiating the above with respect to 𝑥 and applying chain rule, we get
𝑑𝑦 𝑑𝑠𝑖𝑛𝑠𝑖𝑛𝑦 𝑑𝑐𝑜𝑠𝑐𝑜𝑠𝑥 1
+ = .
𝑑𝑥 𝑑𝑥 𝑑𝑥

𝑑𝑦 𝑑 𝑠𝑖𝑛 𝑠𝑖𝑛𝑦 𝑑𝑦 𝑑𝑦 𝑑𝑦
⟹ + . = − 𝑠𝑖𝑛 𝑠𝑖𝑛𝑥 ⟹ +𝑐𝑜𝑠 𝑐𝑜𝑠𝑦. = − 𝑠𝑖𝑛 𝑠𝑖𝑛𝑥
𝑑𝑥 𝑑𝑦 𝑑𝑥 𝑑𝑥 𝑑𝑥
𝑑𝑦 𝑑𝑦 −𝑠𝑖𝑛𝑠𝑖𝑛𝑥
⟹ (1 +𝑐𝑜𝑠 𝑐𝑜𝑠𝑦) = − 𝑠𝑖𝑛 𝑠𝑖𝑛𝑥 = = . 1
𝑑𝑥 𝑑𝑥 1+𝑐𝑜𝑠𝑐𝑜𝑠𝑦

16 Given that, 𝑥 = 𝑎 𝑐𝑜𝑠 𝑐𝑜𝑠𝑡, 𝑦 = 𝑎 𝑠𝑖𝑛 𝑠𝑖𝑛𝑡


𝑑𝑦 𝑑𝑥 1
Therefore, = 𝑎 𝑐𝑜𝑠 𝑐𝑜𝑠𝑡, = −𝑎 𝑠𝑖𝑛 𝑠𝑖𝑛𝑡
𝑑𝑡 𝑑𝑡
𝑑𝑦
𝑑𝑦 𝑑𝑡 𝑎𝑐𝑜𝑠𝑐𝑜𝑠𝑡
Hence, = 𝑑𝑥 = = − 𝑐𝑜𝑡 𝑐𝑜𝑡𝑡.
𝑑𝑥
𝑑𝑡
−𝑎𝑠𝑖𝑛𝑠𝑖𝑛𝑡 1

61
17 . Here y = sin-1{
√1+𝑥 − √1−𝑥
}, 3
2

Put x = cos 2θ
√1+𝑐𝑜𝑠2𝜃 − √1−𝑐𝑜𝑠 2𝜃
y = sin-1{ 2
}

√2 𝑐𝑜𝑠 𝜃 − √2 𝑠𝑖𝑛 𝜃
= sin-1{ 2
}

1 1
= sin-1[ 𝑐𝑜𝑠 𝜃 − 𝑠𝑖𝑛 𝜃]
√2 √2

𝜋 𝜋
= sin-1[𝑠𝑖𝑛 𝑐𝑜𝑠 𝜃 − 𝑐𝑜𝑠 𝑠𝑖𝑛 𝜃]
4 4

𝜋
= sin-1[𝑠𝑖𝑛 ( 4 − 𝜃)]
𝜋
= ( 4 − 𝜃)
𝜋 1
= - 2 cos-1 x
4

Differentiating w.r.t. x
𝑑𝑦 1
= 0 - 2 × (-1)/(1-x2)1/2
𝑑𝑥

= 1/2(1-x2)1/2

𝑠𝑖𝑛 𝑥 + 𝑐𝑜𝑠 𝑥 3
18 𝑓(𝑥) = 𝜋
𝑥−
4

1 1
√2( 𝑠𝑖𝑛 𝑥 − 𝑐𝑜𝑠 𝑥)
√2 √2
= 𝜋
𝑥−
4

𝜋
√2 𝑠𝑖𝑛 ( 𝑥 − 4 )
= 𝜋
𝑥 −
4

= √2
𝜋
f( 4 ) = k
𝜋
For the continuity of f(x) at x =4
𝜋
f( 4 ) = 𝑓(𝑥)

k = √2

62
19 y = ae2x – be-x 3
𝑑𝑦
= 2ae2x + be-x
𝑑𝑥

d2y/dx2 = 4 ae2x – be-x


LHS = d2y/dx2 – dy/dx – 2y
= 4 ae2x – be-x - 2ae2x- be-x -2(ae2x – be-x )
= 4 ae2x – be-x - 2ae2x - be-x -2ae2x +2 be-x
=0
= RHS Proved

20 Here, (cos y)x = ( sin x )y 3


Taking log both sides we get
log (cos y)x =log ( sin x )y
Or, x log cos y = y log sin x
Or, y log sin x = x log cos y
Differentiating both sides w.r.t x
𝑐𝑜𝑠 𝑥 𝑑𝑦 − 𝑠𝑖𝑛 𝑦 𝑑𝑦
y X 𝑠𝑖𝑛 𝑥 + log sin x 𝑑𝑥 = x X + log cos y
𝑐𝑜𝑠 𝑦 𝑑𝑥

𝑑𝑦 𝑑𝑦
Or, y cot x + log sin x 𝑑𝑥 = - x tan y 𝑑𝑥 + log cos y
𝑑𝑦
Or, 𝑑𝑥 (log sin x + x tan y ) = log cos y – y cot x
𝑑𝑦 𝑙𝑜𝑔 𝑐𝑜𝑠 𝑦 – 𝑦 𝑐𝑜𝑡 𝑥
Or, 𝑑𝑥 = 𝑙𝑜𝑔 𝑠𝑖𝑛 𝑥 + 𝑥 𝑡𝑎𝑛 𝑦

63
21 Here, sin2 y + cos xy = k 3
Differentiating both sides w.r.t x
𝑑𝑦 𝑑𝑦
2 sin y cos y 𝑑𝑥 - sin xy (x .𝑑𝑥 + y ) = 0
𝑑𝑦
Or,(2 sin y cos y – x sin xy) 𝑑𝑥 = y sin xy
𝑑𝑦 𝑦 𝑠𝑖𝑛 𝑥𝑦
Or, 𝑑𝑥 = (2 𝑠𝑖𝑛 𝑦 𝑐𝑜𝑠 𝑦 – 𝑥 𝑠𝑖𝑛 𝑥𝑦)

𝑦 𝑠𝑖𝑛 𝑥𝑦
= ( 𝑠𝑖𝑛 2𝑦 – 𝑥 𝑠𝑖𝑛 𝑥𝑦)
𝜋 𝜋
𝑑𝑦 𝜋 𝑠𝑖𝑛 2 𝑋
8 8
(at x = 2, y = 8 ) = 𝜋 𝜋
𝑑𝑥 ( 𝑠𝑖𝑛 2 𝑋 – 2 𝑠𝑖𝑛 2 𝑋 )
8 8

𝜋 𝜋
𝑠𝑖𝑛
8 4
= 𝜋 𝜋
( 𝑠𝑖𝑛 – 2 𝑠𝑖𝑛 )
4 4

𝜋 1
𝑋
8 √2
= 1 1
–2𝑋 )
√2 √2

𝜋 𝑋 √2
= − 8√2
−𝜋
= 8

𝑐𝑜𝑠 𝑥 − 𝑠𝑖𝑛 𝑥 3
22 Let y = cot-1 (𝑐𝑜𝑠 𝑥 + 𝑠𝑖𝑛 𝑥 )
𝑠𝑖𝑛 𝑥
𝑐𝑜𝑠 𝑥 (1− 𝑐𝑜𝑠 𝑥)
= cot-1[ 𝑠𝑖𝑛 𝑥 ]
𝑐𝑜𝑠 𝑥 (1 + 𝑐𝑜𝑠 𝑥)

1 − 𝑡𝑎𝑛 𝑥
= cot-1(1 + 𝑡𝑎𝑛 𝑥)
𝜋
= cot-1[𝑡𝑎𝑛 ( 4 − 𝑥)]
𝜋 𝜋
= cot -1[𝑐𝑜𝑡 { 2 − ( 4 − 𝑥)}]
𝜋
= (4 − 𝑥)

Differentiating both sides w.r.t x


𝑑𝑦
= -1
𝑑𝑥

64
𝑓(1+ℎ) − 𝑓(1) 3
23 RHD = ℎ

𝑓[1+ℎ] − [1]
= ℎ
1− 1
= ℎ

=0
𝑓(1−ℎ) − 𝑓(1)
LHD = −ℎ
𝑓[1−ℎ] − [1]
= −ℎ
0−1
= −ℎ
1
=ℎ

=∞
Since , RHD ≠ LHD
So, f(x) is not differentiable at x = 1.

24 x √1 + 𝑦 - y √1 + 𝑥 = 0 3

Or, x √1 + 𝑦 = y √1 + 𝑥

Squaring both sides

Or,( x √1 + 𝑦 )2 = ( y √1 + 𝑥 )2

Or, x2( 1+ y) = y2 (1 + x)
Or, x2 + x2y = y2 + y2x
Or, x2 + x2y - y2 - y2x = 0
Or, x2 – y2 + x2y – y2 x = 0
Or, (x + y) ( x – y) + xy ( x – y ) = 0
Or, (x – y ) (x + y + xy ) = 0
As x ≠y , (x + y + xy ) = 0
Or, x (1 + y) = - y
−𝑦
Or, x = 1 + 𝑦

Differentiating both sides w.r.t. y


𝑑𝑥
= {- 1 . (1 + y ) – (- y ).1}/(1 + y )2
𝑑𝑦

= (-1-y + y)/ (1 + y )2
𝑑𝑥
= -1/(1+y)2
𝑑𝑦

65
25 Here , y = (cos-1x)2 3
Differentiating both sides w.r.t. x
y’ = {2 cos-1x X -1}/(1-x2)1/2
y’ X (1 – x2 )1/2 = - 2 cos-1x
Again differentiating both sides w.r.t. x
y’’ X (1 – x2 )1/2 + y’X (-2x) .1/2(1-x2)1/2 = 2 / (1-x2)1/2
Or, y’’ X (1 – x2 ) – x y’ = 2
Or, (1 – x2 )y’’- x y’– 2 = 0
𝑦
26 Here,log( x2 + y2 )= 2 cot-1𝑥 3

Differentiating both sides w.r.t. x


𝑑𝑦 𝑑 𝑦
1/(x 2 +y 2 ) X (2 x + 2y 𝑑𝑥 )= -2/{1 +( y/x)2 } 𝑑𝑥 (𝑥 )

Or,(x + y y’)/(x 2 +y 2) ={- x2/(x2 + y2) } { (x y’ – y )/x2 }


Or,(x + y y’) = - x y’ + y
Or, x + y = y’(y – x )
𝑥+𝑦
Or, y’ = 𝑦 − 𝑥

𝑑𝑦 𝑥+𝑦
Or, 𝑑𝑥 = 𝑦 − 𝑥

27 The given function can be written as


f(x) ={5 − 2𝑥, 𝑖𝑓 𝑥 < 13, 𝑖𝑓 1 ≤ 𝑥 < 42𝑥 − 5, 𝑖𝑓 𝑥 ≥ 4
Hence , when x <1 , Then height in terms of x is 5 – 2x .
Ans.(i) (a) 5- 2x
(ii) (d) f(x) is not differentiable at x = 1.
𝑑
(iii)f’ (x) = 𝑑𝑥 (3)

=0
𝑑
Or, f’ (x) = 𝑑𝑥 (2 x - 5)

=2

66
𝑑 𝑑
28 (i) (sin x5 ) = cos x5𝑑𝑥 ( x5 ) = 5x4 cos x5
𝑑𝑥

𝐴𝑛𝑠. (b)
𝑑 𝑑
(𝑖𝑖) 𝑑𝑥 cos (sin x) = - sin (sin x) 𝑑𝑥 (sin x) =- cos x sin (sin x)

𝐴𝑛𝑠. (b)
𝑑 𝜋
(iii)𝑑𝑥 (sin 2x ) at x= 4 .

= 2 cos 2x
𝜋
= 2 cos 2

=2X0
=0
𝑑 𝑑
Or, (sin e2x ) = cos e2x𝑑𝑥 ( e2x )
𝑑𝑥
𝑑
= e2x cos e2x𝑑𝑥 ( 2x)

=2 e2x cos e2x


−3
29 (i) f’(x) = 2
, x <1
−3
So, f’(-2) = 2

(ii) f’(x) = -1 , 1 ≤ x ≤ 3

so, f’( 3) = -1

67
30 (i) (a)
(ii) (c)
(iii) Since f is continuous at x = 3
So, LHL = RHL = f(a)
𝑥+3
= 3+3
=6
So, k = 6
Or,
Since f is continuous at x = 3
So, LHL = RHL = f(a)
𝑘𝑥 𝑘(−𝑥)
= =-k
𝐼𝑥𝐼 𝐼𝑥𝐼

f(0) = 3
So, -k = 3
Or, k = -3

31 Here, 𝑦 = 𝑒 𝑎𝑥 ……………(1)
differentiating (1) w.r.t.𝑥 we get,
𝑑𝑦 𝑒 𝑎𝑥 −𝑎𝑦
= −𝑎 = …………(2)
𝑑𝑥 √1−𝑥 2 √1−𝑥 2

𝑑𝑦 𝑎2 𝑦 2
1
squaring (2) we get, ( )2 =
𝑑𝑥 (1−𝑥 2 )

𝑑𝑦 2
⟹ (1 − 𝑥 2 ). ( ) = 𝑎2 𝑦 2 1
𝑑𝑥
Again differentiating w.r.t.𝑥, we get,

2 ).
𝑑𝑦 𝑑 2 𝑦 𝑑𝑦 𝑑𝑦
(1 − 𝑥 2. . 2 − 2𝑥. ( )2 = 2𝑎2 𝑦
𝑑𝑥 𝑑𝑥 𝑑𝑥 𝑑𝑥 1
𝑑2𝑦 𝑑𝑦
⟹ (1 − 𝑥 2 ). 2
− 𝑥. = 𝑎2 𝑦
𝑑𝑥 𝑑𝑥
1
2
𝑑 𝑦 𝑑𝑦
⟹ (1 − 𝑥 2 ). 2
− 𝑥. − 𝑎2 𝑦 = 0.
𝑑𝑥 𝑑𝑥

68
32 Here, 𝑐𝑜𝑠 𝑐𝑜𝑠𝑦 = 𝑥 𝑐𝑜𝑠 𝑐𝑜𝑠(𝑎 + 𝑦)
𝑐𝑜𝑠𝑐𝑜𝑠𝑦
⟹ = 𝑥……………….(1)
𝑐𝑜𝑠𝑐𝑜𝑠(𝑎+𝑦)

differentiating (1) w.r.t.𝑥 we get,

− 𝑠𝑖𝑛 𝑠𝑖𝑛𝑦. 𝑑𝑦
𝑑𝑥
𝑐𝑜𝑠 𝑐𝑜𝑠(𝑎 + 𝑦) +𝑠𝑖𝑛 𝑠𝑖𝑛(𝑎 + 𝑦) 𝑑𝑦
𝑑𝑥
𝑐𝑜𝑠 𝑐𝑜𝑠𝑦
=1 1
(𝑎 + 𝑦)
𝑑𝑦 𝑑𝑦
⟹ − 𝑠𝑖𝑛 𝑠𝑖𝑛𝑦. 𝑐𝑜𝑠 𝑐𝑜𝑠(𝑎 + 𝑦) +𝑠𝑖𝑛 𝑠𝑖𝑛(𝑎 + 𝑦) 𝑐𝑜𝑠 𝑐𝑜𝑠𝑦 = (𝑎 + 𝑦)
𝑑𝑥 𝑑𝑥
1
𝑑𝑦
⟹ [− 𝑠𝑖𝑛 𝑠𝑖𝑛𝑦.𝑐𝑜𝑠 𝑐𝑜𝑠(𝑎 + 𝑦) +𝑠𝑖𝑛 𝑠𝑖𝑛(𝑎 + 𝑦) 𝑐𝑜𝑠 𝑐𝑜𝑠𝑦] = (𝑎 + 𝑦)
𝑑𝑥
𝑑𝑦
⟹ .𝑠𝑖𝑛 𝑠𝑖𝑛(𝑎 + 𝑦 − 𝑦) = (𝑎 + 𝑦)
𝑑𝑥 1
𝑑𝑦
⟹ .𝑠𝑖𝑛 𝑠𝑖𝑛𝑎 = (𝑎 + 𝑦)
𝑑𝑥
𝑑𝑦 (𝑎 + 𝑦) 1
⟹ =
𝑑𝑥 𝑠𝑖𝑛 𝑠𝑖𝑛𝑎
33 Here, ( 1- u 2 )1/2 + ( 1- v 2 )1/2 = a (u – v),
Let u = sin A and v = sin B
( 1- sin2 A )1/2 + ( 1- sin2B )1/2 = a (sin A – sin B),
Or, cos A + cos B = a (sin A – sin B),
𝐴+𝐵 𝐴−𝐵 𝐴+𝐵 𝐴−𝐵
Or, 2 cos cos = 2 a cos sin
2 2 2 2
𝐴−𝐵 𝐴−𝐵
Or, cos = a sin
2 2
𝐴−𝐵
Or, cot = a
2
𝐴−𝐵
Or, = cot-1 a
2

Or, A – B = 2 cot-1 a
sin-1u – sin-1 v = 2 cot-1 a
Differentiating w.r.t. u
𝑑𝑣
1/( 1- u 2 )1/2 - 1/ ( 1- v 2 )1/2𝑑𝑢 = 0
𝑑𝑣
Or, 𝑑𝑢 = ( 1- v 2 )1/2 / ( 1- u 2 )1/2 .

69
𝜋
34 If x = a tan 3 θ , y = a sec3 θ, then find d2y/dx2 at θ = 3 .

Soln. x = a tan 3 θ
Differentiating w.r.t. θ
𝑑𝑥
= a ×3 tan2 θ sec2 θ
𝑑𝜃

Again, y = a sec3 θ
Differentiating w.r.t. θ
𝑑𝑦
= a ×3 sec2 θ sec θ tan θ = 3 a tan θ sec3 θ
𝑑𝜃
𝑑𝑦
= 3 a tan θ sec3 θ/ a ×3 tan2 θ sec2 θ
𝑑𝑥
𝑠𝑒𝑐 𝜃
= 𝑡𝑎𝑛 𝜃

= cosec θ
Differentiating w.r.t. x
𝑑
d2y/dx2 =𝑑𝑥 (cosec θ)

𝑑𝜃
= - cosec θ cot θ 𝑑𝑥

= - cosec θ cot θ / a ×3 tan2 θ sec2 θ


= - cosec θ cot θ × cos4 θ / a ×3 × sin2 θ
= - cos5 θ/ a ×3 × sin4 θ
𝜋
d2y/dx2 at θ = 3 ,

={ - (1/2)5 }/{3a (√3 /2)4}


= {-1/32}/{27a/16}
−1
=54 𝑎

70
CHAPTER-6: APPLICATION OF DERIVATIVES
1 MARK QUESTIONS
Q.NO. QUESTIONS MARKS

1. Find the rate of change of the area of circle with respect to its radius r when r = 3

1 (a) 3π cm (b) 6π cm (c) 9π cm (d)12π cm 1

2 For the curve y = 5x -2x3, if x increases at the of 2 unit/sec, then how fast in the slope of 1
curve changes when x =3
(a) -72 (b) - 48 (c) 24 (d) 108

3 The sides of an equilateral triangle are increasing at the rate of 2cm/sec, find the rate at 1
which the area increases, when side is 10 cm
(a) 4√3 cm2/sec (b) 6√3 cm2/sec (c) 8√3 cm2/sec (d) 10√3 cm2/sec
4 The function f ( x ) = cos x , xϵ[0,2π], is strictly increasing on 1

(a) (0, π) (b) (  , 2 ) (c) (π/2,π) (d) None

5 It is given that at x= 1 the function f ( x) = x 4 − 62 x 2 + ax + 9 attains its maximum value on 1


the interval [0,2]. Find the value of a?
(a) 110 (b) -25 (c)120 (d) 256

6 The function 𝑓(𝑥) = [𝑥(𝑥 − 2)]2 is strictly increasing in the set 1


(a)(−∞, 0) ∪ (2, ∞) (b) (−∞, 1) (c) (0,1) ∪ (2, ∞) (d) (1,2)

7 The function 𝑓(𝑥) = 𝑎𝑥 + 𝑏 is strictly increasing for all real 𝑥,if 1


(a) 𝑎 > 0 (b) 𝑎 < 0 (c) 𝑎 = 0 (d) 𝑎 ≤ 0

8 If 𝑓(𝑥) = 3𝑥 4 + 4𝑥 3 − 12𝑥 2 + 12 then 𝑓(𝑥) is 1


(a)Strictly increasing in (−2,0) and strictly decreasing in (0,1)
(b) Strictly decreasing in (−2,0)
(c) Strictly decreasing in (−2,0) and strictly increasing in (0,1)
(d) Strictly decreasing in (−∞, −2) ∪ (1, ∞)

9 A stone is dropped into a quiet lake and waves move in circles at a speed of 4 cm per 1

71
second. At the instant, when the radius of the circular wave is 10 cm, how fast is the
enclosed area increasing?
(a) 10𝜋𝑐𝑚2 /𝑠 (b) 40𝜋𝑐𝑚2 /𝑠 (c) 80𝜋𝑐𝑚2 /𝑠 (d) 120𝜋𝑐𝑚2 /𝑠

1
If the rate of change of volume of a sphere is equal to the rate of change of its radius then
10 find its radius.
(a) 1/3√𝜋 (b) -1/2√𝜋 (c) 1/4√𝜋 (d) 1/2√𝜋

11 The function f(x) = x2-2x+1 is increasing function  1


(a) x  1 (b) x  1/2 (c) x≤1 (d) None

12 Sand is pouring from a pipe at the rate of 12cm3/sec. the falling sand forms a cone on the 1
ground in such a way that the height of the cone is always one- sixth of the radius of the
base. How fast is the height of the sand-cone increasing, when height is 4 cm.
(a) 1/48π cm/s (b)3/16 π cm/s (c) 3/5 π cm/s (d) 5/16 cm/s

13 If the radius of a soap bubble is increasing at the rate of ½ cm/sec. At what rate its volume 1
is increasing when the radius is 1 cm.
(a) π cm3/s (b) 2π cm3/s (c) 3π cm3/s (d)4π cm3/s

14 For the function f(x)= 2x3 - 3x2 -12x +4 , find the points of local maxima and minima. 1
(a) (-1,2) (b) (1,2) (c) (-1,-2) (d) (2,2)
15 2
The value of x for which (x-x ) is maximum is 1
3 1 1 1
(a) (b)4 (c) 3 (d)2
4

16 17.The rate of change of area of a square is 40 sq cm/s. What will be the rate of change of 1
side if the side is 5 cm.
(a) 2 cm/s (b) 4 cm/s (c) 6 cm/s (d)8 cm/s

2 MARKS QUESTIONS

Q. QUESTION MARK
NO

1 The volume of a cube is increasing at the rate of 7 cubic cm per second. How fast is the 2
surface area of the cube increasing when the length of an edge is 12 cm

72
2 Find the rate of change of the volume of a sphere with respect to its surface area when 2
the radius is 2 cm.

3 A ladder 5 m long is leaning against a wall. The bottom of the ladder is pulled along the 2
ground, away from the wall, at the rate of 2 cm/s. How fast is its height on the wall
decreasing when the foot of the ladder is 4 m away from the wall.

4 Find the intervals in which the function 𝑓(𝑥) = 5 + 36𝑥 + 3𝑥 2 – 2𝑥 3 is strictly 2


increasing or strictly decreasing.

5 Find the intervals in which the function 𝑓(𝑥) = 2𝑥 3 – 9𝑥 2 + 12𝑥 + 15 is (i) strictly 2
increasing and (ii) strictly decreasing.

6 The volume of a cube is increasing at the rate of 9 𝑐𝑚3 / 𝑠𝑒𝑐 fast is its surface area 2
increasing when the length of an edge in 10cm?

7 For what value of K, the function 𝑓(𝑥) = 𝑘(𝑥 + 𝑠𝑖𝑛𝑥) + 𝑘 is strictly increasing. 2

8 A particle moves along the curve 6𝑦 = 𝑥 3 + 2. Find points on the curve at which y co- 2
ordinate is changing 8 times as fast as x co-ordinate.

9 Find the intervals in which the function 𝑓(𝑥) = 3𝑥 4 − 4𝑥 3 − 12𝑥 2 + 5 is 2


I. strictly increasing
II. strictly decreasing

10 Find the intervals in which the function 𝑓(𝑥) = 𝑠𝑖𝑛𝑥 + 𝑐𝑜𝑠𝑥, 0 ≤ 𝑥 ≤ 𝜋 is 2


I. strictly increasing
II. strictly decreasing

11 Find the interval in which 𝑦 = 𝑥 2 𝑒 −𝑥 is strictly increasing. 2

12 Find the interval in which 𝑓(𝑥) = 𝑥 𝑥 , 𝑥 > 0 is strictly increasing. 2

13 The side of a square is increasing at the rate of 0.2 cm/sec. Find the rate of increase of 2
perimeter of the square.

14 𝑥2 2
The total cost of manufacturing of x pocket radios is Rs. ( 4 + 35𝑥 + 25) and the rate at
100−𝑥
which they may be sold to a distributor is Rs. each. What should be the daily output
2
to attain maximum profit.

15 In the printed page of a book, the combined width of the margin at the top and bottom is 2
3 inches and that on sides is 2 inches. Find the dimensions of the page, so that the area of
the printed matter may be maximum, if the total area of the page is 150 sq. inches.

16 For the curve 𝑦 = 5𝑥 − 2𝑥 3 , if x increases at the rate of 2 units/sec, then how fast is the
lope of the curve changing when x = 3?

17 A ladder 5 m long is leaning against a wall. The bottom of the ladder is pulled away along
the ground, away from the wall, at the rate of 2 m/sec. How fast is its height on the wall
decreasing when the foot of the ladder is 4 m away from the wall?

73
18 For what value of k the function 𝑓(𝑥) = 𝑘(𝑥 + 𝑠𝑖𝑛𝑥) + 𝑘 is increasing?

19 Find the least value of a so that the function 𝑓(𝑥) = 𝑥 2 + 𝑎𝑥 + 1 is strictly increasing on 2
(1,2).

20 Find the interval in which the function 𝑓(𝑥) = 𝑥 𝑥 , 𝑥 > 0 is strictly increasing 2
1
21 Let I be an interval disjoint from (-1, 1), prove that the function 𝑓(𝑥) = 𝑥 + is strictly 2
𝑥
increasing on I.

22 Find the maximum and minimum value of the function f given by 𝑓(𝑥) = 𝑠𝑖𝑛𝑥 + 𝑐𝑜𝑠𝑥 2

23 At what points in the interval [0, 2𝜋] does the function sin2x attain its maximum value? 2
1
24 Show that the local maximum value of 𝑥 + 𝑥 is less than the local minimum value. 2

25 At what point, the slope of the curve 𝑦 = −𝑥 3 + 3𝑥 2 + 9𝑥 − 27 𝑖𝑠 𝑚𝑎𝑥𝑖𝑚𝑢𝑚? Also find 2


the maximum slope.

5 MARKS QUESTIONS

Q.NO. QUESTIONS MARKS


1 The length of the sides of an isosceles triangle are 9 + x2, 9 + x2 and 18 – 2x2 units. Calculate 5
the area of the triangle in terms of x and find the value of x which makes the area
maximum.
2 Show that the height of the cylinder of maximum volume that can be inscribed in a sphere 5
2𝑅
of radius 𝑅 is . Also find the maximum volume.
√3

3 A tank with rectangular sides open at the top is to be constructed so that its depth is 2 m 5
and volume is 8 𝑚3 . If building of tank costs Rs. 70/𝑚2 for the base and Rs. 45/𝑚2 for sides,
what is cost of the least expensive tank?
4 Show that the semi vertical angle of the cone of the maximum volume and of given slant 5
height is 𝑡𝑎𝑛−1 √2.
5 ind the intervals in which the function 𝑓(𝑥) = 2𝑥 3 − 15𝑥 2 + 36𝑥 + 1 is strictly increasing 5
or decreasing. Also find the points on which the tangents are parallel to x-xis.
6 If sum of the lengths of the hypotenuse and a side of a right angled triangle is given, show 5
𝜋
that the area of the triangle is maximum when the angle between them is 3.

7 A metal box with a square base and vertical sides is to contain 1024 cubic cm. The material 5
for the top and bottom costs Rs. 5 per sq cm and material for the sides costs Rs. 2.50 per sq
cm. Find the least cost of the box.

74
8 A window is in the form of a rectangle surmounted by a semicircle. If the perimeter of the 5
window is 30m, find the dimensions of the window so that the maximum possible light is
admitted.
9 A enemy helicopter is moving among the curve 𝑦 = 𝑥 2 + 7. A soldier placed at (3, 7) wants 5
to shoot down the helicopter when it nearest to him. Find the nearest distance. Also find
the coordinates of the helicopter when it is nearest to the soldier.

CASE BASED QUESTIONS

Q.NO. QUESTIONS MARKS


1 𝑃(𝑥) = −5𝑥 2 + 125𝑥 + 37500 is the total profit function of a company, where x is the 4
production of the company.

Based on the above information, answer the following questions:


(i) What will be the production when the profit is maximum?
(ii) What will be the maximum profit?
(iii) When the production is 2 units what will be the profit of the company?
OR
What will be production of the company when the profit is Rs. 38250?
2 Suraj’s father wants to construct a rectangular garden using a brick wall on one side of the 4
garden and wire fencing for the other three sides. He has 200m of fencing wire.

75
Based on the above information, answer the following questions:
(i) Let x metres denote the length of the side of the garden perpendicular to the
brick wall and y metres denote the length of the side parallel to the brick wall.
Determine the relation representing the total length of fencing wire and also
write A(x), area of the garden.
Determine the maximum value of A(x).
3 The relation between the height of the plant (y in cm ) with respect o exposure to sunlight is 4
1
governed by the following equation 𝑦 = 4𝑥 − 2 𝑥 2 , where x is the number of days exposed
to sunlight.

Based on the above information, answer the following questions:


(i) What is the rate of growth of the plant with respect to sunlight?
(ii) What is number of days it will take for the plant to grow to the maximum height?
(iii) What is the maximum height to which the plant will grow?
Or
If the height of plant is 7/2 cm, then for how many days it has to exposed to the sunlight?

76
4 Megha wants to prepare a handmade gift box for her friend's birthday at home. For making 4
lower part of box, she takes a square piece of cardboard of side 20 cm.

Based on the above information, answer the following questions.


(i) If x cm be the length of each side of the square cardboard which is to be cut off from
corners of the square piece of side 20 cm, then what will be the possible of value of x?
(ii) How Volume of the open box formed by folding up the cutting corner can be expressed?
(iii) Megha is interested in maximising the volume of the box. So, what should be the side of
the square to be cut off so that the volume of the box is maximum?
Or
What will be the maximum value of the volume?

ANSWERS/SOLUTIONS

Q.NO. QUESTIONS MARKS


1 𝑑𝐴 1
(b) A =πr2 ,𝑑𝑟 = 2πr = 6π cm

2 (a) 1
𝑑𝑦 𝑑𝑚 𝑑𝑥
𝑠𝑙𝑜𝑝𝑒𝑚𝑚 = = 5 − 6𝑥 2 ⇒ = −12𝑥 = −12.3.2 = −72
𝑑𝑥 𝑑𝑡 𝑑𝑡

3 .(d) 1
Area of the equilateral triangle
√3 2 𝑑𝐴 √3 𝑑𝑙 √3
𝐴= 𝑙 ⇒ = 2𝑙 = 2.10.2 = 10√3
4 𝑑𝑡 4 𝑑𝑡 4

4 . (b) 1
F1(x) = -sinx> 0 ⇒sinx< 0 ⇒ θ € (π ,2π)

5 . (c) 1

f ' ( x) = 4 x 3 − 124 x + a

77
f ' (1) = 0
a = 120

6 (c) 𝑓(𝑥) = [𝑥(𝑥 − 2)]2 1


f’(x) =2𝑥(𝑥 − 2)[x +x-2]
f’(x) =0 ⇒0,2,1
f’(x) >0 when xϵ (0,1) ∪ (2, ∞)

7 . (a) 1
f’(x)= a ,f’(x)>0 ⇒ a>0

8 .(a) 1
If 𝑓′(𝑥) = 12𝑥 3 + 12𝑥 2 − 24𝑥 =12x(x+2)(x-1)
F’(x)>0 in (-2,0)U(1,∞) and f’(x)<0 in (-∞, −2)U(0,1)

9 . (c) 1
2
𝐴 = 𝜋𝑟
𝑑𝐴 𝑑𝑟 𝑑𝑟
= 2𝜋𝑟 𝑑𝑡 Given = 4𝑐𝑚/𝑠, Therefore when r=10cm,
𝑑𝑡 𝑑𝑡

(d) 1
𝑑𝑣 𝑑𝑟
=
10 𝑑𝑡 𝑑𝑡

𝑑𝑟 𝑑𝑟
4𝜋𝑟 2 𝑑𝑡 = 𝑑𝑡
r = 1/2√𝜋

11 (a) 1
f(x) = x2-2x+1  x  1
diff. w.r.t x

𝑓 (𝑥) = 2x-2
= 2(x-1)
𝑓 (𝑥) = + ve> 0  x  1

12 (a) 1
Let h be the height, V, volume and r be the radius of the base of cone at time t, Given

78
13 1 dV 1
h= r  r = 6h.........(i ), = 12cm 3 / sec
6 dt
1 1
V = r 2 h =  (6h) 2 h = 12h 3
3 3
dV dh dh
= 12 .3h 2 .  12 = 36h 2 .
dt dt dt
When h=4 cm ,dh/dt=1/48π cm/s

14 4 𝑑𝑣 𝑑𝑟 𝑑𝑟 1 1
b) V=3 𝜋𝑟 3 , 𝑑𝑡 = 4𝜋𝑟 2 𝑑𝑡 , given 𝑑𝑡 = 2 cm/s
𝑑𝑣 𝑑𝑟 1
When r=1 cm , 𝑑𝑡 = 4𝜋𝑟 2 𝑑𝑡 = 4𝜋 × 1 × 2 = 2 𝜋 cm3/s

15 a) f(x)= 2x3 - 3x2 -12x +4, f’(x) = 6x2 -6x- 12 = 6(x2 –x -2) 1
f’(x)=0 ⇒ x=2,-1.

16 1
(d) f(x) = x-x2, f’(x)= 1-2x, f’(x)= 0 ⇒x=1/2, f’’(x)= -2<0

f is maximum at x=1/2 and max value=1/2

2 MARKS QUESTIONS

79
1 Let, the edge of the cube be a, the volume be 𝑉 and surface area be 𝑆. 2
𝑑𝑉
Now, 𝑑𝑡 = 7 cm/sec
𝑑 3
⟹ (𝑎 ) = 7
𝑑𝑡
𝑑𝑎
⟹ 3𝑎2 = 7
𝑑𝑡
𝑑𝑎 7
⟹ = 2
𝑑𝑡 3𝑎
𝑁𝑜𝑤, 𝑆 = 6𝑎2
𝑑𝑆 𝑑𝑎 7 28 𝑑𝑆 28
∴ = 12 𝑎 = 12 𝑎. ( 2 ) = ⟹ = [𝑤ℎ𝑒𝑟𝑒 𝑎 = 12 𝑐𝑚]
𝑑𝑡 𝑑𝑡 3𝑎 𝑎 𝑑𝑡 12
𝑑𝑆 7
⟹ = 𝑠𝑞. 𝑐𝑚/𝑠𝑒𝑐.
𝑑𝑡 3
2 4 3 2
𝑉= 𝜋𝑟
3

𝑑𝑉
⟹ = 4𝜋𝑟 2
𝑑𝑟
𝑁𝑜𝑤, 𝑆 = 4𝜋𝑟 2
𝑑𝑆
⟹ = 8𝜋𝑟
𝑑𝑟
𝑑𝑉
𝑑𝑉 𝑑𝑟 4𝜋𝑟 2 𝑟 𝑑𝑉 2
∴ = 𝑑𝑆 = = ⟹ = [𝑤ℎ𝑒𝑟𝑒 𝑟 = 12 𝑐𝑚]
𝑑𝑆 8𝜋𝑟 2 𝑑𝑆 2
𝑑𝑟

𝑑𝑉
⟹ = 1 𝑐𝑢. 𝑐𝑚/ 𝑠𝑞. 𝑐𝑚.
𝑑𝑆
3 𝑑𝑥 2
Given 𝑑𝑡 = 2 𝑐𝑚/𝑠𝑒𝑐

𝑥 2 + 𝑦 2 = 25
When 𝑥 = 4, 𝑡ℎ𝑒𝑛 𝑦 = 3
𝑥 2 + 𝑦 2 = 25
𝑑𝑥 𝑑𝑦
⟹ 2𝑥 + 2𝑦 =0
𝑑𝑡 𝑑𝑡

𝑑𝑦
⟹ 2𝑥. 2 + 2𝑦 =0
𝑑𝑡
𝑑𝑦 2𝑥 8
⟹ =− = − (𝑖𝑓 𝑥 = 4, 𝑡ℎ𝑒𝑛 𝑦 = 3)
𝑑𝑡 𝑦 3

8
Rate of decrease of height on the wall = 3 cm/sec.
80
4 𝑓(𝑥) = 5 + 36𝑥 + 3𝑥 2 – 2𝑥 3 2
⟹ 𝑓 ′ (𝑥) = 36 + 6𝑥 − 6𝑥 2 = 6(6 + 𝑥 − 𝑥 2 ) = −6(𝑥 − 3) (𝑥 + 2)
𝑁𝑜𝑤, 𝑓 ′ (𝑥) = 0 ⇒ 𝑥 = −2, 3
Plotting these points on number line

∴ 𝑓(𝑥) 𝑖𝑠 𝑠𝑡𝑟𝑖𝑐𝑡𝑙𝑦 𝑖𝑛𝑐𝑟𝑒𝑎𝑠𝑖𝑛𝑔 𝑜𝑛 (−2, 3) 𝑎𝑛𝑑 𝑠𝑡𝑟𝑖𝑐𝑡𝑙𝑦 𝑑𝑒𝑐𝑟𝑒𝑎𝑠𝑖𝑛𝑔 𝑜𝑛 (−∞, −2)


∪ (3, ∞).

5 We have, 𝑓(𝑥) = 2𝑥 3 – 9𝑥 2 + 12𝑥 + 15 2

⟹ 𝑓 ′ (𝑥) = 6𝑥 2 – 18𝑥 + 12 = 6(𝑥 2 – 3𝑥 + 4) = 6(𝑥 – 1)(𝑥 – 2).


𝐹𝑜𝑟 𝑖𝑛𝑐𝑟𝑒𝑎𝑠𝑖𝑛𝑔 𝑎𝑛𝑑 𝑑𝑒𝑐𝑟𝑒𝑎𝑠𝑖𝑛𝑔 𝑓𝑢𝑛𝑐𝑡𝑖𝑜𝑛 𝑓 ′ (𝑥) = 0
⟹ 6(𝑥 – 1)(𝑥 – 2) = 0 ⟹ 𝑥
= 1, 2𝑇ℎ𝑒𝑟𝑒𝑓𝑜𝑟𝑒, 𝑑𝑖𝑠𝑗𝑜𝑖𝑛𝑡 𝑖𝑛𝑡𝑒𝑟𝑣𝑎𝑙𝑠 𝑜𝑛 𝑟𝑒𝑎𝑙 𝑛𝑢𝑚𝑏𝑒𝑟 𝑙𝑖𝑛𝑒 𝑎𝑟𝑒 (– ∞, 1), (1, 2), (2, ∞)

Intervals Test Value Nature of 𝑓’(𝑥) 𝑓(𝑥)


𝑓’(𝑥)
= 6(𝑥 – 2)(𝑥 – 1)
( – ∞, 1) 𝑥 = 0 ( + ) (– ) (– ) Strictly
= (+) > 0 increasing
(1, 2) 𝑥 = 1.5 ( + )( – )( – ) Strictly
= (– ) < 0 decreasing
(2, ∞) 𝑥 = 3 ( + )( + )( + ) Strictly
= (+) > 0 increasing
∴ 𝑓(𝑥) is strictly increasing on (– ∞, 1)𝑈(2, ∞) and strictly decreasing on (1, 2).

81
6 𝑑𝑣 2
Let 𝑉 be the Volume of the cube, then 𝑑𝑡 = 9 cm3/sec

Surface area 𝑆 = 6𝑥 2 , where 𝑥 is the side of the cube.


𝑉 = 𝑥3
𝑑𝑉 𝑑𝑥
⇒ 𝑑𝑡
= 3𝑥2 𝑑𝑡
𝑑𝑥 1 𝑑𝑉
⇒ 𝑑𝑡
= 3𝑥2 𝑑𝑡

𝑑𝑥 3
⇒[ ] = 𝑚/𝑠
𝑑𝑡 𝑥=10 100

𝑆 = 6𝑥 2
𝑑𝑠 𝑑𝑥
⇒ = 12𝑥.
𝑑𝑡 𝑑𝑡
𝑑𝑆 3
∴[ ] = 120 × = 3.6 𝑚2 /𝑠
𝑑𝑡 𝑥=10 100
7 𝑓(𝑥) = 𝑘(𝑥 + 𝑠𝑖𝑛𝑥) + 𝑘 2
𝑓 ′ (𝑥) = 𝑘(1 + 𝑐𝑜𝑠𝑥)
Now,
0 ≤𝑐𝑜𝑠 𝑐𝑜𝑠𝑥 ≤ 1
⇒ 1 ≤ (1 + 𝑐𝑜𝑠𝑥) ≤ 2

∴ 𝑓 ′ (𝑥) = 𝑘(1 + 𝑐𝑜𝑠𝑥) ≥ 0 ∀ 𝑘 ≥ 0.

8 6𝑦 = 𝑥 3 + 2 2
𝐷𝑖𝑓𝑓𝑒𝑟𝑒𝑛𝑡𝑖𝑎𝑡𝑖𝑛𝑔 𝑏𝑜𝑡ℎ 𝑠𝑖𝑑𝑒𝑠 𝑤. 𝑟. 𝑡 𝑥
𝑑𝑦
6 = 3𝑥 2
𝑑𝑥
𝑑𝑦 3𝑥2
⇒ = … … … … … (𝑖)
𝑑𝑥 6
𝑑𝑦
ATQ,𝑑𝑥 = 8 … … … . . (𝑖𝑖)

From (i) and (ii),

3𝑥 2
=8
6
⇒ 3𝑥 2 = 48𝑥 2
⇒ 𝑥 = ±4
31
𝑃𝑢𝑡𝑡𝑖𝑛𝑔 𝑥 = ±4 𝑖𝑛 𝑔𝑖𝑣𝑒𝑛 𝑒𝑞𝑢𝑎𝑡𝑖𝑜𝑛 𝑜𝑓 𝑐𝑢𝑟𝑣𝑒 𝑤𝑒 ℎ𝑎𝑣𝑒 𝑦 = 11, −
3
31
∴ 𝑅𝑒𝑞𝑢𝑖𝑟𝑒𝑑 𝑝𝑜𝑖𝑛𝑡𝑠 𝑎𝑟𝑒 (4,11) 𝑎𝑛𝑑 (−4, − )
3

82
9 𝑓(𝑥) = 3𝑥 4 − 4𝑥 3 − 12𝑥 2 + 5 2
𝐷𝑖𝑓𝑓𝑒𝑟𝑒𝑛𝑡𝑖𝑎𝑡𝑖𝑛𝑔 𝑏𝑜𝑡ℎ 𝑠𝑖𝑑𝑒𝑠 𝑤. 𝑟. 𝑡 𝑥
𝑓 ′ (𝑥) = 12𝑥 3 − 12𝑥 2 − 24𝑥 = 12𝑥(𝑥 2 − 𝑥 − 2) = 12𝑥(𝑥 + 1)(𝑥 − 2)

𝑓 ′ (𝑥) < 0 𝑖𝑓 𝑥 ∈ (−∞, −1) ∪ (0,2)


∴ 𝑓(𝑥) 𝑖𝑠 𝑠𝑡𝑟𝑖𝑐𝑡𝑙𝑦 𝑑𝑒𝑐𝑟𝑒𝑎𝑠𝑖𝑛𝑔 𝑜𝑛 (−∞, −1) ∪ (0,2)
𝑆𝑖𝑚𝑖𝑙𝑎𝑟𝑙𝑦, 𝑓 ′ (𝑥) > 0 𝑖𝑓 𝑥 ∈ (−1,0) ∪ (2, ∞)
∴ 𝑓(𝑥) 𝑖𝑠 𝑠𝑡𝑟𝑖𝑐𝑡𝑙𝑦 𝑖𝑛𝑐𝑟𝑒𝑎𝑠𝑖𝑛𝑔 𝑜𝑛 (−1,0) ∪ (2, ∞)
10 𝑓(𝑥) = 𝑠𝑖𝑛𝑥 + 𝑐𝑜𝑠𝑥 2
𝑓 ′ (𝑥) = 𝑐𝑜𝑠𝑥 − 𝑠𝑖𝑛𝑥

⇒ 𝑓 (𝑥) = 0 ⇒ 𝑐𝑜𝑠𝑥 − 𝑠𝑖𝑛𝑥 = 0

⇒ 𝑐𝑜𝑠𝑥 = 𝑠𝑖𝑛𝑥
𝜋 5𝜋
⇒𝑥= ,
4 4
Intervals Sign of 𝑓 ′ (𝑥) Nature

𝜋 + ve Strictly increasing
[0, )
4
𝜋 5𝜋 - ve Strictly
( , )
4 4
decreasing

5𝜋 + ve Strictly increasing
( , 2𝜋]
4
11 𝑦 = 𝑥2𝑒 𝑥 2
𝑑𝑦
⇒ = 𝑥2 𝑒𝑥 + 2𝑥𝑒𝑥 = 𝑒𝑥 𝑥(𝑥 + 2)
𝑑𝑥
𝑑𝑦
∴ > 0 𝑜𝑛 (−∞, −2) ∪ (0, ∞)
𝑑𝑥
𝐻𝑒𝑛𝑐𝑒, 𝑦 𝑖𝑠 𝑠𝑡𝑟𝑖𝑐𝑡𝑙𝑦 𝑖𝑛𝑐𝑟𝑒𝑠𝑖𝑛𝑔 𝑜𝑛 (−∞, −2) ∪ (0, ∞).
12 𝑓(𝑥) = 𝑥 𝑥 , 𝑥 > 0 2
𝑑𝑓
⇒ = 𝑥𝑥 (1 + 𝑙𝑛𝑥)
𝑑𝑥
𝑑𝑓 1
∴ > 0 𝑖𝑓 𝑥 >
𝑑𝑥 𝑒
1
𝐻𝑒𝑛𝑐𝑒, 𝑓(𝑥) 𝑖𝑠 𝑠𝑡𝑟𝑖𝑐𝑡𝑙𝑦 𝑖𝑛𝑐𝑟𝑒𝑠𝑖𝑛𝑔 𝑖𝑓 𝑥 >
𝑒

83
13 Let, the side of the square be 𝑎, the perimeter be 𝑃. 2
𝑑𝑎
= 0.2 cm/sec
𝑑𝑡

𝑁𝑜𝑤, 𝑃 = 4𝑎
𝑑𝑃 𝑑𝑎
∴ =4 = 4 × 0.2 = 0.8 𝑐𝑚/𝑠𝑒𝑐
𝑑𝑡 𝑑𝑡
14 Let, 𝑃(𝑥) be the profit function. 2
𝑃(𝑥) = 𝑆. 𝑃 − 𝐶. 𝑃
100 − 𝑥 𝑥2
⇒ 𝑃(𝑥) = 𝑥 × − ( + 35𝑥 + 25)
2 4
100𝑥 − 𝑥2 𝑥2
⇒ 𝑃(𝑥) = − ( + 35𝑥 + 25)
2 4
3
⇒ 𝑃(𝑥) = − 𝑥2 + 15𝑥 − 25
4
6
∴ 𝑃′ (𝑥) = − 𝑥 + 15
4
6 3
∴ 𝑃’’(𝑥) = − = −
4 2
For maxima or minima,
𝑃’(𝑥) = 0 ⇒ 𝑥 = 10
3
𝑃’’(10) = − <0
2
Hence, profit is maximum when daily output is 10 radios

84
15 Let, 𝑥 𝑖𝑛𝑐ℎ𝑒𝑠 𝑎𝑛𝑑 𝑦 𝑖𝑛𝑐ℎ𝑒𝑠 be the dimension of pages. 2
∴ The dimension of printed pages would be 𝑥 − 2, 𝑦 −
As per the question
150
𝑥𝑦 = 150 , 𝑠𝑜 𝑦 =
𝑥
Let, 𝐴 be the printed area, then
150 300
𝐴 = (𝑥 − 2)(𝑦 − 3) = (𝑥 − 2) ( − 3) = 150 − 3𝑥 − +6
𝑥 𝑥
300
⇒ 𝐴’ = −3
𝑥2
600
⇒ 𝐴’’ = −
𝑥3
For maxima minima,
𝐴’ = 0
300
⇒ −3=0
𝑥2
⇒ 𝑥 = ±10
600 3
𝐴”(10) = − =− <0
1000 5
600 3
𝐴”(−10) = = >0
1000 5
∴ Area is maximum when 𝑥 = 10 𝑎𝑛𝑑 𝑦 = 15
Hence, required dimension of page are 10 × 15 inches

16 𝑑𝑦 𝑑 𝑑𝑦 𝑑𝑥 2
𝑦 = 5𝑥 − 2𝑥 3 ⇒ = 5 − 6𝑥 2 ⟹ ( ) = 0 − 6.2𝑥.
𝑑𝑥 𝑑𝑡 𝑑𝑥 𝑑𝑡
𝑑 𝑑𝑦 𝑑𝑥
⟹ ( ) = −12𝑥. 2 = −24𝑥 (𝑠𝑖𝑛𝑐𝑒 = 2)
𝑑𝑡 𝑑𝑥 𝑑𝑡
𝑑 𝑑𝑦
When x = 3, 𝑑𝑡 (𝑑𝑥 ) = −24 × 3 = 72 𝑢𝑛𝑖𝑡𝑠/𝑠𝑒𝑐

85
Let foot of the ladder be at a distance x m from the wall and y m be the ht of the ladder at 2
any time t, then
17
𝑑𝑥 𝑑𝑦 𝑑𝑦 𝑥 𝑑𝑥
𝑥 2 + 𝑦 2 = 25 ⟹ 2𝑥 + 2𝑦 =0⟹ =−
𝑑𝑡 𝑑𝑡 𝑑𝑡 𝑦 𝑑𝑡
𝑑𝑦 𝑥
⟹ = − . 2, 𝑤ℎ𝑒𝑛 𝑥 = 4 ⟹ 𝑦 2 = 25 − 16 = 9 ⟹ 𝑦 = 3
𝑑𝑡 𝑦
𝑑𝑦 4 8
⟹ = − .2 = −
𝑑𝑡 3 3
8
Hence the ht of the ladder on the wall is decreasing at the rate of3 𝑚/𝑠𝑒𝑐

18 𝑓(𝑥) = 𝑘(𝑥 + 𝑠𝑖𝑛𝑥) + 𝑘 ⇒ 𝑓 ′ (𝑥) = 𝑘(1 + 𝑐𝑜𝑠𝑥) 2


We know that −1 ≤ 𝑐𝑜𝑠𝑥 ≤ 1 ⇒ 0 ≤ 1 + 𝑐𝑜𝑠𝑥 ≤ 2
⇒ 1 + 𝑐𝑜𝑠𝑥 ≥ 0 ∀𝑥 ∈ 𝑅, 𝑁𝑜𝑤 𝑓 𝑖𝑠 𝑖𝑛𝑐𝑟𝑒𝑎𝑠𝑖𝑛𝑔 𝑖𝑓 𝑘(1 + 𝑐𝑜𝑠𝑥) ≥ 0

⇒𝑘>0

19 𝑓(𝑥) = 𝑥 2 + 𝑎𝑥 + 1 ⇒ 𝑓 ′ (𝑥) = 2𝑥 + 𝑎 2
1 < 𝑥 < 2 ⇒ 2 + 𝑎 < 2𝑥 + 𝑎 < 4 + 𝑎 ⇒ 2 + 𝑎 < 𝑓 ′ (𝑥) < 4 + 𝑎
For f to be strictly increasing on (1, 2), 2 + 𝑎 ≥ 0 ⇒ 𝑎 ≥ −2
Hence the lest value of a = -2

20 1 𝑑𝑦 1 2
𝐿𝑒𝑡 𝑦 = 𝑥 𝑥 , 𝑥 > 0 ⇒ 𝑙𝑜𝑔𝑦 = 𝑥𝑙𝑜𝑔𝑥 ⇒ = 𝑥. + 𝑙𝑜𝑔𝑥
𝑦 𝑑𝑥 𝑥
𝑑𝑦
⇒ = 𝑥𝑥 (1 + 𝑙𝑜𝑔𝑥), 𝑓′ (𝑥) > 0 𝑖𝑓𝑓 𝑥𝑥 (1 + 𝑙𝑜𝑔𝑥) > 0
𝑑𝑥
1
𝑏𝑢𝑡 𝑥 𝑥 > 0 ∀𝑥 > 0 ⇒ 1 + 𝑙𝑜𝑔𝑥 > 0 ⇒ 𝑙𝑜𝑔𝑥 > −1 ⇒ 𝑥 > 𝑒 −1 ⇒ 𝑥 >
𝑒
1
Hence f is strictly increasing in (𝑒 , ∞)

21 Since I is an interval disjoint form [-1, 1], so I = (−∞, 1) ∪ (1, ∞) 2

1 1 𝑥2 − 1
𝑓(𝑥) = 𝑥 + ⇒ 𝑓 ′ (𝑥) = 1 − 2 =
𝑥 𝑥 𝑥2
𝑥 2 −1
Now for 𝑥 < −1 𝑜𝑟 𝑥 > 1, 𝑥 2 > 1 ⇒ 𝑥 2 − 1 > 0 ⇒ >0
𝑥2

𝑓 ′ (𝑥) > 0 ∀𝑥 ∈ (−∞, 1) ∪ (1, ∞), hence f is strictly increasing in I

86
22 1 1 2
𝑓(𝑥) = 𝑠𝑖𝑛𝑥 + 𝑐𝑜𝑠𝑥 ⇒ √2 ( 𝑠𝑖𝑛𝑥 + 𝑐𝑜𝑠𝑥)
√2 √2
𝜋 𝜋 𝜋
⇒ √2 (𝑠𝑖𝑛𝑥𝑐𝑜𝑠 + 𝑥𝑐𝑜𝑠𝑥𝑠𝑖𝑛 ) ⇒ √2 𝑠𝑖𝑛 𝑠𝑖𝑛 (𝑥 + )
4 4 4
𝜋 𝜋
We know that −1 ≤𝑠𝑖𝑛 𝑠𝑖𝑛 (𝑥 + 4 ) ≤ 1 ⇒ −√2 ≤ √2 𝑠𝑖𝑛 𝑠𝑖𝑛 (𝑥 + 4 ) ≤ √2

⇒ −√2 ≤ 𝑓(𝑥) ≤ √2, Max value is √2 and minimum value is −√2.

23 Given 𝑓(𝑥) = 𝑠𝑖𝑛2𝑥, 𝑥 ∈ [0, 2𝜋] ⇒ 𝑓 ′ (𝑥) = 2𝑐𝑜𝑠2𝑥 2


𝑓 ′ (𝑥) = 0 ⇒ 𝑐𝑜𝑠2𝑥 = 0, 0 ≤ 𝑥 ≤ 2𝜋 ⇒ 0 ≤ 2𝑥 ≤ 4𝜋
𝜋 3𝜋 5𝜋 7𝜋 𝜋 3𝜋 5𝜋 7𝜋
⇒ 2𝑥 = , , , ⇒𝑥= , , ,
2 2 2 2 4 4 4 4
𝜋 𝜋 3𝜋 3𝜋 5𝜋 5𝜋
Now, 𝑓 (4 ) = 𝑠𝑖𝑛 2 = 1, 𝑓 ( 4 ) = 𝑠𝑖𝑛 = −1, 𝑓 ( 4 ) = 𝑠𝑖𝑛 =1
2 2

7𝜋 7𝜋
𝑓( ) = 𝑠𝑖𝑛 = −1, 𝑓(0) = 𝑠𝑖𝑛0 = 0, 𝑓(𝜋) = 𝑠𝑖𝑛2𝜋 = 0
4 2
𝜋 5𝜋
Max value of the function is 1 attains its max value at 𝑥 = &𝑥 =
4 4
1 1 2
24 Let 𝑓(𝑥) = 𝑥 + 𝑥 , 𝑥 ≠ 0 ⇒ 𝑓 ′ (𝑥) = 1 − 𝑥 2 𝑎𝑛𝑑 ⇒ 𝑓 ′′ (𝑥) = 𝑥 3 2

1
Now, 𝑓 ′ (𝑥) = 0 ⇒ 1 − 𝑥 2 = 0 ⇒ 𝑥 2 = 1 ⇒ 𝑥 = 1, −1

𝑓 ′′ (1) = 2 > 0 ⇒ 𝑓 ℎ𝑎𝑠 𝑙𝑜𝑐𝑎𝑙 𝑚𝑖𝑛𝑖𝑚𝑎 𝑎𝑡 𝑥 = 1 𝑎𝑛𝑑


Local minimum value is 𝑓(1) = 2
𝑓 ′′ (−1) = −2 < 0 ⇒ 𝑓 ℎ𝑎𝑠 𝑙𝑜𝑐𝑎𝑙 𝑚𝑎𝑥𝑖𝑚𝑎 𝑎𝑡 𝑥 = −1 𝑎𝑛𝑑
Local maximum value is 𝑎𝑛𝑑 𝑓(−1) = −2
Hence local maximum value is less than local minimum value.

87
25 𝑓(𝑥) = 2𝑥 3 − 15𝑥 2 + 36𝑥 + 1 ⇒ 𝑓 ′ (𝑥) = 6𝑥 2 − 30𝑥 + 36
𝑓 ′ (𝑥) = 6(𝑥 2 − 5𝑥 + 6) = 6(𝑥 − 2)(𝑥 − 3)
𝑓 ′ (𝑥) > 0 ⇒ (𝑥 − 2)(𝑥 − 3) > 0 ⇒ 𝑥 < 2 𝑜𝑟 𝑥 > 3
⇒ 𝑓 𝑖𝑠 𝑠𝑡𝑟𝑖𝑐𝑡𝑙𝑦 𝑖𝑛𝑐𝑟𝑒𝑎𝑠𝑖𝑛𝑔 𝑖𝑛 (−∞, 2) ∪ (3, ∞)

𝑓 ′ (𝑥) < 0 ⇒ (𝑥 − 2)(𝑥 − 3) < 0 ⇒ 2 < 𝑥 < 3


⇒ 𝑓 𝑖𝑠 𝑠𝑡𝑟𝑖𝑐𝑡𝑙𝑦 𝑑𝑒𝑐𝑒𝑎𝑠𝑖𝑛𝑔 𝑖𝑛 (2, 3)

Further, the tangents will be parallel to x-axis iff𝑓 ′ (𝑥) = 0


⇒ 6(𝑥 − 2)(𝑥 − 3) = 0 ⇒ 𝑥 = 2, 3

When x = 2, y = 𝑓(2) = 2. 23 − 15. 22 + 36.2 + 1 = 29


When x = 3, y = 𝑓(3) = 2. 33 − 15. 32 + 36.3 + 1 = 28
Hence, the points on the given curve where the tangents are parallel to x-axis are (2, 29)
and (3, 28).

5 MARKS QUESTIONS
Q.NO. QUESTIONS MARKS
1 Given sides of an isosceles triangle are 9 + x2, 9 + x2 and 18 – 2x2 units 5

9 + 𝑥 2 + 9 + 𝑥 2 + 18 − 2𝑥 2 36
𝑆= = = 18
2 2
Let, 𝐴 be the area of the triangle, then

𝐴 = √𝑆(𝑆 − 𝑎)(𝑆 − 𝑏)(𝑆 − 𝑐)

88
= √18(18 − 9 − 𝑥 2 )(18 − 9 − 𝑥 2 )(18 − 18 + 2𝑥 2 )

= √18(9 − 𝑥 2 )(9 − 𝑥 2 )(2𝑥 2 ) = 6𝑥(9 − 𝑥 2 )


𝐴(𝑥) = 54𝑥 − 6𝑥 3
𝑑𝐴
⇒ = 54 − 18𝑥2
𝑑𝑥
𝑑2 𝐴
⇒ = −36 𝑥
𝑑𝑥2
Let, us use second derivative test to obtain the maximum value.
For critical points,
𝑑𝐴
=0
𝑑𝑥
⇒ 54 − 18𝑥2 = 0

⇒ 𝑥 = ±√3

Now,

𝑑2𝐴
[ 2] = −36 √3 < 0
𝑑𝑥 𝑥=√3

𝑑2𝐴
[ ] = 36 √3 > 0
𝑑𝑥 2 𝑥=−√3

Hence, for 𝑥 = √3 , area 𝐴 is maximum.


2 Let 𝑟 be the radius and ℎ be height of the cylinder and 𝑅 be the radius of the sphere. 5

2
ℎ2 2
𝑅 = 𝑟 +
4

2
ℎ2 2 2
ℎ3
𝑉 = 𝜋𝑟 ℎ = 𝜋 (𝑅 − ) ℎ = 𝜋 (𝑅 ℎ − )
4 4

𝑑𝑉 3ℎ2
⇒ = 𝜋 (𝑅2 − )
𝑑ℎ 4

𝑑2 𝑉 3𝜋ℎ
⇒ 2
=−
𝑑ℎ 2
Now,
𝑑𝑉
⇒ =0
𝑑ℎ
2𝑅
⇒ℎ=
√3
𝑑2𝑉 3𝜋ℎ
[ 2] =− <0
𝑑ℎ ℎ=2𝑅 2
√3

89
2𝑅
So, the volume is maximum if ℎ =
√3

2
ℎ2 22
4𝑅 2 8𝑅 2
𝑟 = 𝑅 − =𝑅 − =
4 3×4 12
4𝜋𝑅 3
∴Maximum volume = 𝜋𝑟 2 ℎ = 𝑐𝑢𝑏𝑖𝑐 𝑢𝑛𝑖𝑡
3√3

3 Let, x and y be the length and breadth of the tank. 5

𝑉 = 8 𝑚3 = 𝑥 × 𝑦 × 2
⇒ 𝑥𝑦 = 4
4
𝐶(𝑥) = 280 + 180 (𝑥 + )
𝑥
𝑑𝐶 4
⇒ = 180 (1 − 2 )
𝑑𝑥 𝑥

𝑑2 𝐶 8
⇒ 2
= 180 × ( 3 )
𝑑𝑥 𝑥
Now,
𝑑𝐶
⇒ =0
𝑑𝑥
⇒𝑥=2

𝑑2𝐶 8
[ 2
] = 180 × = 180 > 0
𝑑𝑥 𝑥=2 8

So, the cost is minimum if 𝑥 = 2


Minimum cost of building is Rs. 1000.

Let, x and y be the length and breadth of the tank. 5


𝑉 = 8 𝑚3 = 𝑥 × 𝑦 × 2
⇒ 𝑥𝑦 = 4
4
𝐶(𝑥) = 280 + 180 (𝑥 + )
𝑥
4 𝑑𝐶 4
⇒ = 180 (1 − 2 )
𝑑𝑥 𝑥

𝑑2 𝐶 8
⇒ = 180 × ( )
𝑑𝑥2 𝑥3

90
Now,
𝑑𝐶
⇒ =0
𝑑𝑥
⇒𝑥=2

𝑑2𝐶 8
[ 2
] = 180 × = 180 > 0
𝑑𝑥 𝑥=2 8

So, the cost is minimum if 𝑥 = 2


Minimum cost of building is Rs. 1000.
5 Let ABC be a triangle right angled B. Let AC=l and AB=x, then 5

𝐵𝐶 2 = 𝑙 2 − 𝑥 2 ⇒ 𝐵𝐶 = √𝑙 2 − 𝑥 2 , 𝐺𝑖𝑣𝑒𝑛 𝑙 + 𝑥 = 𝑘(𝑓𝑖𝑥𝑒𝑑) ⇒ 𝑙 = 𝑘 − 𝑥
1 1
Let A be area of ∆𝐴𝐵𝐶, 𝑡ℎ𝑒𝑛 𝐴 = 𝐴𝐵 × 𝐵𝐶 ⇒ 𝐴 = 2 𝑥√𝑙 2 − 𝑥 2
2

1 1 1
⇒𝐴= 𝑥√(𝑘 − 𝑥)2 − 𝑥2 ⇒ 𝐴2 = 𝑥2 (𝑘2 − 2𝑘𝑥) = (𝑘2 𝑥2 − 2𝑘𝑥3 )
2 4 4
1 1
Let 𝐴2 = 𝑓(𝑥) = 4 (𝑘 2 𝑥 2 − 2𝑘𝑥 3 ) ⇒ 𝑓 ′ (𝑥) = 4 (2𝑘 2 𝑥 − 6𝑘𝑥 2 )and

1
𝑓 ′′ (𝑥) = (2𝑘 2 − 12𝑘𝑥)
4
𝑘
Now, 𝑓 ′ (𝑥) = 0 ⇒ 2𝑘 2 𝑥 − 6𝑘𝑥 2 = 0 ⇒ 𝑥 = 3 , 𝑎𝑠 𝑥 > 0

𝑘 1 1 𝑘
𝑓 ′′ ( ) = (2𝑘 2 − 4𝑘 2 ) = − 𝑘 2 < 0 ⇒ 𝑓 𝑖𝑠 𝑚𝑎𝑥𝑖𝑚𝑢𝑚 𝑤ℎ𝑒𝑛 𝑥 =
3 4 2 3
𝑘
𝑘 𝑘 2𝑘 𝑥 3 1
𝑤ℎ𝑒𝑛 𝑥 = , 𝑙 = 𝑘 − = ⇒ 𝐹𝑟𝑜𝑚 ∆𝐴𝐵𝐶, 𝑐𝑜𝑠𝐴 = = 2𝑘 =
3 3 3 𝑙 2
3
𝜋
⇒𝐴=
3
6 Let x cm be the side of a square box and h cm be its ht, 5

91
1024
then volume of box =𝑥 2 ℎ = 1024 ⇒ ℎ = 𝑥2

Let C be the cost of the box, then


5
C=5(area of top+area of bottom)+2 × 𝑎𝑟𝑒𝑎 𝑜𝑓 𝑠𝑖𝑑𝑒𝑠

5 1024 1024
𝐶 = 5(𝑥 2 + 𝑥 2 ) + × 4𝑥ℎ = 10𝑥 2 + 10𝑥 × 2 = 10(𝑥 2 + )
2 𝑥 𝑥
𝑑𝐶 1024 𝑑2 𝐶 1024
= 10 (2𝑥 − 2 ) 𝑎𝑛𝑑 2
= 10 (2 + 2. 3 )
𝑑𝑥 𝑥 𝑑𝑥 𝑥
𝑑𝐶 1024 512
Now, 𝑑𝑥 = 0 ⇒ 10 (2𝑥 − )=0⇒𝑥− =0
𝑥2 𝑥2

𝑑2 𝐶 1024
⇒ 𝑥3 = 512 ⇒ 𝑥 = 8 and 𝑎𝑡 𝑥 = 8 = 10 (2 + 2. ) = 60 > 0
𝑑𝑥 2 512

⇒ 𝐶 𝑖𝑠 𝑚𝑖𝑛𝑖𝑚𝑢𝑚 𝑤ℎ𝑒𝑛 𝑥 = 8.
1024
𝑀𝑖𝑛𝑖𝑚𝑢𝑚 𝑐𝑜𝑠𝑡 = 𝑅𝑠. 10 (64 + ) = 10(64 + 128) = 𝑅𝑠. 1920
8
7 Let r be the radius and x be the side of rectangle ABCD, 5

1
then perimeter of the rectangle = 2r+2x+𝜋𝑟 = 30 ⇒ 𝑥 = 2 (30 − 2𝑟 − 𝜋𝑟)

Let the area of window be A,


1 1 1 1
A= 2𝑟𝑥 + 2 𝜋𝑟 2 = 2𝑟.2 (30 − 2𝑟 − 𝜋𝑟) + 2 𝜋𝑟 2 = 30𝑟 − 2𝑟 2 − 2 𝜋𝑟 2

𝑑𝐴 𝑑2 𝐴
⇒ = 30 − 4𝑟 − 𝜋𝑟 𝑎𝑛𝑑 = −4 − 𝜋
𝑑𝑟 𝑑𝑟2

92
𝑑𝐴 30 𝑑2 𝐴
Now, 𝑑𝑟 = 0 ⇒ 𝑟 = 4+𝜋 𝑎𝑛𝑑 = −4 − 𝜋 < 0
𝑑𝑟 2

30 1 30 30
⇒A is maximum when 𝑟 = , then 𝑥 = 2 (30 − (2 + 𝜋) 4+𝜋) = 4+𝜋
4+𝜋

Hence, the maximum light is admitted when both the radius of semicircle and side BC of the
30
rectangle =4+𝜋 m

8 Let A(3, 7)be the position of our soldier and P(x, y) be the position of enemy helicopter on 5
the curve 𝑦 = 𝑥 2 + 7, then

AP = √(𝑥 − 3)2 + (𝑦 − 7)2 = √(𝑥 − 3)2 + (𝑥 2 + 7 − 7)2

= √𝑥 4 + 𝑥 2 − 6𝑥 + 9 ⇒ 𝐿𝑒𝑡 𝑓(𝑥) = 𝐴𝑃2 = 𝑥 4 + 𝑥 2 − 6𝑥 + 9



⇒ 𝑓 (𝑥) = 4𝑥3 + 2𝑥 − 6 = 2(𝑥 − 1)(2𝑥2 + 2𝑥 + 3)

𝑎𝑛𝑑 𝑓 ′′ (𝑥) = 12𝑥 2 + 2


𝑓 ′ (𝑥) = 0 ⇒ 2(𝑥 − 1)(2𝑥 2 + 2𝑥 + 3) = 0 ⇒ 𝑥 = 1
Also 𝑓 ′′ (1) = 12. 12 + 2 = 14 > 0 ⇒ 𝑓 𝑖𝑠 𝑚𝑖𝑛𝑖𝑚𝑢𝑚 𝑎𝑡 𝑥 = 1

And minimum value of f(x) = f(1) = 14 + 12 − 6.1 + 9 = 5

So, the minimum of value of AP =√5

ℎ𝑒𝑛𝑐𝑒 𝑡ℎ𝑒 𝑛𝑒𝑎𝑟𝑒𝑠𝑡 𝑑𝑖𝑠𝑡𝑎𝑛𝑐𝑒 𝑖𝑠 = √5


when x = 1, 𝑦 = 12 + 7 = 8
Therefore, the coordinates of the helicopter when it is nearest to the soldier are (1, 8).

CASE BASED QUESTIONS

Q.NO. QUESTIONS MARKS


1 𝑃(𝑥) = −5𝑥 2 + 125𝑥 + 37500 ⇒ 𝑃′ (𝑥) = −10𝑥 + 125 𝑎𝑛𝑑 𝑃′′ (𝑥) = −10 4
𝑃′ (𝑥) = 0 ⇒ 𝑥 = 12.5 and 𝑎𝑛𝑑 𝑃′′ (𝑥)(𝑎𝑡 𝑥 = 12.5) = −10 < 0
(i) x = 12.5
(ii) P(12.5)= = −5(12. .5)2 + 125(12.5) + 37500 = 𝑅𝑠. 38281.25
(iii) P(2) =−5(2)2 + 125(2) + 37500 = 𝑅𝑠. 37730
Or,

𝑃(𝑥) = 38250 ⇒ −5𝑥 2 + 125𝑥 + 37500 = 38250


⇒ 5𝑥 2 + 125𝑥 + 750 = 0 ⇒ 𝑥 = 15
2 (i) 2𝑥 + 𝑦 = 200, 𝐴(𝑥) = 200𝑥 − 2𝑥 2 4
(ii) 𝐴(𝑥) = 200𝑥 − 2𝑥 2 ⇒ 𝐴′(𝑥) = 200 − 4𝑥
𝐴′ (𝑥) = 0 ⇒ 𝑥 = 50 𝑎𝑛𝑑 𝐴′′(𝑥) = −4 < 0
93
Hence 𝑥 = 50 𝑖𝑠 𝑎 𝑝𝑜𝑖𝑛𝑡 𝑜𝑓 𝑚𝑎𝑥𝑖𝑚𝑎

𝑎𝑛𝑑 𝐴(50) = 200(50) − 2(50)2 = 10000 − 5000 = 5000𝑚2


3 1 2 𝑑𝑦 𝑑2𝑦 4
𝑦 = 4𝑥 − 𝑥 ⇒ = 4 − 𝑥 𝑎𝑛𝑑 = −1
2 𝑑𝑥 𝑑𝑥 2
𝑑𝑦 𝑑2𝑦
= 0 ⇒ 𝑥 = 4 𝑎𝑛𝑑 = −1 < 0
𝑑𝑥 𝑑𝑥 2
𝑑𝑦
(i) =4−𝑥
𝑑𝑥

(ii) 4
1
(iii) 𝑦(4) = 4 × 4 − 2 × 42 = 8𝑐𝑚

Or,
7 1 7
𝑦= 4𝑥 − 𝑥 2 = ⇒ 𝑥 2 − 8𝑥 + 7 = 0 ⇒ 𝑥 = 1
2 2 2
4 (i) 𝑖𝑓 𝑥 ∈ (0, 10) 4
(ii) 𝐿𝑒𝑡 𝑉 𝑏𝑒 𝑡ℎ𝑒 𝑣𝑜𝑙𝑢𝑚𝑒 𝑡ℎ𝑒𝑛, 𝑉 = 𝑥 × (20 − 2𝑥) × (20 − 2𝑥)
𝑑𝑉
𝑉 = 𝑥(20 − 2𝑥)2 ⇒ = −4𝑥(20 − 2𝑥) + (20 − 2𝑥)2
𝑑𝑥
= (20 − 2𝑥)(20 − 6𝑥)
𝑑𝑉 10 𝑑2𝑉
= 0 ⇒ 𝑥 = 10 𝑜𝑟 𝑎𝑛𝑑 = (20 − 2𝑥)(−6) + (20 − 6𝑥)(−2)
𝑑𝑥 3 𝑑𝑥 2
𝑑2 𝑉 10 𝑑2 𝑉
(iii) When 𝑥 = 10, > 0 𝑎𝑛𝑑 𝑤ℎ𝑒𝑛 𝑥 = , 𝑑𝑥 2 < 0
𝑑𝑥 2 3
10
Hence volume is maximum when 𝑥 = 3

Or,
10 10 2 10 40 40 16000
Maximum volume is = (20 − 2 × ) = × × = 𝑐𝑚3
3 3 3 3 3 27

CHAPTER-7: INTEGRAL
Q. QUESTIONS MARKS
NO

1 ∫ √𝑥 2 + 𝑎2 𝑑𝑥 is equal to 1

𝑥 𝑎2
a) 2 √𝑥 2 + 𝑎2 − 𝑙𝑜𝑔(𝑥 + √(𝑥 2 + 𝑎2 )
2

b) 𝑡𝑎𝑛−1 (𝑥 2 + 𝑥 + 2) + 𝐶

𝑥 𝑎2
c) 2 √𝑥 2 + 𝑎2 + 𝑙𝑜𝑔(𝑥 + √(𝑥 2 + 𝑎2 )
2

94
𝑥 𝑎2 𝑥
d) 2 √𝑥 2 + 𝑎2 + 𝑠𝑖𝑛−1 (𝑎)
2

2 𝜋 1
1−𝑥
∫ √ 𝑑𝑥 =
0 1+𝑥

𝜋
a)
2
𝜋
b) −1
2
𝜋
c) +1
2
d) none of these

∞ 𝑥
3 The value of the integral: ∫0 𝑑𝑥 , is 1
(1+𝑥)(1+𝑥 2 )

a) π/3

b) π/4

c) π/6

d) π/2

4 𝑑𝑥 1
∫ =?
√𝑥 2 − 3𝑥 + 2

a) 𝑙𝑜𝑔|𝑥 − √𝑥 2 − 3𝑥 + 2| + 𝐶

3
b) 𝑙𝑜𝑔 |(𝑥 − 2) + √𝑥 2 − 3𝑥 + 2| + 𝐶

c) 𝑙𝑜𝑔|𝑥 + √𝑥 2 − 3𝑥 + 2| + 𝐶

d) None of these

5 1 1
∫ 𝑑𝑥 𝑖𝑠 𝑒𝑞𝑢𝑎𝑙 𝑡𝑜
𝑐𝑜𝑠𝑥 + √3𝑠𝑖𝑛𝑥
1 𝑥 𝜋
a) 2 𝑙𝑜𝑔𝑡𝑎𝑛 (2 + 3 ) + 𝐶

𝜋 𝑥
b) 𝑙𝑜𝑔𝑡𝑎𝑛 (3 + 2) + 𝐶

𝑥 𝜋
c) 𝑙𝑜𝑔𝑡𝑎𝑛 (2 − 3 ) + 𝐶

d) none of these

6 ∫ √𝑥 2 − 16𝑑𝑥 =? 1

a) 𝑥√𝑥 2 − 16 − 4𝑙𝑜𝑔|𝑥 + √𝑥 2 − 16| + 𝐶


𝑥
b) 2 √𝑥 2 − 16 − 8𝑙𝑜𝑔|𝑥 + √𝑥 2 − 16| + 𝐶

95
𝑥
c) 2 √𝑥 2 − 16 + 8𝑙𝑜𝑔|𝑥 + √𝑥 2 − 16| + 𝐶

d) none of these

7 𝑠𝑖𝑛𝑥 1
∫ 𝑑𝑥 =?
(1 + 𝑠𝑖𝑛𝑥)

a) x + tan x – sec x + C
2
b) 𝑥 + 𝑥 +𝑐
𝑡𝑎𝑛 +1
2

c) x – tan x – sec x + C

d) None of these

8 1 1
∫ 3 𝑑𝑥 =?
√𝑥
2
3
a) 2 𝑥 3 + 𝐶

3
b) 2 +𝐶
2𝑥 3

3
2
c) 3 𝑥 2 + 𝐶

2
d) 2 +𝐶
3𝑥 3

9 1 1
∫ 𝑑𝑥 =?
𝑠𝑖𝑛2 𝑥𝑐𝑜𝑠 2 𝑥
a) tan x + cot x + C

b) tan x - cot x + C

c) - tan x + cot x + C

d) None of these
𝜋
10 1
∫04 𝑙𝑜𝑔(1 + 𝑡𝑎𝑛𝑥)𝑑𝑥 = ?
𝜋
a) 8 𝑙𝑜𝑔2
𝜋
b) 4 𝑙𝑜𝑔2
𝜋
c) 4

d) 0

11 𝑠𝑖𝑛𝑥 1
∫ 𝑑𝑥
3 + 4𝑐𝑜𝑠 2 𝑥

96
1 2𝑐𝑜𝑠𝑥
a) 2√3 𝑡𝑎𝑛−1 ( )+𝑐
√3

1 2𝑐𝑜𝑠𝑥
b) − 𝑡𝑎𝑛−1 ( )+𝐶
2√3 √3

c) 𝑙𝑜𝑔(3 + 4𝑐𝑜𝑠 2 𝑥) + 𝑐
1 𝑐𝑜𝑠𝑥
d) 2√3 𝑡𝑎𝑛−1 ( )+𝑐
√3

12 𝑠𝑒𝑐 2 𝑥 1
∫ 𝑑𝑥 = ?
𝑐𝑜𝑠𝑒𝑐 2 𝑥

a) tan x - x + C

b) tan x + x + C

c) - tan x + x + C

d) - tan x - x + C

13 1
1+𝑥
∫√ 𝑑𝑥 =?
1−𝑥

a) 𝑠𝑖𝑛−1 𝑥 + (1 + 𝑥 2 ) + 𝐶

b) 𝑠𝑖𝑛−1 𝑥 − √1 − 𝑥 2 + 𝐶

c) 𝑠𝑖𝑛−1 𝑥 + √1 − 𝑥 2 + 𝐶

d) None of these

14 ∫ (√𝑠𝑖𝑛𝑥)𝑐𝑜𝑠𝑥 𝑑𝑥 =? 1

2
a) 5 (𝑠𝑖𝑛𝑥)3/2 + 𝐶

3
b) 2 (𝑠𝑖𝑛𝑥)3/2 + 𝐶

2
c) 3 (𝑠𝑖𝑛𝑥)3/2 + 𝐶

2
d) 3 (𝑐𝑜𝑠𝑥)3/2 + 𝐶

15 𝑠𝑖𝑛2 𝑥 1
∫ 𝑑𝑥 =
𝑐𝑜𝑠4 𝑥

97
1 1
a) 3 𝑡𝑎𝑛2 𝑥 + 𝑐 3 𝑡𝑎𝑛2 𝑥 + 𝑐

1
b) 2 𝑡𝑎𝑛2 𝑥 + 𝐶

1
c) 3 𝑡𝑎𝑛3 𝑥 + 𝑐

d) none of these

16 𝜋/2 1
∫ 𝑠𝑖𝑛2 𝑥𝑑𝑥 =?
0

𝜋
a) 3
𝜋
b) 2

2𝜋
c) 3

𝜋
d) 4
1 1
17 Assertion (A):∫ 𝑠𝑒𝑐 4 𝑥 tan x dx = tan 2 x + tan 2 x + y 1
2 4

Reason (R): For integration by parts, we have not followed I Late Rules.

a) Both A and R are true and R is the correct explanation of A.

b) Both A and R are true but R is not the correct explanation of A.

c) A is true but R is false.

d) A is false but R is true.

18 𝑑𝑥 1 𝑥+1 1
Assertion (A):∫ = 𝑡𝑎𝑛−1 ( )+𝑐
𝑥 2 +2𝑥+3 √2 √2

𝑑𝑥 1 𝑥
Reason (R):∫ = 𝑎 𝑡𝑎𝑛−1 (𝑎) + 𝑐
𝑥 2 +𝑎2

a) Both A and R are true and R is the correct explanation of A.

b) Both A and R are true but R is not the correct explanation of A.

c) A is true but R is false.

d) A is false but R is true.

19 1 1
Assertion (A):∫ (1+𝑥 2 ) 𝑑𝑥 = tan −1 (x 2 ) + c
1
Reason (R):∫ 𝑑𝑥 = tan −1 x + c
1+𝑥 2

a) Both A and R are true and R is the correct explanation of A.

b) Both A and R are true but R is not the correct explanation of A.

98
c) A is true but R is false.

d) A is false but R is true.

20 Assertion (A):∫ 𝑥 𝑥 (1 + log x) dx = x 𝑥 + c 1


𝑑
Reason (R):𝑑𝑥 (𝑥 𝑥 ) = x 𝑥 (1 + log x)

a) Both A and R are true and R is the correct explanation of A.

b) Both A and R are true but R is not the correct explanation of A.

c) A is true but R is false.

d) A is false but R is true.

21 Read the text carefully and answer the questions: Rajni and Priyanka practice the 4
𝑓′(𝑥)
problems based on integrals. They will try to evaluate the integrals based upon∫ dx =
𝑓(𝑥)
log |f(x)| + c. Rajni first explains the steps to solve this type of integrals.
𝑓′(𝑥)
Step 1: Obtain the integral, let it be I = ∫ dx
𝑓(𝑥)

1
Step 2: Put f(x) = t and replace f’(x)dx by dt to obtain I = ∫ dt
𝑡

Step 3: Evaluate integral obtained in step II to obtain I = log |t| + c


Step 4: Replace t by f(x) step III to get I = log |f(x)| + c
2𝑥+5
1. Evaluate:∫ dx
𝑥 2 +5𝑥−7

1
2. Evaluate:∫ 𝑑𝑥
𝑥(3+𝑙𝑜𝑔𝑥)

1
3. Evaluate:∫ dx
1+𝑒 −𝑥

𝑒 𝑥 −𝑒 −𝑥
4. Evaluate:∫ dx
𝑒 𝑥 +𝑒 −𝑥

22 Read the text carefully and answer the questions: Study the case given below: 4

1
1. Find:∫(−1) 𝑥 99 𝑑𝑥 .
𝜋
2. Find∫−𝜋x cos x dx.
𝜋
3. Find∫ sin 3 x dx.
2
𝜋

2

99
𝜋
4. Find∫−𝜋 tan x sec 2 x dx.

23 Read the text carefully and answer the questions: The given integral∫ 𝑓(𝑥) dx can be 4
transformed into another form by changing the independent variable x to t by substituting
𝑑𝑥
x = g(t) Consider 𝐼 = ∫ 𝑓(𝑥)𝑑𝑥 Put 𝑥 = 𝑔(𝑡)𝑠𝑜𝑡ℎ𝑎𝑡 𝑑𝑡 = 𝑔′ (𝑡) We write dx = g’(t) dt Thus
𝐼 = ∫ 𝑓(𝑥)𝑑𝑥 = ∫ 𝑓(𝑔(𝑡))𝑔′ (𝑡)𝑑𝑡 This change of variable formula is one of the
important tools available to us in the name of integration by substitution. For example:
∫ 2𝑥𝑠𝑖𝑛(𝑥 2 + 1)𝑑𝑥 Put x 2 + 1 = t 2x dx = dt Thus, ∫ sin(t) dt = - cos(t) + C = - cos(x2 + 1)
+C

𝑠𝑖𝑛(𝑡𝑎𝑛−1 𝑥)
1. Find∫ dx is equal to:
1+𝑥 2

2. Find∫ tan x dx is equal to:


2𝑥
3. Find∫ dx.
1+𝑥 2

1
4. Find∫ dx is equal to:
𝑥+𝑥𝑙𝑜𝑔𝑥

24 Read the text carefully and answer the questions:∫ 𝑒 𝑥 [𝑓(𝑥) + 𝑓 ′ (𝑥)]𝑑𝑥 = = 4
∫ 𝑒 𝑥 𝑓(𝑥)𝑑𝑥 + ∫ 𝑒 𝑥 𝑓 ′ (𝑥)𝑑𝑥 Using integration by parts = 𝑓(𝑥)∫ 𝑒 𝑥 𝑑𝑥 − ∫ 𝑓 ′ (𝑥)𝑒 𝑥 𝑑𝑥 +
∫ 𝑓 ′ (𝑥)𝑒 𝑥 dx = f(x) e 𝑥 - ∫ 𝑓 ′ (𝑥)𝑒 𝑥 𝑑𝑥 + ∫ 𝑓 ′ (𝑥)𝑒 𝑥 dx = e 𝑥 f(x) + C

1. Find∫ 𝑒 𝑥 (sin x + cos x) dx.


𝑥−1
2. Find∫ 𝑒 𝑥 ( 𝑥 2 ) 𝑑𝑥 .

3. Find∫ 𝑒 𝑥 (1 + x) dx.
𝜋
4. Find∫0 𝑒 𝑥 (tan x + sec 2 x) dx.

2 MARKS QUESTION

Q.NO. QUESTIONS MARKS


1 x + cos 6 x 2
Evaluate :  dx
3 x 2 + sin 6 x
2 2
Evaluate :  cos x dx
x

100
 x + x log x
3 Evaluate:
dx 2

4 Evaluate  xe x dx 2
1
5 Find the value of k if ∫0 (3𝑥 2 + 2𝑥 + 𝑘)=0 2
6 𝑑𝑥 2
Evaluate:-∫
√𝑥 2 −4𝑥
1
7 Evaluate:-∫0 𝑥(1 − 𝑥)89 dx 2
8 Evaluate:-∫ 𝑒 𝑥 (1 + 𝑡𝑎𝑛𝑥)𝑠𝑒𝑐𝑥 𝑑𝑥 2
9 Evaluate :∫(1 − 𝑥)√𝑥 dx 2
𝑥−1
10 Evaluate:-∫( 𝑥 2 ) ex dx 2
11 1 2
Evaluate: 
1
dx
2x + 3
0
2𝑐𝑜𝑠𝑥
12 Evaluate:-∫ 3 sin2 𝑥 2
13 sin2 𝑥−cos2 𝑥 2
Evaluate:-∫ 𝑑𝑥
𝑠𝑖𝑛𝑥 𝑐𝑜𝑠𝑥
14 sec2 (logx) 2
Evaluate:-∫ 𝑑𝑥
𝑥
15 Evaluate:∫1
√3 1
𝑑𝑥
1 + 𝑥2

3 MARKS QUESTION

Q.NO. QUESTIONS MARKS


1 sin( x − a ) 3
 sin( x + a ) dx (CBSE2013)
2 1 3
 x 2 + 2x + 10 dx Integration using standard results
3 2x + 1 3
 (x + 1)(x − 1) dx Integration using Partial Fraction
4 1 − x2 3
 x(1 − 2 x )
dx (CBSE-2010)

−1  1 − x 
5 2 3
 cos  
 1 + x 2 .dx
 
6
e (tan x + log sec x )dx 3
x

7 1 3
2x + 3
 5x 2 + 1 dx
0
8 4
3
0
( x + x − 2 + x − 4 )dx (CBSE-2013)

9 2 3
x x + 2 dx
0
10 / 4 3
 log (1 + tan x ) dx
0

101
11 𝑥𝑒 𝑥 3
𝐼𝑛𝑡𝑒𝑔𝑟𝑎𝑡𝑒 ( )
(1 + 𝑥)2

1
12 tan−1 𝑥 3
𝐸𝑣𝑎𝑙𝑢𝑎𝑡𝑒 ∫ 2
0 1+ 𝑥
13 3
∫√(3 − 2𝑥 − 𝑥 2 ) 𝑑𝑥

14 3
∫ 𝑒 2𝑥 sin 3𝑥 𝑑𝑥

5 MARKS QUESTION

Q.NO. QUESTIONS MARKS


1 5𝑥 + 3 5
∫ 𝑑𝑥
√𝑥 2 + 4𝑥 + 10
2 3𝑥 + 5 5
∫ 3 𝑑𝑥
𝑥 − 𝑥2 − 𝑥 + 1
3 5
∫(sin−1 𝑥)2 𝑑𝑥
𝜋
4 4 5
sin 𝑥 cos 𝑥
Evaluate: ∫ 𝑑𝑥
cos4 𝑥 + sin4 𝑥
0
𝜋
5 𝑥 tan 𝑥 5
Evaluate: ∫ 𝑑𝑥
sec 𝑥 + tan 𝑥
0

ANSWERS:

Q. NO ANSWER MARKS

1 (c) 1
𝑥 𝑎2
Explanation: = 2 √𝑥 2 + 𝑎2 + 𝑙𝑜𝑔(𝑥 + √(𝑥 2 + 𝑎2 )
2

Standard Formulae Can be done by By-Part taking '1' as the second function and
√(𝑥2 + 𝑎2 )as first function.

2 (d) none of these 1

102
3 b) π/4 1

4 3 1
b) 𝑙𝑜𝑔 |(𝑥 − 2) + √𝑥 2 − 3𝑥 + 2| + 𝐶

103
5 1 𝑥 𝜋 1
a) 2 𝑙𝑜𝑔𝑡𝑎𝑛 (2 + 3 ) + 𝐶

𝑥
6 b) 2 √𝑥 2 − 16 − 8𝑙𝑜𝑔|𝑥 + √𝑥 2 − 16| + 𝐶 1

2
7 b) 𝑥 + 𝑥 +𝑐 1
𝑡𝑎𝑛 +1
2

2
8 3
a) 2 𝑥 3 + 𝐶 1

104
9 b) tan x - cot x + C 1

𝜋
10 a) 8 𝑙𝑜𝑔2 1

11 1 2𝑐𝑜𝑠𝑥 1
b) − 2√3 𝑡𝑎𝑛−1 ( )+𝐶
√3

12 a) tan x - x + C 1

105
13 𝑏)𝑠𝑖𝑛−1 𝑥 − √1 − 𝑥 2 + 𝐶 1

2
14 c) 3 (𝑠𝑖𝑛𝑥)3/2 + 𝐶 1

1
15 c) 3 𝑡𝑎𝑛3 𝑥 + 𝑐 1

𝜋
16 d) 1
4

106
17 (c) A is true but R is false. 1

18 (a) Both A and R are true and R is the correct explanation of A. 1

19 (d) A is false but R is true 1

20 (a) Both A and R are true and R is the correct explanation of A. 1

107
21 4

108
22 4

109
23 4

110
24 4

111
ANSWER OF 2MARKS QUESTION

Q.NO. ANSWERS MARKS


1 Put 3𝑥 + sin 6𝑥 = t , So that (x + cos 6x ) dx = dt/6
2
2
∴I =1/6𝑙𝑜𝑔|3𝑥 2 + sin 6𝑥| + 𝐶

2 𝑑𝑥 2
Put √𝑥 = t , so that 𝑥= 2dt

∴I = 2𝑠𝑖𝑛√𝑥 + 𝐶

3 𝑑𝑥 2
Put 1 + log x = t so that = dt
𝑥

∴I =𝑙𝑜𝑔|1 + log 𝑥| + 𝐶

4 2
𝐼 = ∫ 𝑥𝑒 𝑥 𝑑𝑥 = 𝑥 ∫ 𝑒 𝑥 𝑑𝑥 − ∫ 1. (∫ 𝑒 𝑥 𝑑𝑥) 𝑑𝑥

= 𝑥𝑒 𝑥 − ∫ 𝑒 𝑥 𝑑𝑥
= 𝑥𝑒 𝑥 − 𝑒 𝑥 + C
1 1 1
5 ∫0 3𝑥 2 𝑑𝑥 +∫0 2𝑥 𝑑𝑥+𝑘 ∫0 𝑑𝑥 =0 2

 ⟦𝑥 3 ⟧10 +⟦𝑥 2 ⟧10 +⟦𝑘𝑥⟧10 = 0


 1+1+k=0
K=-2
6 √𝑥 2 − 4𝑥 = √{(𝑥 − 2)2 − 22 } 2
𝑑𝑥
∴∫ = 𝑙𝑜𝑔|𝑥 − 2 + √{(𝑥 − 2)2 − 22 }| + 𝐶
√{(𝑥−2)2 −22 }

= 𝑙𝑜𝑔 |𝑥 − 2 + √(𝑥 2 − 4𝑥)| + 𝐶

112
𝑎 𝑎
7 2
𝑢𝑠𝑖𝑛𝑔 𝑝𝑟𝑜𝑝𝑒𝑟𝑡𝑖𝑒𝑠 ∫ 𝑓(𝑥)𝑑𝑥 = ∫ 𝑓(𝑎 − 𝑥)𝑑𝑥, 𝑤𝑒 𝑔𝑒𝑡
0 0
1 1 1
𝐼 = ∫0 (1 − 𝑥)𝑥 89 dx=∫0 𝑥 89 𝑑𝑥 -∫0 𝑥 90 𝑑𝑥
1 1
=90 − 91
1
=8190

8 2
𝑤𝑒 ℎ𝑎𝑣𝑒 𝐼 = ∫ 𝑒 𝑥 (𝑠𝑒𝑐𝑥 + 𝑠𝑒𝑐𝑥 𝑡𝑎𝑛𝑥) 𝑑𝑥

𝐿𝑒𝑡 𝑓(𝑥) = 𝑠𝑒𝑐𝑥 𝑠𝑜 𝑡ℎ𝑎𝑡 𝑓 ′ (𝑥) = 𝑠𝑒𝑐𝑥 𝑡𝑎𝑛𝑥


∴ 𝐼 = ∫ 𝑒 𝑥 [𝑓(𝑥) + 𝑓 ′ (𝑥)] 𝑑𝑥 = 𝑒 𝑥 𝑓(𝑥)+C
∴ 𝐼 = 𝑒 𝑥 sec𝑥 +c
1 3
9 2
𝐼 = ∫ (𝑥 2 − 𝑥 2 ) 𝑑𝑥

2 3/2 2 5/2
= 𝑥 − 𝑥 + 𝑐
3 5

1 1
10 𝐼 = ∫(𝑥 − 𝑥 2 ) ex dx 2

1 1
𝐿𝑒𝑡 𝑓(𝑥) = 𝑠𝑜 𝑡ℎ𝑎𝑡 𝑓 ′ (𝑥) = − 2
𝑥 𝑥

∴ 𝐼 = ∫ 𝑒 𝑥 [𝑓(𝑥) + 𝑓 ′ (𝑥)] 𝑑𝑥

= 𝑒 𝑥 𝑓(𝑥)+C (∵∫ 𝑒 𝑥 [𝑓(𝑥) + 𝑓 ′ (𝑥)] 𝑑𝑥 = 𝑒 𝑥 𝑓(𝑥)+C )


1
= 𝑒 𝑥 . 𝑥+ C
𝑒𝑥
= +𝐶
𝑥
11 𝑑𝑧 2
Putting 2𝑥 + 3 = 𝑧 𝑠𝑜 𝑡ℎ𝑎𝑡 𝑑𝑥 = 2

𝐴𝑙𝑠𝑜 𝑤ℎ𝑒𝑛 𝑥 = 0 𝑡ℎ𝑒𝑛 𝑧 = 3 𝑎𝑛𝑑 𝑤ℎ𝑒𝑛 𝑥 = 1 𝑡ℎ𝑒𝑛 𝑧 = 5


5
1 𝑑𝑧
∴𝐼= ∫
3 √𝑧 2
5
= ⟦√𝑧⟧3

= √5 − √3
12 2 𝑐𝑜𝑠𝑥 𝑐𝑜𝑒𝑐𝑥 𝑑𝑥 2

3 𝑠𝑖𝑛𝑥
2 2
=3 ∫ 𝑐𝑜𝑡𝑥 𝑐𝑜𝑠𝑒𝑐𝑥 𝑑𝑥 = − 𝑐𝑜𝑠𝑒𝑐𝑥 +C
3
13 2𝑐𝑜𝑠2𝑥 2
−∫ 𝑑𝑥
𝑠𝑖𝑛2𝑥
= -2∫ 𝑐𝑜𝑡2𝑥𝑑𝑥

113
1
= -2 . 2 log|𝑠𝑖𝑛2𝑥| +C

= log|𝑐𝑜𝑠𝑒𝑐 2𝑥| + C

14 Put 𝑙𝑜𝑔𝑥 = 𝑡 2
𝑠𝑜 𝑡ℎ𝑎𝑡 𝑑𝑥/𝑥 = 𝑑𝑡

∴ 𝐼 = ∫ sec 2 𝑡 𝑑𝑡 = 𝑡𝑎𝑛𝑡 + 𝐶

= 𝑡𝑎𝑛(𝑙𝑜𝑔𝑥) + 𝐶
15 √3
1
𝐼= ∫ 𝑑𝑥
1 + 𝑥2
1

= ⟦𝑡𝑎𝑛−1 𝑥⟧1√3
= 𝑡𝑎𝑛−1 √3 − 𝑡𝑎𝑛−1 1
𝜋 𝜋 𝜋
= − =
3 4 12

ANSWERS OF 3 MARKS QUESTION

Q.NO. ANSWERS MARKS


1 I = {sin(x-a)/ sin (x +a)} dx 3
put x + a=t
so dx=dt
I={sin(t-2a) /sint } dt (sin(a-b)=sina.cosb – cosa.sinb)
I = {(sintcos2a -costsin2a)/sint } dt
I = {cos2a - cottsin2a} dt
I= {tcos2a -sin2alogsint} +c
I= {(x + a) cos2a - sin2a.logsin(x + a)} + c
2 1 3
∫ 𝑑𝑥
𝑥 2 + 2𝑥 + 10
1
=∫ 𝑑𝑥
(𝑥+1)2 +9
1
=∫ 𝑑𝑥
(𝑥+1)2 +32

1 (𝑥+1)
= tan−1 +c
3 3

3 2𝑥 + 1 3
∫ 𝑑𝑥
(𝑥 + 1)(𝑥 − 1)
3 −1
=∫[(2)(𝑥−1) + (−2)(𝑥+1)]𝑑𝑥 By Partial Fraction

114
3 1
= log(𝑥 − 1) + log(𝑥 + 1)+ c
2 2

4 1 − 𝑥2 3
∫ 𝑑𝑥
𝑥(1 − 2𝑥)
𝑥 2 −1
=∫ 𝑑𝑥
𝑥(2𝑥−1)
1 1 2 3
=∫[ + ( − )𝑑𝑥 [by splitting]
2 2 𝑥 2𝑥−1
1 3
= 𝑥 + log|𝑥| + log(2𝑥 − 1) + c
2 4

−1  1 −
x2 
5 3
 cos  1 + x 2 .dx

= ∫ 2tan−1 𝑥 𝑑𝑥

d
= 2[tan−1 𝑥 ∫ 1𝑑𝑥 - ∫{ (tan−1 𝑥) ∫ 1𝑑𝑥}𝑑𝑥]
dx
x
=2[tan−1 𝑥 . 𝑥 − ∫ 𝑑𝑥]
1+𝑥 2

=2[𝑥. tan−1 𝑥 – (½) log(1 + 𝑥 2 )] + c


=2x tan−1 𝑥- log (1 + 𝑥 2 )] + c

6
e (tan x + log sec x )dx 3
x

Using ∫ 𝑒 𝑥 (𝑓(𝑥) + 𝑓 ′ (𝑥))𝑑𝑥 = 𝑒 𝑥 (𝑓(𝑥) + 𝐶

=𝑒 𝑥 tan−1 𝑥 + 𝐶

7 1 3
2x + 3
 5x 2 + 1 dx
0

1 2𝑥 3
=∫0 [ 2 + 2 ]𝑑𝑥
5𝑥 +1 5𝑥 +1
1 3
=[ log(5𝑥 2 + 1) + tan−1 √5 𝑥] 𝑓𝑟𝑜𝑚 0 𝑡𝑜 1
5 √5
1 3
=[ log(6) + tan−1 √5]
5 √5

8 4
3
0
( x + x − 2 + x − 4 )dx

115
4 2 4 4
=∫0 𝑥𝑑𝑥 − ∫0 (𝑥 − 2)𝑑𝑥 + ∫2 (𝑥 − 2)𝑑𝑥 − ∫0 (𝑥 − 4)𝑑𝑥

=(8-0)+(4-2) +(8-8)+(-2+4)+(16-8) after integration & substitution


=20
9 2 3
 x x + 2 dx
0

1
2
=∫0 {(𝑥 + 2)3/2 − 2(𝑥 + 2)2 } 𝑑𝑥
3
2 5/2 4
= [ (𝑥 + 2) − (𝑥 + 2) ] from 0 to 2
2
5 3
2 4 3/2 2 4
= [ 45/2 - 4 ] – [ 25/2 - 23/2]
5 3 5 3

32(1−2√2)
=
15

10 / 4 3
I=  log (1 + tan x ) dx ---------(i)
0

𝜋/4 𝜋
=∫0 log(1 + tan ( − 𝑥))𝑑𝑥 by P4
4
𝜋/4
=∫0 log(2/(1 + 𝑡𝑎𝑛𝑥 ))𝑑𝑥---------(ii)

Adding both we get


𝜋/4
2I = ∫0 log(2) 𝑑𝑥
𝜋/4
Or 2I = log2 ∫0 1. 𝑑𝑥

Or 2I = log2 [x] from 0 to 𝜋/4


𝜋
Or 2I = (𝑙𝑜𝑔2)
4
𝜋
Or I = log 2
8

11 𝑥𝑒 𝑥 3
∫ ( ) 𝑑𝑥
(1 + 𝑥)2
1 1
= ∫ 𝑒 𝑥 (1+𝑥 − (1+𝑥)2 ) 𝑑𝑥
1
=𝑒 𝑥 1+𝑥
+ c byUsing ∫ 𝑒 𝑥 (𝑓(𝑥) + 𝑓 ′ (𝑥))𝑑𝑥 = 𝑒 𝑥 (𝑓(𝑥) + 𝐶

116
12 1 tan−1 𝑥 3
𝐼 = ∫0 dx
1+ 𝑥 2

Put tan−1 𝑥 = 𝑡& dt = dx/(1 + 𝑥 2 )


x→ 0, 𝑡 → 0 & 𝑥 → 1, 𝑡 → 𝜋/4
𝜋/4
I = ∫0 𝑡 𝑑𝑡
𝜋
= [t2/2] from 0 to
4
𝜋2
=
32

13 3
∫√(3 − 2𝑥 − 𝑥 2 ) 𝑑𝑥

=∫ √{4 − (𝑥 + 1)2 } 𝑑𝑥 By method of completing square


(𝑥+1) 4 (𝑥+1)
= √(3 − 2𝑥 − 𝑥 2 ) + sin−1 +c Using ∫ √{𝑎2 − 𝑥 2 } 𝑑𝑥
2 2 2
(𝑥+1) 4 (𝑥+1)
= √(3 − 2𝑥 − 𝑥 2 ) + sin−1 +c
2 2 2

14 3
𝐼 = ∫ 𝑒 2𝑥 sin 3𝑥 𝑑𝑥

𝑒 2𝑥
= [ 2𝑠𝑖𝑛3𝑥 − 𝑏𝑐𝑜𝑠3𝑥] + 𝑐
(22 +32)
𝑒 𝑎𝑥
using ∫ 𝑒 𝑎𝑥 sin 𝑏𝑥 𝑑𝑥 = (𝑎2 +𝑏2) [𝑎𝑠𝑖𝑛 𝑏𝑥 − 𝑏𝑐𝑜𝑠𝑏𝑥] + 𝑐 by parts special case

𝑒 2𝑥
= [ 2𝑠𝑖𝑛3𝑥 − 𝑏𝑐𝑜𝑠3𝑥] + 𝑐
13

SOLUTIONS OF 5 MARKS QUESTION

Q.NO. ANSWERS MARKS


1 5𝑥+3 5
Let I = ∫ √𝑥 2 𝑑𝑥
+4𝑥+10

Let,
𝑑 2
5𝑥 + 3 = A (𝑥 + 4𝑥 + 10) + B
𝑑𝑥
⇒ 5𝑥 + 3 = A(2𝑥 + 4) + B … … … … (1)

⇒ 5𝑥 + 3 = 2 A 𝑥 + 4 A + B

117
Equating the coefficients of x and constants,

5 = 2A and 3 = 4A + B
5 5
⇒A= and 3 = 4. + B
2 2
5
⇒A= and 3 = 10 + B
2
5
⇒A= and B = −7
2
From (1), we get

5
5𝑥 + 3 = (2𝑥 + 4) − 7
2
Thus, the given integral changes to
5
2
(2𝑥 + 4) − 7
I=∫ 𝑑𝑥
√𝑥 2 + 4𝑥 + 10
5
(2𝑥 + 4) 7
2
= ∫[ − ] 𝑑𝑥
√𝑥 2 + 4𝑥 + 10 √𝑥 2 + 4𝑥 + 10

5 (2𝑥 + 4) 1
= ∫ 𝑑𝑥 − 7 ∫ 𝑑𝑥
2 √𝑥 + 4𝑥 + 10
2 2
√𝑥 + 4𝑥 + 10
5
I = I1 − 7 I2 … … … … … (2)
2
Where,

(2𝑥 + 4)
I1 = ∫ 𝑑𝑥
√𝑥 2 + 4𝑥 + 10
Let,
𝑥 2 + 4𝑥 + 10 = 𝑡

⇒ (2𝑥 + 4) 𝑑𝑥 = 𝑑𝑡

1
∴ I1 = ∫ 𝑑𝑡
√𝑡

I1 = 2√𝑡 + C1

I1 = 2√𝑥 2 + 4𝑥 + 10 + C1 … … … … … (3)

And,

1
I2 = ∫ 𝑑𝑥
√𝑥 2 + 4𝑥 + 10

118
1
⇒ I2 = ∫ 𝑑𝑥
√𝑥 2 + 2. 𝑥. 2 + 22 − 22 + 10
1
⇒ I2 = ∫ 𝑑𝑥
√(𝑥 + 2)2 − 4 + 10

1
⇒ I2 = ∫ 𝑑𝑥
√(𝑥 + 2)2 + 6

1
⇒ I2 = ∫ 𝑑𝑥
2
√(𝑥 + 2)2 + (√6)

2 3𝑥 + 5 5
I=∫ 𝑑𝑥
𝑥3 − 𝑥2 − 𝑥 + 1
3𝑥 + 5
=∫ 2 𝑑𝑥
𝑥 (𝑥 − 1) − 1(𝑥 − 1)

3𝑥 + 5
=∫ 𝑑𝑥
(𝑥 − 1)(𝑥 2 − 1)

3𝑥 + 5
=∫ 𝑑𝑥
(𝑥 − 1)(𝑥 − 1)(𝑥 + 1)

3𝑥 + 5
I=∫ 𝑑𝑥 … … … … … . (1)
(𝑥 − 1)2 (𝑥 + 1)

Let

3𝑥 + 5 A B C
= + + … … … … … (2)
(𝑥 − 1)2 (𝑥 + 1) 𝑥 − 1 (𝑥 − 1)2 𝑥 + 1

3𝑥 + 5 A(𝑥 − 1)(𝑥 + 1) + B(𝑥 + 1) + C(𝑥 − 1)2


⇒ =
(𝑥 − 1)2 (𝑥 + 1) (𝑥 − 1)2 (𝑥 + 1)

⇒ 3𝑥 + 5 = A(𝑥 − 1)(𝑥 + 1) + B(𝑥 + 1) + C(𝑥 − 1)2 … … … … (3)

Putting 𝑥 = 1

3.1 + 5 = A(1 − 1)(1 + 1) + B(1 + 1) + C(1 − 1)2

⇒ 8 = A(0)(2) + B(2) + C(0)

⇒ 8 = 2B

∴𝐵=4

Putting 𝑥 = −1

3. (−1) + 5 = A(−1 − 1)(−1 + 1) + B(−1 + 1) + C(−1 − 1)2

⇒ −3 + 5 = A(0) + B(0) + C. (4)


119
⇒ 2 = 4C

2 1
⇒C= =
4 2

From Equation (3),

3𝑥 + 5 = A(𝑥 − 1)(𝑥 + 1) + B(𝑥 + 1) + C(𝑥 − 1)2

⇒ 3𝑥 + 5 = A(𝑥 2 − 1) + B(𝑥 + 1) + C(𝑥 2 − 2𝑥 + 1)

⇒ 3𝑥 + 5 = 𝐴 𝑥 2 − 𝐴 + 𝐵 𝑥 + 𝐵 + 𝐶𝑥 2 − 2𝐶𝑥 + 𝐶

⇒ 3𝑥 + 5 = 𝑥 2 (A + C) + 𝑥(B − 2C) + (−A + B + C)

3 Let I = ∫(sin−1 𝑥)2 𝑑𝑥 5


Let sin−1 𝑥 = 𝑡

⇒ 𝑥 = sin 𝑡

⇒ 𝑑𝑥 = cos 𝑡 𝑑𝑡

∴ I = ∫ 𝑡 2 cos 𝑡 𝑑𝑡

Integrating by parts taking 𝑡 2 as first function and cos 𝑡 as second function

𝑑 2
I = 𝑡 2 ∫ cos 𝑡 𝑑𝑡 − ∫ [ (𝑡 ) ∫ cos 𝑡 𝑑𝑡] 𝑑𝑡
𝑑𝑡

= 𝑡 2 . sin 𝑡 − ∫[2𝑡 × sin 𝑡] 𝑑𝑡

= 𝑡 2 sin 𝑡 − 2 ∫ 𝑡 sin 𝑡 𝑑𝑡

= 𝑡 2 sin 𝑡 − 2 ∫ 𝑡 sin 𝑡 𝑑𝑡

Again, integrating by parts taking t as first function and sin 𝑡 as second function

I = 𝑡 2 sin 𝑡 − 2 ∫ 𝑡 sin 𝑡 𝑑𝑡

𝑑
= 𝑡 2 sin 𝑡 − 2 [𝑡 ∫ sin 𝑡 𝑑𝑡 − ∫ { (𝑡) ∫ sin 𝑡 𝑑𝑡} 𝑑𝑡]
𝑑𝑡

= 𝑡 2 sin 𝑡 − 2 [𝑡(− cos 𝑡) − ∫{1. (− cos 𝑡)} 𝑑𝑡]

= 𝑡 2 sin 𝑡 − 2 [−𝑡 cos 𝑡 + ∫ cos 𝑡 𝑑𝑡]

120
= 𝑡 2 sin 𝑡 − 2 [−𝑡 cos 𝑡 + sin 𝑡] + 𝐶

= 𝑡 2 sin 𝑡 + 2 𝑡 cos 𝑡 − 2 sin 𝑡 + 𝐶

= 𝑡 2 𝑠𝑖𝑛 𝑡 + 2 𝑡 √1 − 𝑠𝑖𝑛2 𝑡 − 2 𝑠𝑖𝑛 𝑡 + 𝐶

= (𝑠𝑖𝑛−1 𝑥)2 𝑥 + 2 𝑠𝑖𝑛−1 𝑥 √1 − 𝑥 2 − 2 𝑥 + 𝐶 [∵ sin−1 𝑥 = 𝑡 𝑎𝑛𝑑 𝑥 = sin 𝑡]

= 𝑥(𝑠𝑖𝑛−1 𝑥)2 + 2 𝑠𝑖𝑛−1 𝑥 √1 − 𝑥 2 − 2 𝑥 + 𝐶


𝜋
4 sin 𝑥cos 𝑥
LetI = ∫04 cos4𝑥+sin4𝑥 𝑑𝑥 5
Dividing num. and denominator by cos 4 𝑥
𝜋
4 sin 𝑥cos 𝑥
4
⇒ I = ∫ cos4cos 𝑥
𝑥+sin4 𝑥
𝑑𝑥
0 cos4 𝑥
𝜋
4
tan 𝑥 sec 2 𝑥
⇒I=∫ 𝑑𝑥
1 + tan4 𝑥
0
𝜋
4
tan 𝑥 sec 2 𝑥
⇒I=∫ 𝑑𝑥
1 + (tan2 𝑥)2
0
Let tan2 𝑥 = 𝑡
⇒ 2 tan 𝑥 sec 2 𝑥 𝑑𝑥 = 𝑑𝑡
1
⇒ tan 𝑥 sec 2 𝑥 𝑑𝑥 = 𝑑𝑡
2
𝜋
When 𝑥 = then t = tan2 𝜋4 = 12 = 1
4
When 𝑥 = 0 then t = tan2 0 = 02 = 0
1 1 1
I= ∫ dt
2 0 1 + 𝑡2
1
⇒ I = [tan−1 𝑡]10
2
1
⇒ I = [tan−1 1 − tan−1 0]
2
1 𝜋
⇒ I = [ − 0]
2 4
𝜋
⇒ I = Ans.
8
𝜋 𝑥 tan 𝑥
5 Let I = ∫0 sec 𝑥+tan 𝑥 𝑑𝑥 … … … … … (1) 5
𝜋
(𝜋 − 𝑥). tan(𝜋 − 𝑥)
⇒I=∫ 𝑑𝑥
sec(𝜋 − 𝑥) + tan(𝜋 − 𝑥)
0
𝜋
(𝜋 − 𝑥)(− tan 𝑥)
⇒I=∫ 𝑑𝑥
(−sec 𝑥) + (− tan 𝑥)
0

𝜋
−(𝜋 − 𝑥)tan 𝑥
⇒I=∫ 𝑑𝑥
−(sec 𝑥 + tan 𝑥)
0
𝜋
(𝜋 − 𝑥)tan 𝑥
⇒I=∫ 𝑑𝑥
(sec 𝑥 + tan 𝑥)
0
121
𝜋 𝜋
𝜋 tan 𝑥 𝑥 tan 𝑥
⇒I=∫ 𝑑𝑥 − ∫ 𝑑𝑥
(sec 𝑥 + tan 𝑥) (sec 𝑥 + tan 𝑥)
0 0
𝜋
tan 𝑥
⇒ I = 𝜋∫ 𝑑𝑥 − I [Using (1)]
sec 𝑥 + tan 𝑥
0
𝜋
tan 𝑥
⇒ 2I = 𝜋 ∫ 𝑑𝑥
sec 𝑥 + tan 𝑥
0
𝜋
tan 𝑥 sec 𝑥 − tan 𝑥
⇒ 2I = 𝜋 ∫ × 𝑑𝑥
sec 𝑥 + tan 𝑥 sec 𝑥 − tan 𝑥
0
𝜋
tan 𝑥(sec 𝑥 − tan 𝑥)
⇒ 2I = 𝜋 ∫ 𝑑𝑥
sec 2 𝑥 − tan2 𝑥
0
𝜋
tan 𝑥(sec 𝑥 − tan 𝑥)
⇒ 2I = 𝜋 ∫ 𝑑𝑥 [∵ sec 2 𝑥 − tan2 𝑥 = 1]
1
0
𝜋

⇒ 2I = 𝜋 ∫[sec 𝑥 tan 𝑥 − tan2 𝑥] 𝑑𝑥


0
𝜋

⇒ 2I = 𝜋 ∫[sec 𝑥 tan 𝑥 − (sec 2 𝑥 − 1)] 𝑑𝑥


0
𝜋

⇒ 2I = 𝜋 ∫[sec 𝑥 tan 𝑥 − sec 2 𝑥 + 1] 𝑑𝑥


0
⇒ 2I = 𝜋[sec 𝑥 − tan 𝑥 + 𝑥]𝜋0
⇒ 2I = 𝜋[(sec 𝜋 − tan 𝜋 + 𝜋) − (sec 0 − tan 0 + 0)]
⇒ 2I = 𝜋[(−1 − 0 + 𝜋) − (1 − 0 + 0)]
⇒ 2I = 𝜋[−1 + 𝜋 − 1]
⇒ 2I = 𝜋(𝜋 − 2)
𝜋
∴I= (𝜋 − 2)
2

CHAPTER-8: APPLICATION OF INTEGRALS


2 MARKS QUESTION

Q.NO. QUESTIONS MARKS


1 Find the area of the curve y = sin x between 0 and π 2
2 Find the area of the region bounded by the curve ay2 = x 3 , the y-axis and the lines y 2
= a and y = 2a.
3 Find the area of the region bounded by the parabola y2 = 2x and the straight line x – 2
y = 4.
4 Find the area of the region bounded by the parabolas y2 = 6x 2
and x2 = 6y
122
5 𝑥2 𝑌2 2
Find the area enclosed by the curve + =1
9 4

3 MARKS QUESTION

Q.NO. QUESTIONS MARKS


1 Find the area of the region bounded by the curves y = x2 + 2, y = x, x = 0 and x = 3. 3
2 Find the area between the curve y = |x + 3|, the x - axis and the lines x = - 6 and x = 0. 3

3 Sketch the region{(𝑥, 0): 𝑦 = √4 − 𝑥 2 } and x - axis. Find the area of the region using 3
integration.
4 𝑥2 𝑦2 3
Find the area enclosed by the ellipse𝑎2 + =1
𝑏2
5 Find the area of the region{(𝑥, 𝑦): 𝑥 2 ≤ 𝑦 ≤ 𝑥} 3
6 Find the area bounded by the line y = x, the x - axis and the ordinates x = - 1, x = 2. 3
7 The slope of tangent to a curve y=f(x) at (x, f(x)) is 2x+1. If the curve passes through 3
the point (1,2), then find the area bounded by the curve, the X - axis and the line
x=1.
8 Find the area of the region lying in the first quadrant and bounded by y = 4x 2 , x = 0, y = 3
1 and y = 4.
9 Sketch the region {(x, y ): 9x2 + 4y 2 = 36} and find the area of the region enclosed by it, 3
using integration.
10 Find the area of the region bounded by the parabola y2 = 4x, the x - axis, and the lines x 3
= 1 and x = 4.
11 Find the area of the region bounded by x2 = 16y, y = l, y = 4 and the y - axis in the 3
first quadrant.
12 Find the area of the region enclosed by the parabola x2 = y and the line y = x + 2 3
13 𝑥2 𝑦2 3
Evaluate the area bounded by the ellipse 4 + = 1 above the x - axis.
9
14 Find the area of the region bounded by the curves y2 = 9x, y = 3x. 3
15 Find the area of the region bounded by the curve𝑦 = 𝑥 2 and the line y = x. 3

5 MARKS QUESTION

Q.NO. QUESTIONS MARKS


3𝑥 2
1 Find the area of the region included between the parabola y = 4 and the line 3x – 2y 5
+ 12 = 0.
𝑎
2 Find the area of a minor segment of the circle x2 + y2 = a2 cut off by the line x=2 5
3 Find the area of region bounded by the triangle whose vertices are (–1, 1), (0, 5) 5
and (3, 2), using integration.
4 Compute the area bounded by the lines x + 2y = 2, 2x + y = 7 5

123
and y – x = 1.
5 Find the area bounded by the lines y = 4x + 5, y = 5 – x and 4y = x + 5. 5
6 Find the area of the region bounded by the curves x = at2 and y = 2at between the 5
ordinate corresponding to t = 1 and t = 2.
0
7 Sketch the graph of y = |𝑥 + 3| and evaluate∫−6|𝑋 + 3|𝑑𝑥. 5

8 Using integration, find the area of the triangle whose sides are given by2x + y = 4, 5
3x – 2y = 6 andx – 3y+ 5 = 0.
9 Find the area of the region enclosed between curves y = |x − 1| and 5
y= 3 − |x|.
10 Using integration, find the area of the triangle whose vertices are 5
(–1, 0),(1, 3) and (3, 2).

ANSWERS

ANSWER OF 2 MARKS QUESTIONS


Q.NO. ANSWERS MARKS
1 𝜋 𝜋 𝜋 2
We have Area OAB =∫0 𝑦𝑑𝑥 =∫0 𝑠𝑖𝑛𝑥𝑑𝑥 = [−𝑐𝑜𝑠𝑥] = cos0 – cosπ = 2 sq units.
0

2𝑎 2𝑎
2 We have Area BMNC=∫𝑎 𝑥𝑑𝑦 = ∫𝑎 𝑎1/3 𝑦 2/3 𝑑𝑦 2

3𝑎1/3
= [𝑦 5/3 ]2𝑎𝑎 = 3/5a2[2. 22/3 − 1]sq units
5

124
3 The intersecting points of the given curves are obtained by solving the equations 2
x – y = 4 and y2 = 2x for x and y.
We have y2 = 8 + 2y
i.e., (y – 4) (y + 2) = 0
which gives y = 4, –2 and x = 8, 2.
Thus, the points of intersection are (8, 4), (2, –2).
4 1 𝑦2 1
Hence Area = ∫−2 (4 + 𝑦 − 2 𝑦 2 )dy= [4𝑦 + − 6 𝑦 3 ] 4 − 2= 18 sq units.
2

4 The intersecting points of the given parabolas are obtained by solving these equations 2
for x and y, which are 0(0, 0) and (6, 6).
6 𝑥2 𝑥 2/3 𝑥3
Hence Area OABC = ∫0 (√6𝑥 − )dx=[2√6 − 18] 60 = 12 sq units.
6 3

5 3 2 8 𝑥 9𝑥 2
The required area = 4 ∫0 √9 − 𝑥 2 𝑑𝑥 =3 [2 √9 − 𝑥 2 + 2 3] 30 =6 π sq units.
3

125
ANSWERS OF 5 MARKS QUESTION

Q.NO. ANSWERS MARKS


1 3𝑥 2 5
Solving the equations of the given curves y = 4 and 3x-2x + 12 = 0,

we get 3x2 – 6x – 24 = 0 ⇒ (x – 4) (x + 2) = 0
⇒ x = 4, x = –2 which give y = 12, y = 3
4 12𝑥+3 3𝑥 2
The required area = area of ABC =∫−2 [ dx– dx ]= 27 sq units.
2 4

𝑎
2 Solving the equation x 2 + y 2 = a 2 and x = , we obtain their points of intersection 5
2
𝑎 𝑎 𝑎 𝑎
which are (2 , √3 𝑎)and (2 , −√3 𝑎)
𝑎
Hence Required Area = 2Area of OAB = 2∫𝑎/2 √𝑎2 − 𝑥 2 dx

𝑥 𝑎2 𝑥
= 2[2 √𝑎2 − 𝑥2 + 2 𝑎
] 𝑎𝑎/2

126
3 Let we have the vertices of △ABC as A(−1,1),B(0,5) and C(3,2). 5
𝟓−𝟏
∴ Equation of AB is y−1=(𝟎+𝟏)(x+1)
⇒y−1=4x+4
⇒y=4x+5....(i)
𝟐−𝟓
And equation of BC is y−5=(𝟑−𝟎)(x−0)
⇒y−5=−3/3(x)
⇒y=5−x....(ii)
𝟐−𝟏
Similarly, equation of AC is y−1=(𝟑+𝟏)(x+1)
⇒y−1=1/4(x+1)
⇒4y=x+5....(iii)
𝟎 𝟑
∴ Area of shaded region =∫−𝟏 (𝒚𝟏 − 𝒚𝟐)𝒅𝒙+∫𝟎 (𝒚𝟏 − 𝒚𝟐)𝒅𝒙
𝟎 𝒙+𝟓 𝟑 𝒙+𝟓
=∫−𝟏 (𝟒𝒙 + 𝟓 − ) 𝒅𝒙+∫𝟎 (𝟓 − 𝒙 − ) 𝒅𝒙
𝟒 𝟒
𝟒𝒙𝟐 𝒙𝟐 𝟓 𝒙𝟐 𝒙𝟐 𝟓
=[ + 𝟓𝒙 − − 𝟒 𝒙] 𝟎 − 𝟏+[𝟓𝒙 − − − 𝟒 𝒙] 𝟑𝟎
𝟐 𝟖 𝟐 𝟖
𝟒 𝟏 𝟓 𝟗 𝟗 𝟏𝟓
=[𝟎 − (𝟐 + 𝟓(−𝟏) − 𝟖 + 𝟒)]+[𝟏𝟓 − 𝟐 − 𝟖 − − 𝟎]
𝟒
𝟏𝟓
= 𝟐 sq units.

4 Given equation are 5


x + 2y = 2…..(i)
2x + y = 7….(ii)

127
and y – x = 1….(iii)
(i) x + 2y = 2

x 0 2

y 1 0

(0,1);(2,0)

(ii) 2x + y = 7

x 0 27

y 7 0

(0,7);(27,0)

(iii) y – x = 1

x 0 −1

y 1 0

(0,1);(−1,0)

Now, we find points of intersection


1) (I) and (II)
x+2y=2,y+2x=7
y=7−2x
⇒x+2[7−2x]=2
x+14−4x=2
−3x=2−14=−12
x=12/3=4
y=7−2(4)=7−8=−1
∴ point A(4,−1)

2) (II) and (III)


y+2x=7, y−x=1

128
y=x+1
⇒x+1+2x=7
3x=6
x=2
⇒y=2+1=3
Point B(2,3)
3) (I) and (III)
x+2y=2, y−x=1
y=x+1
⇒x+2[x+1]=2
x+2x+2=2
3x=0
x=0
⇒y=0+1=1
Point C(0,1)

We draw graph
To find shaded tregion
2 2 0
ar(△ABC)= ∫0 𝑦𝐶𝐵𝑑𝑥+∫4 𝑦𝐵𝐴. 𝑑𝑥 − ∫4 𝑦𝐴𝐶. 𝑑𝑥
2 2 0 2−𝑥
= ∫0 (𝑥 + 1)𝑑𝑥 +∫4 (7 − 2𝑥). 𝑑𝑥 − ∫4 𝑑𝑥
2

𝑥2 𝑥2 1 𝑥2
=[ 2 + 𝑥] 20 +[7𝑥 − 2 2 ] 24 − 2 [2𝑥 − ] 40
2

=[2+2]−[0]+[14−4]−[28−16]−1/2[0]+1/2[8−8]
=4+10−12+0
=14−12
=2sq.units

5 Given equations of lines are 5


y=4x+5....(i)
129
y=5−x....(ii) and
4y=x+5....(iii)
On solving Eqs. (i) and (ii), we get
4x+5=5−x
⇒x=0
On solving Eqs. (i) and (iii)
4(4x+5)=x+5
⇒16x+20=x+5
⇒15x=−15
⇒x=−1
On solving Eqs. (ii) and (iii), we get
4(5−x)=x+5
⇒20−4x=x+5
⇒x=3

0 3 1 3
∴ Required area =∫−1 (4𝑥 + 5)𝑑𝑥 +∫0 (5 − 𝑥)𝑑𝑥 − 4 ∫−1 (𝑥 + 5)𝑑𝑥

𝑥2 𝑥2 1 𝑥2
=[4 2 + 5𝑥]0 − 1 + [5𝑥 − ] 30 − 4 [ 2 + 5𝑥]3 − 1
2
=[0−2+5]+[15−9/2−0]−1/4[9/2+15−1/2+5]
=3+21/2222-1/4(24)
=21/2−3=15/2sq units.

6 5

Given that x = at2 ...(i),


y = 2at ...(ii)
⇒ t = y/2a putting the value of t in (i),
we get y 2 = 4ax

130
Putting t = 1 and t = 2 in (i),
we get x = a, and x = 4a
Required area = 2 area of ABCD
4𝑎 4𝑎
= 2∫𝑎 𝑦𝑑𝑥= 2×2∫𝑎 √𝑎𝑥𝑑𝑥
𝑥 3/2
=8√𝑎 | | 4𝑎𝑎
3
56
= 𝑎2 sq. units.
3

7 5

8 The given equations of lines are 5


2x+y=4.........(1)
3x−2y=6.........(2)
131
And, x−3y+5=0..........(3)
The area of the region bounded by the lines is the area of ΔABC. AL and CM are the
perpendicular on x-axis.
Area(ΔABC)=Area(ALMCA)−Area(ALB)−Area(CMB)

𝟒 𝟐 𝟒
=∫𝟏 (𝒙 + 𝟓)/𝟑𝒅𝒙−∫𝟏 (𝟒 − 𝟐𝒙)𝒅𝒙−∫𝟐 (𝟑𝒙 − 𝟔)/𝟐dx
=1/3[x2/2 +5x]𝟒𝟏 -[4x-2x2/2]𝟐𝟏-[3x2/2 -6x]𝟒𝟐
=1/3[8+20-1/2-5] - [8-4-4+1]- ½[24-24-6+12]
=(1/3×45/2)−(1)−1/2(6)
=15/2−1−3
=7/2 sq.units.

9 5

Intersection point of y=x−1 and y=3−x is (2,1)


and intersection point of y=−x+1 and y=3+x is (−1,2)
0 1 2
A=∫−1 [(3 + 𝑥) − (−𝑥 + 1)]𝑑𝑥 + ∫0 [(3 − 𝑥) − (−𝑥 + 1)]𝑑𝑥 + ∫1 [(3 − 𝑥) −
(𝑥 − 1)]𝑑𝑥
0 1 2
=∫−1 [2 + 2𝑥]𝑑𝑥 + ∫0 [2]𝑑𝑥 + ∫1 [4 − 2𝑥]𝑑𝑥

=[2x+x2]0 − 1+2[x]10+[4x-x2]21
=0−(−2+1)+2(1−0)+(8−4)−(4−1)
=1+2+4−3=4 sq. units

132
10 5

BL and CM are drawn perpendicular to x-axis.


It can be observed from the figure that,
Area(ΔACB)=Area(ALBA)+Area(BLMCB)−Area(AMCA) ......... (1)
Equating of line segment AB is
3−0
y−0= (x+1)
1+1
3
y=2(x+1)

𝟏 𝟑 𝟑 𝒙𝟐 𝟑 𝟏 𝟏
∴Area(ALBA)=∫−𝟏 (x+1)dx=𝟐 [ 𝟐 +x]𝟏 − 𝟏=𝟐 [ 𝟐 +1−𝟐+1]=3sq. units
𝟐
Equating of line segment BC is
𝟐−𝟑
y−3=𝟑−𝟏(x−1)
−𝟏
y= (x−1)+3
𝟐
𝟏
y=𝟐(−x+7)
𝟑 𝟏 𝟏
∴Area(BLMCB)=∫𝟏 (−x+7)dx=𝟐[−x2/2+7x]𝟑𝟏=1/2[−9/2+21+1/2−7]=5 sq. units
𝟐
Equation of line segment AC is
𝟐−𝟎
y−0=𝟑+𝟏(x+1)
y=1/2(x+1)
𝟏 𝟑
∴Area(AMCA)=𝟐 ∫−𝟏 (𝒙 + 𝟏)𝒅𝒙 =1/2[x2//2+x]𝟑 − 𝟏=1/2[9/2+3−1/2+1]=4 sq. units
Therefore, from equation (1), we obtain
Area(ΔABC)=(3+5−4)=4 sq. units

CHAPTER-9: DIFFERENTIAL EQUATIONS


Q No. QUESTION MARKS

133
1 Find the solution of the differential equation- 2
𝑑𝑦
= 𝑥 3 𝑒 −2𝑦
𝑑𝑥
2 Find the integrating factor of the differential equation- 2
𝑑𝑦
𝑥 − 2𝑦 = 2𝑥 2
𝑑𝑥
3 Find the integrating factor of the differential equation- 2

(𝑦 − 𝑥)𝑑𝑦 = (1 + 𝑦 2 )𝑑𝑥
4 Find the sum of the order and degree of the differential equation- 2

𝑑 𝑑𝑦 3
{( ) } = 0
𝑑𝑥 𝑑𝑥

5 Find the solution of the differential equation- 2


𝑑𝑦 1 −𝑐𝑜𝑠 𝑐𝑜𝑠𝑥
=
𝑑𝑥 1 +𝑐𝑜𝑠 𝑐𝑜𝑠𝑥
6 Verify that the given function is a solution of the corresponding differential 2
𝑥𝑦
equation: 𝑦 = √1 + 𝑥 2 ; 𝑦 ′ = 1+𝑥 2

7 Verify that the given function is a solution of the corresponding differential 2


equation: 𝑦 −𝑐𝑜𝑠 𝑐𝑜𝑠𝑦 = 𝑥 ; (𝑦 𝑠𝑖𝑛 𝑠𝑖𝑛𝑦 +𝑐𝑜𝑠 𝑐𝑜𝑠𝑦 + 𝑥)𝑦 ′ = 𝑦

8 Find the solution of the differential equation- 2


(𝑒 𝑥 + 𝑒 −𝑥 )𝑑𝑦 − (𝑒 𝑥 − 𝑒 −𝑥 )𝑑𝑥 = 0

9 Find the integrating factor of the differential equation- 2


𝑑𝑦
𝑥 + 𝑦 − 𝑥 + 𝑥𝑦 𝑐𝑜𝑡 𝑐𝑜𝑡𝑥 = 0(𝑥 ≠ 0)
𝑑𝑥
10 Solve the differential equation: 2
𝑥𝑑𝑦 − (𝑦 + 2𝑥 2 )𝑑𝑥 = 0
11 Solve the following differential equation: 3
𝑑𝑦
+ 𝑦 𝑐𝑜𝑡 𝑐𝑜𝑡𝑥 = 4𝑥 𝑐𝑠𝑐 𝑐𝑠𝑐𝑥
𝑑𝑥
12 Solve the differential equation- 3
𝑑𝑦 𝑑𝑦
𝑦+𝑥 =𝑥−𝑦
𝑑𝑥 𝑑𝑥

134
13 Solve the differential equation- 3
𝑑𝑦
+ 𝑦 𝑠𝑒𝑐 𝑠𝑒𝑐𝑥 =𝑡𝑎𝑛 𝑡𝑎𝑛𝑥
𝑑𝑥
14 Solve the differential equation- 3
𝑑𝑦 𝑥 + 𝑦
=
𝑑𝑥 𝑥 − 𝑦
15 Solve the differential equation- 3
(𝑥 2 − 𝑦 2 )𝑑𝑥 + 2𝑥𝑦𝑑𝑦 = 0 given that 𝑦 = 1 when 𝑥 = 1

16 Find the particular solution of the differential equation satisfying the given 3
condition: 𝑥 2 𝑑𝑦 + (𝑥𝑦 + 𝑦 2 )𝑑𝑥 = 0; 𝑦 = 1 when 𝑥 = 1

17 Solve the following differential equation- 3


𝑑𝑦
(𝑥 2 + 1) + 2𝑥𝑦 = √𝑥 2 + 4
𝑑𝑥
18 Solve the differential equation- 3
𝑑𝑦 𝑦
𝑥 = 𝑦−𝑥 )
𝑑𝑥 𝑥
19 If 𝑦(𝑥) is a solution of the differential equation 3
2+𝑠𝑖𝑛𝑠𝑖𝑛𝑥 𝑑𝑦 𝜋
(
1+𝑦
)
𝑑𝑥
= − 𝑐𝑜𝑠 𝑐𝑜𝑠𝑥 and 𝑦(0) = 1, then find the value of 𝑦( 2 )

20 Find the particular solution, satisfying the given condition for the following 3
differential equation:
𝑑𝑦 𝑦 𝑦
− 𝑥 +𝑐𝑜𝑠 𝑐𝑜𝑠(𝑥 ) = 0, 𝑦 = 0 when 𝑥 = 1
𝑑𝑥

135
CHAPTER-10: VECTORS

1 MARK QUESTIONS

Q. QUESTION MARK
NO
1 Which of the following is not a vector quantity? 1
(i) force (ii) mass
(iii) weight (iv) velocity
2 The magnitude of vector 3 𝑖̂ + 6𝑗̂ + 2𝑘̂ is 1

(i) 5 (ii) 7
(iii) 12 (iv) 11
3 The value of 𝑥, 𝑦 𝑎𝑛𝑑 𝑧 𝑖𝑓 𝑣𝑒𝑐𝑡𝑜𝑟𝑠 𝑥𝑖̂ − 2𝑗̂ + 𝑧𝑘̂ = 3 𝑖̂ − 𝑦𝑗̂ − 2𝑘̂ are 1

(i) 3,2,-2 (ii) 3,2,1


(iii) 3,2,2 (iv) -2,3,2
4 The area of parallelogram with adjacent sides as 𝑎⃗ 𝑎𝑛𝑑 𝑏⃗⃗ 𝑖𝑠 is given by 1
1
(i) ⃗⃗ × ⃗⃗⃗
|⃗𝑎 𝑏| (ii) |𝑎⃗ × 𝑏⃗⃗|
2
1 1
(iii)
4
|𝑎⃗ × 𝑏⃗⃗| (iv)
8
|𝑎⃗ × 𝑏⃗⃗|

5 The value of 𝜇 for which the vectors 3 𝑖̂ − 6𝑗̂ + 𝑘̂ is parallel to 2 𝑖̂ − 4𝑗̂ + 𝜇𝑘̂ 1
is
2 3
(i) (ii)
3 2
5 2
(iii) (iv)
2 5

6 The scalar projection of the vector 3𝑖̂ − 𝑗̂ − 2𝑘̂ on 5𝑖̂ + 12𝑗̂ is 1


7 7
(i) (ii)
√14 14
3 7
(iii) (iv)
13 2

136
7 The vector having initial point and terminal points as (2,5,0)𝑎𝑛𝑑 (−3,7,4) 1
respectively is

(i) −𝑖̂ + 12𝑗̂ + 4𝑘̂ (ii) 5𝑖̂ + 2𝑗̂ − 4𝑘̂

(iii)−5𝑖̂ + 2𝑗̂ + 4𝑘̂ (iv)𝑖̂ + 𝑗̂ + 𝑘̂


8 If 𝑎̂ 𝑖𝑠 𝑢𝑛𝑖𝑡 𝑣𝑒𝑐𝑡𝑜𝑟, 𝑡ℎ𝑒𝑛 |𝑎̂|is 1
(i) 0 (ii) 1
(iii) 2 (iv) None of these
9 If 𝑎⃗ 𝑖𝑠 𝑎 𝑢𝑛𝑖𝑡 𝑣𝑒𝑐𝑡𝑜𝑟 𝑎𝑛𝑑 (𝑥⃗ + 𝑎⃗). (𝑥⃗ − 𝑎⃗) = 24, 𝑡ℎ𝑒𝑛 |𝑥⃗|is 1
(i) 16 (ii) 4
(iii) 2 (iv) 5
10 The projection of vector 𝑎⃗ 𝑜𝑛 𝑏⃗⃗ 𝑖𝑠 𝑔𝑖𝑣𝑒𝑛 𝑏𝑦 1
⃗⃗
𝑎⃗⃗.𝑏 ⃗⃗⃗×𝑏
𝑎 ⃗⃗⃗
(i) (ii) |⃗𝑎
⃗⃗|
|𝑎⃗⃗|

𝑎⃗⃗.𝑏⃗⃗ ⃗⃗⃗
𝑏×𝑎 ⃗⃗⃗
(iii) (iv) |𝑎|

⃗⃗
|𝑏⃗⃗|

11 The scalar product of vector 𝑎⃗ 𝑎𝑛𝑑 𝑏⃗⃗, 𝑤ℎ𝑒𝑟𝑒 𝜃 𝑖𝑠 𝑎𝑛𝑔𝑙𝑒 between 𝑎⃗ 𝑎𝑛𝑑 𝑏⃗⃗ is 1
given by

(i) 𝑎⃗. 𝑏⃗⃗ = |𝑎⃗||𝑏⃗⃗|𝑐𝑜𝑠𝜃 (ii)𝑎⃗. 𝑏⃗⃗ = |𝑎⃗||𝑏⃗⃗|𝑠𝑖𝑛𝜃

(ii) 𝑎⃗. 𝑏⃗⃗ = |𝑎⃗||𝑏⃗⃗| (iv) 𝑎⃗. 𝑏⃗⃗ = |𝑎⃗||𝑏⃗⃗|𝑡𝑎𝑛𝜃


12 Two vectors 𝑎⃗ 𝑎𝑛𝑑 𝑏⃗⃗ are perpendicular to each other then 1

(i) 𝑎⃗. 𝑏⃗⃗ = 1 (ii) 𝑎⃗. 𝑏⃗⃗ = 0

(iii) 𝑎⃗ × 𝑏⃗⃗ = 0 (iv) None of these

13 The value of 𝜆 for which the vector 2 𝑖̂ + 𝜆𝑗̂ + 𝑘̂ and 𝑖̂ + 2𝑗̂ + 3𝑘̂ are 1
perpendicular to each other is
(i) 0 (ii) 1
3 5
(iii)
2
(iv) −
2

137
14 The vector of 5 magnitude in the direction of 𝑖̂ + 𝑗̂ + 𝑘̂ is 1

(i)
̂
𝑖̂+𝑗̂ +𝑘 (ii) 5(𝑖̂ + 𝑗̂ + 𝑘̂)
3
̂
𝑖̂+𝑗̂ +𝑘 ̂
𝑖̂+𝑗̂ +𝑘
(iii) (iv) 5( )
√3 √3

15 If 𝑎⃗ = 2𝑖̂ − 𝑗̂ + 𝑘̂,𝑏⃗⃗ = 3𝑖̂ + 2𝑗̂ − 3𝑘̂ and 𝑐⃗⃗⃗ = 𝑖̂ + 2𝑗̂ + 𝑘̂then 2𝑎⃗ + 𝑏⃗⃗ − 𝑐⃗ = 1

(i) 6𝑖̂ − 2𝑗̂ − 2𝑘̂ (ii) 6𝑖̂ + 2𝑗̂ − 2𝑘̂


(iii) 6𝑖̂ − 2𝑗̂ + 2𝑘̂ (iv) 6𝑖̂ + 2𝑗̂ + 2𝑘̂
16 𝑂𝐴 = 𝑖̂ + 2𝑗̂ − 2𝑘̂,𝑂𝐵
If ⃗⃗⃗⃗⃗⃗ ⃗⃗⃗⃗⃗⃗ = 6𝑖̂ − 𝑗̂ + 4𝑘̂ then vector ⃗⃗⃗⃗⃗⃗
𝐴𝐵𝑖𝑠 1

(i) −5𝑖̂ + 3𝑗̂ − 6𝑘̂ (ii) 5𝑖̂ − 3𝑗̂ + 6𝑘̂


(iii) −5𝑖̂ − 3𝑗̂ − 6𝑘̂ (iv) −5𝑖̂ + 3𝑗̂ + 6𝑘̂
17 The value of (𝑖̂. 𝑖̂) + (𝑗̂ × 𝑗̂) + (𝑘̂. 𝑘̂) is 1

(i) 0 (ii) 1
(iii) 2 (iv) 2+𝑘̂
18 If 𝜃 is the angle between vectors 𝑎⃗ 𝑎𝑛𝑑 𝑏⃗⃗ and |𝑎⃗. 𝑏⃗⃗| = |𝑎⃗ × 𝑏⃗⃗|, then the 1
value of 𝜃 is
(i) 0 (ii) 𝜋
𝜋 𝜋
(iii) (iv)
4 2

19 The magnitude of a vector can never be 1


(i) negative (ii) positive
(iii) zero (iv) none of these
20 The position vector of the point which divides the line joining the points with 1
position vectors 2𝑎⃗ − 3𝑏⃗⃗ 𝑎𝑛𝑑 𝑎⃗ + 𝑏⃗⃗ in the ratio 3:1 is
⃗⃗
3𝑎⃗⃗−2𝑏 ⃗⃗
7𝑎⃗⃗−8𝑏
(i) (ii)
2 4
3𝑎⃗⃗ 5𝑎⃗⃗
(iii) (iv)
4 4

2MARKS QUESTIONS

Q.NO. QUESTIONS MARKS


1 Find the unit vector in the direction of ⃗⃗⃗⃗⃗⃗
𝑃𝑄 ,where P and Q are the points (1,2,3) and 2
138
(4,5,6) respectively.
2 Find the value of p for which 𝑎⃗ = 3 ̂𝑖 +2𝑗̂ +9𝑘̂ and 𝑏⃗⃗ = ̂𝑖 +p𝑗̂ + 3𝑘̂ are parallel vectors. 2
3 If |𝑎⃗| = √3 , |𝑏⃗⃗| = 2 and 𝑎⃗ . 𝑏⃗⃗ = 3 , find the angle between 𝑎⃗ and 𝑏⃗⃗ 2
4 Find the projection of 𝑎⃗ on 𝑏⃗⃗ if 𝑎⃗ . 𝑏⃗⃗ = 8 and 𝑏⃗⃗ =2 ̂𝑖 +6𝑗̂ + 3𝑘̂ 2
5 If 𝑎⃗ . 𝑎⃗ = 0 and 𝑎⃗ . 𝑏⃗⃗ = 0 ,what can you conclude about the vector 𝑏⃗⃗ ? 2
6 Find the value of x for which x ( ̂𝑖 +𝑗̂ +𝑘̂ ) is a unit vector. 2
𝜋 𝜋
7 If a unit vector 𝑎⃗ makes angles with 𝑖̂ , with 𝑗̂ and an acute angle 𝜃 with 𝑘̂ , then 2
3 4
find 𝜃 .
8 Find the direction cosines of the vector ̂𝑖 +2𝑗̂ + 3𝑘̂ . 2
9 Find the area of the parallelogram whose diagonals are 4 ̂𝑖 -𝑗̂ - 3𝑘̂ and −2 ̂𝑖 + 𝑗̂ - 2𝑘̂ 2
10 If two vectors 𝑎⃗ and 𝑏⃗⃗ are such that |𝑎⃗| =2, |𝑏⃗⃗| = 1 and 𝑎⃗ . 𝑏⃗⃗ = 1, then find the value of 2
𝑎 - 5 𝑏⃗⃗) . (2⃗⃗⃗⃗
(3⃗⃗⃗⃗ 𝑎 + 7 𝑏⃗⃗) .

3 MARKS QUESTION

Q.NO. QUESTIONS MARKS


1 Find the value of 𝛾 if (2 ̂𝑖 +6𝑗̂ +14𝑘̂) x ( ̂𝑖 - 𝛾𝑗̂ +7𝑘̂) = ⃗⃗ 0 3
2 If 𝑎⃗ = 2 ̂𝑖 +2𝑗̂ +3𝑘̂, 𝑏⃗⃗ = − ̂𝑖 +2𝑗̂ +𝑘̂ and 𝑐⃗ = 3 ̂𝑖 +𝑗̂ are such that 𝑎⃗ +𝛽⃗𝑏⃗ is 3
perpendicular to 𝑐⃗ ,then find 𝛽
3 Find |𝑥⃗| , if for a unit vector 𝑎⃗ , (𝑥⃗ - 𝑎⃗ )(𝑥⃗ + 𝑎⃗) =20. 3
4 If the points (-1 , -1, 2), (2 , m, 5) , and ( 3, 11, 6) are collinear, find the value of m. 3
5 ⃗⃗ ⃗⃗ ⃗⃗ ⃗
⃗ ⃗

If 𝑎⃗ , 𝑏, 𝑐⃗ are unit vectors such that 𝑎⃗ + 𝑏 + 𝑐⃗ = 0 , find the value of 𝑎⃗ . 𝑏+ 𝑏 . 𝑐⃗ + 𝑐⃗ . 𝑎⃗ 3
6 Let 𝑎⃗ = ̂𝑖 +4𝑗̂ +2𝑘̂, 𝑏⃗⃗ = 3 ̂𝑖 -2𝑗̂ +7𝑘̂ and 𝑐⃗ = 2 ̂𝑖 -𝑗̂ + 4𝑘̂ . Find a vector 𝑑⃗ which is 3
perpendicular to both 𝑎⃗ and 𝑏⃗⃗ , and 𝑐⃗ .𝑑⃗ =15
7 If |𝑎⃗| =2, |𝑏⃗⃗| =5 and |𝑎⃗ x 𝑏⃗⃗ | = 8 , find 𝑎⃗ . 𝑏⃗⃗ 3
8 Find a vector of magnitude 9 , which is perpendicular to both the vectors 4 ̂𝑖 -𝑗̂ + 3𝑘̂ and 3
−2 ̂𝑖 + 𝑗̂ - 2𝑘 ̂
9 Find the values of x and y if the vectors 𝑎⃗ = 3 ̂𝑖 + x𝑗̂ - 𝑘̂ and 𝑏⃗⃗ = 2 ̂𝑖 +𝑗̂ + y 𝑘̂ are 3
mutually perpendicular vectors of equal magnitude.
10 Show that the vectors 2 ̂𝑖 -𝑗̂ + 𝑘̂ , 𝑖̂ - 3𝑗̂ -5 𝑘̂ and 3 ̂𝑖 - 4 𝑗̂ - 4𝑘̂ form the sides of a right 3
angled triangle.

4 MARKS QUESTION

Q.NO. QUESTIONS MARKS


1 Ginni purchased an air plant holder which is in the shape of a tetrahedron. 4
Let A, B, C and D are the coordinates of the air plant holder where
A=(1,1,1), B=(2,1,3), C=(3,2,2) and D=(3,3,4).

139
Based on the above information, answer the following questions.

(i) Find the vector ⃗⃗⃗⃗⃗⃗


𝐴𝐵.

(ii) ⃗⃗⃗⃗⃗⃗⃗
Find the vector 𝐴𝐶.
(iii) Find the area of ∆ABC.
OR,

Find the unit vector along ⃗⃗⃗⃗⃗⃗


𝐴𝐷
2 A plane started from airport situated at O with a velocity of 1200 km/h 4
towards east. Air is blowing at a velocity of 50 km/h towards north as
shown in the figure.
As a result, for 2 hour, the plane travelled with the resultant velocity in
direction OP as shown in the figure. Then from P to R plane travelled 1 hour
keeping velocity of 1200 km/h and finally landed at R.

Based on the above information, answer the following questions (Take O as


the origin, 𝑖̂along east and 𝑗̂along north).

(i) ⃗⃗⃗⃗⃗⃗ and 𝑃𝑅


Find the velocity vector along direction 𝑂𝑃 ⃗⃗⃗⃗⃗⃗.

(ii) Find the position vector of point P.

(iii) ⃗⃗⃗⃗⃗⃗.
Find the unit vector along 𝑂𝑅
OR
140
Find the net direction of travel of plane from O to R with east.
3 A pyramid shaped greenhouse is to be 4
constructed in the form of a pyramid ABCD
as shown in the figure.

Let its angular points are A(0,1,2), B(3,0,1), C(4,3,6) and D(2,3,2) and G be
the point of intersection of the medians of ∆BCD.
Based on the above information, answer the following questions.
(i) Find the position vector of point G.

(ii) ⃗⃗⃗⃗⃗⃗ .
Find the unit vector along 𝐴𝐺

(iii) ⃗⃗⃗⃗⃗⃗ and ⃗⃗⃗⃗⃗⃗


Find the magnitude of the sum of 𝐴𝐶 𝐶𝐵.
OR
Find |⃗⃗⃗⃗⃗⃗⃗⃗
𝐴𝐵 × ⃗⃗⃗⃗⃗⃗⃗⃗
𝐴𝐶|.
4 Solar panels are to be installed on a slanting roof. A surveyor determines 4
the coordinates of the four corners of the roof where solar panels are
mounted. Suppose the points are labeled as P(6,8,4), Q(21,8,4), R(21,16,10)
and S(6,16,10).

Based on the
above information, answer the following questions.

(i) ⃗⃗⃗⃗⃗⃗ and ⃗⃗⃗⃗⃗


Find the scalar components of vectors 𝑃𝑄 𝑆𝑃.
(ii) Find a unit vector perpendicular to the surface of the roof.

5 A girl walks 3 km towards west to reach point A and then walks 5 km in a 4


direction 30° east of north and stops at point B. Let the girl starts from O
141
(origin) and take 𝑖̂along east
and𝑗̂ along north.

Based on the above information,


answer the following questions.

(i) ⃗⃗⃗⃗⃗⃗ .
Find the scalar components of 𝐴𝐵

(ii) Find the unit vector along ⃗⃗⃗⃗⃗⃗


𝐴𝐵.
(iii) Find the position vector of point B.

ANSWERS:
Q. NO ANSWER MARKS
1 (ii) mass 1
2 (ii) 7 1
3 (i)3,2,-2 1
4 ⃗⃗⃗ × ⃗⃗⃗
(i)|𝑎 𝑏| 1
3 −6 1 2
5 (i)∵ 𝑎⃗ ∥ 𝑏⃗⃗ ∴ = = ⟹𝜇= 1
2 −4 𝜇 3

6 ̂ ).(5𝑖̂+12𝑗̂ )
(3𝑖̂−𝑗̂ −2𝑘 3 1
(iii) |5𝑖̂+12𝑗̂ |
=
13

7 (iii)Required vector=(−3 − 2)𝑖̂ + (7 − 5)𝑗̂ + (4 − 0)𝑘̂ =−5𝑖̂ + 2𝑗̂ + 4𝑘̂ 1

8 (ii)1 1
9 (iv)5 ,|𝑥⃗|2 − |𝑎⃗|2 = 24, |𝑥⃗|2 = 25 1
10 𝑎⃗⃗.𝑏⃗⃗ 1
(iii) ⃗⃗|
|𝑏

11 (i)𝑎⃗. 𝑏⃗⃗ = |𝑎⃗||𝑏⃗⃗|𝑐𝑜𝑠𝜃 1

12 (ii)𝑎⃗. 𝑏⃗⃗ = 0 1

142
5
13 (iv)𝜆 = − , Use 𝑎⃗. 𝑏⃗⃗ = 0 1
2

14 ̂
𝑖̂+𝑗̂ +𝑘 1
(iv)5 ( )
√3

15 (i)6𝑖̂ − 2𝑗̂ − 2𝑘̂ 1

16 (ii)5𝑖̂ − 3𝑗̂ + 6𝑘̂, 𝐴𝐵


⃗⃗⃗⃗⃗⃗ = 𝑂𝐵
⃗⃗⃗⃗⃗⃗ − 𝑂𝐴
⃗⃗⃗⃗⃗⃗ 1

17 (iii)2 , 1
𝜋 𝜋
18 (iii) , 𝑎𝑏 𝑐𝑜𝑠𝜃 = 𝑎𝑏 𝑠𝑖𝑛𝜃 ⇒ 𝑡𝑎𝑛𝜃 = 1 ∴ 𝜃 = 1
4 4

19 (i)negative 1
20 5𝑎⃗⃗ ⃗⃗)+1(2𝑎⃗⃗−3𝑏
3(𝑎⃗⃗+𝑏 ⃗⃗) 1
(iv) , Required position vector =
4 3+1

2 MARKS QUESTION

Q.NO. QUESTIONS MARKS


1 ⃗⃗⃗⃗⃗⃗ = (4 ̂𝑖 +5 ̂𝑗 + 6𝑘̂) - ( ̂𝑖 +2𝑗̂ +3𝑘̂) = 3 ̂𝑖 +3𝑗̂ +3𝑘̂
𝑃𝑄 2

⃗⃗⃗⃗⃗⃗ | = 3√3
|𝑃𝑄
1
𝑃𝑄 is = ( ̂𝑖 +𝑗̂ +𝑘̂)
The unit vector in the direction of ⃗⃗⃗⃗⃗⃗
√3
2 1 2
3
3 𝜋 2
6

4 8 2
7
5 𝑏⃗⃗ is any vector 2
6 1 2
±
√3
7 𝜋 2
3
1 2 3
8 , , 2
√14 √14 √14
9 15 2
sq. units
2

143
10 0 2

3 MARKS QUESTION

Q.NO. QUESTIONS MARKS


1 0 3
2 -3 3
3 8 3
4 √21 3
5 8 3
6 −3 3
2
1
7 (160𝑖̂-5𝑗̂ +70𝑘̂) 3
3

8 6 3

9 −3𝑖̂+6𝑗̂ +6𝑘̂ 3

10 −31 41 3
x= , y = 12
12

ANSWERS OF 4 MARKS QUESTIONS

Q.NO. ANSWERS MARKS


1 (i) ⃗⃗⃗⃗⃗⃗ = (2 − 1)𝑖̂ + (1 − 1)𝑗̂ + (3 − 1)𝑘̂ = 𝑖̂ + 2𝑘̂.
𝐴𝐵 1

(ii) ⃗⃗⃗⃗⃗⃗ = (3 − 1)𝑖̂ + (2 − 1)𝑗̂ + (2 − 1)𝑘̂ = 2𝑖̂ + 𝑗̂ + 𝑘̂.


𝐴𝐶 1

1
(iii) ⃗⃗⃗⃗⃗⃗ × 𝐴𝐶
Ar(∆𝐴𝐵𝐶) = |𝐴𝐵 ⃗⃗⃗⃗⃗⃗ |
2

⃗⃗⃗⃗⃗⃗
𝐴𝐵 × 𝐴𝐶⃗⃗⃗⃗⃗⃗ = |𝑖̂𝑗̂𝑘̂102211| = −2𝑖̂ + 3𝑗̂ + 𝑘̂
2
|⃗⃗⃗⃗⃗⃗⃗⃗
𝐴𝐵 × ⃗⃗⃗⃗⃗⃗⃗⃗
𝐴𝐶| = √14
1
Ar(∆𝐴𝐵𝐶) = √14sq. units.
2

OR
⃗⃗⃗⃗⃗⃗⃗
𝐴𝐷 ̂
2𝑖̂+2𝑗̂ +3𝑘
⃗⃗⃗⃗⃗⃗ =
Unit vector along 𝐴𝐷 =
⃗⃗⃗⃗⃗⃗⃗|
|𝐴𝐷 √17
2 (i) ⃗⃗⃗⃗⃗⃗ = (1200𝑖̂ + 50𝑗̂) km/h.
Velocity vector along 𝑂𝑃 1

⃗⃗⃗⃗⃗⃗ = (1200 + 50)𝑗̂ = (1250𝑗̂) km/h.


Velocity vector along 𝑃𝑅 1

144
(ii) ⃗⃗⃗⃗⃗⃗ = (1200𝑖̂ + 50𝑗̂) × 2 = 2400𝑖̂ + 100𝑗̂.
𝑂𝑃

(iii) ⃗⃗⃗⃗⃗⃗ = 1250𝑗̂ × 1 = 1250𝑗̂


𝑃𝑅 2

⃗⃗⃗⃗⃗⃗ = 𝑂𝑃
𝑂𝑅 ⃗⃗⃗⃗⃗⃗ + 𝑃𝑅
⃗⃗⃗⃗⃗⃗ = (2400𝑖̂ + 100𝑗̂) + (1250𝑗̂) = 2400𝑖̂ + 1350𝑗̂
⃗⃗⃗⃗⃗⃗⃗
𝑂𝑅 (2400𝑖̂+1350𝑗̂ ) (2400𝑖̂+1350𝑗̂ )
⃗⃗⃗⃗⃗⃗ =
Unit vector along 𝑂𝑅 = = =
⃗⃗⃗⃗⃗⃗⃗ |
|𝑂𝑅 √7582500 150√337
16 90
𝑖̂ + 𝑗̂.
√337 √337

OR

(iv) ⃗⃗⃗⃗⃗⃗ = 1250𝑗̂ × 1 = 1250𝑗̂


𝑃𝑅
⃗⃗⃗⃗⃗⃗ = 𝑂𝑃
𝑂𝑅 ⃗⃗⃗⃗⃗⃗ + 𝑃𝑅
⃗⃗⃗⃗⃗⃗ = (2400𝑖̂ + 100𝑗̂) + (1250𝑗̂) = 2400𝑖̂ + 1350𝑗̂

So, net direction of travel of plane from O to R with east = (1350


2400
)=
45
(8)

3 (3+4+2) (0+3+3) 1+6+2 1


(i) Position vector of point G= 𝑖̂ + 𝑗̂ + 𝑘̂ =
3 3 3
1
3𝑖̂ + 2𝑗̂ + 3𝑘̂.

(ii) ⃗⃗⃗⃗⃗⃗ = (3 − 0)𝑖̂ + (2 − 1)𝑗̂ + (3 − 2)𝑘̂ = 3𝑖̂ + 𝑗̂ + 𝑘̂


𝐴𝐺
⃗⃗⃗⃗⃗⃗
𝐴𝐺 ̂
3𝑖̂+𝑗̂ +𝑘
⃗⃗⃗⃗⃗⃗ =
Unit vector along 𝐴𝐺 = .
⃗⃗⃗⃗⃗⃗ |
|𝐴𝐺 √11 2

(iii) ⃗⃗⃗⃗⃗⃗ + 𝐶𝐵
𝐴𝐶 ⃗⃗⃗⃗⃗⃗ = (3 − 0)𝑖̂ + (0 − 1)𝑗̂ + (1 − 2)𝑘̂ = 3𝑖̂ − 𝑗̂ − 𝑘̂
⃗⃗⃗⃗⃗⃗ = 𝐴𝐵

⃗⃗⃗⃗⃗⃗ + 𝐶𝐵
|𝐴𝐶 ⃗⃗⃗⃗⃗⃗ | = √11.

𝑂𝑅

𝐴𝐵 = 3𝑖̂ − 𝑗̂ − 𝑘̂
⃗⃗⃗⃗⃗⃗

⃗⃗⃗⃗⃗⃗ = 4𝑖̂ + 2𝑗̂ + 4𝑘̂


𝐴𝐶
⃗⃗⃗⃗⃗⃗ = |𝑖̂𝑗̂𝑘̂3 − 1 − 1424| = −2𝑖̂ − 16𝑗̂ + 10𝑘̂
⃗⃗⃗⃗⃗⃗ × 𝐴𝐶
𝐴𝐵

|⃗⃗⃗⃗⃗⃗⃗⃗
𝐴𝐵 × ⃗⃗⃗⃗⃗⃗⃗⃗
𝐴𝐶| = √4 + 256 + 100 = √360 = 6√10.
4 (i) ⃗⃗⃗⃗⃗⃗ = (21 − 6)𝑖̂ + (8 − 8)𝑗̂ + (4 − 4)𝑘̂ = 15𝑖̂
𝑃𝑄 2

⃗⃗⃗⃗⃗⃗ are 15, 0, 0.


Scalar components of 𝑃𝑄

𝑆𝑃 = (6 − 6)𝑖̂ + (16 − 8)𝑗̂ + (10 − 4)𝑘̂ = 8𝑗̂ + 6𝑘̂


⃗⃗⃗⃗⃗

Scalar components of ⃗⃗⃗⃗⃗


𝑆𝑃 are 0, 8, 6.

(ii) ⃗⃗⃗⃗⃗⃗ and 𝑆𝑃


Vector perpendicular to roof will be perpendicular to 𝑃𝑄 ⃗⃗⃗⃗⃗. 2

145
𝑆𝑃 = |𝑖̂𝑗̂𝑘̂ 1500086| = −90𝑗̂ + 120𝑘̂
⃗⃗⃗⃗⃗⃗ × ⃗⃗⃗⃗⃗
So, required vector=𝑃𝑄
3 4
So unit vector is − 𝑗̂ + 𝑘̂.
5 5

5 (i) Projection of ⃗⃗⃗⃗⃗⃗ ⃗⃗⃗⃗⃗⃗ | 𝑐𝑜𝑠 𝑐𝑜𝑠60° = 5 × 1 = 2.5


𝐴𝐵 along 𝑖̂=|𝐴𝐵 2
2

Projection of ⃗⃗⃗⃗⃗⃗ ⃗⃗⃗⃗⃗⃗ | 𝑠𝑖𝑛 𝑠𝑖𝑛60° = 5 × √3 = 2.5√3


𝐴𝐵 along 𝑗̂ = |𝐴𝐵
2

So, scalar components of ⃗⃗⃗⃗⃗⃗


𝐴𝐵 are 2.5, 2.5√3.
5
⃗⃗⃗⃗⃗⃗
𝐴𝐵 (𝑖̂+√3𝑗̂ ) 1 2
(ii) Unit vector along ⃗⃗⃗⃗⃗⃗
𝐴𝐵 = ⃗⃗⃗⃗⃗⃗
= 2
= (𝑖̂ + √3𝑗̂).
|𝐴𝐵 | 25 75 2
√ +
4 4

5 5 √3 1 5 √3
⃗⃗⃗⃗⃗⃗ = ⃗⃗⃗⃗⃗⃗
𝑂𝐵 𝑂𝐴 + ⃗⃗⃗⃗⃗⃗
𝐴𝐵 = −3𝑖̂ + ( 𝑖̂ + 𝑗̂) = − 𝑖̂ + 𝑗̂.
2 2 2 2

CHAPTER-11

THREE DIMENSIONAL GEOMETRY

SECTION A

Q. NO QUESTION MARK

146
1 The vector equation of equation of straight line 1
𝑥−5 𝑦+4 𝑧−6
= = is
3 7 2

(a) 𝑟⃗ = (3𝑖 + 7𝑗 + 2𝑘) + 𝜇(5𝑖 + 4𝑗 − 6𝑘)

(b)𝑟⃗ = (5𝑖 + 4𝑗 − 6𝑘)+𝜇(3𝑖 + 7𝑗 + 2𝑘)

(c) 𝑟⃗ = (5𝑖 − 4j − 6𝑘)+𝜇(3𝑖 − 7𝑗 − 2𝑘)

(d)𝑟⃗ = (5𝑖 − 4𝑗 + 6𝑘)+𝜇(3𝑖 + 7𝑗 + 2𝑘)

2 The angle between the lines 2x = 3y = -z and 6x = -y = -4z is 1

(a)00 (b)300 (c) 450 d)900

3 Find the direction cosines of the line passing through the points (2,1,-2) and 1
(1,2,1)
−1 1 −1 1 −1 2 −1 1 3
(a) ( , , ) (b) ( , , ) (c) ( , , )
√11 √11 √11 √11 √111 √11 √11 √11 √11
−1 3 −1
(d) ( , , )
√11 √11 √11

4 If a line makes A, B, C with axes respectively , then the value of cos2A +cos2B + 1
cos2C

(a) -2 (b) -1 (c) 1 (d) 2

5 Write the Cartesian equation of the following line given in the vector form1

𝑟⃗ =2𝑖̂ +𝑗̂ -4𝑘̂ +⋋ (𝑖̂ -𝑗̂ -𝑘̂ )


𝑥−2 𝑦−1 𝑧+4 𝑥+2 𝑦−1 𝑧+4 𝑥−2 𝑦−1 𝑧−4 𝑥−2
(𝑎 ) = = (b) = = (c) = = (d) =
1 −1 −1 1 1 −1 −1 −1 1 −1
𝑦−1 𝑧+4
=
1 −1

6 Write the direction ratio of a line parallel to z axis. 1

(a) (1,1,0) (b) (0,1,0) (c) (0,0,1) (d) (1,0,1)

147
7 −𝑥−3 2𝑦+2 𝑧−5
If the equation of a line AB is = = find the direction ratio of 1
1 −2 4
line parallel to AB

(a) (1,-2,4) (b) (1,-1,4) (c) (-1,-2,4) (d) (-1,-1,4)

If a line makes an angle 300 , 450 , 600 with positive direction of x ,y, z axis 1
8 respectively, then its direction cosines are

1 1 √3 √3 1 1 1 √3 1 √3 1 1
(a) ( , , ) (b) ( , , ) (c) ( , , ) (d) ( , , )
2 √2 2 2 √2 2 2 2 √2 2 2 √2

9 Find the direction ratios of a line passing through the points (– 1, 0, 2) and 1
(3, 4, 6).

(a) (4,4,4) (b) (2,4,4) (c) ( -4, -4, -4) (d) (2,-4,4)

10 Equation of a line passing through the point(1,1,1) and parallel to z-axis is 1

𝑥 𝑦 𝑧 𝑥−1 𝑦−1 𝑧−1 𝑥 𝑦 𝑧−1 𝑥−1 𝑦−1 𝑧−1


(𝑎 ) = = (b) = = (c) = = (d) = =
1 1 1 1 1 1 0 0 1 0 0 1

11 The two lines x=ay+b ,z= cy+d and x= ey+f ,z= gy+h will be perpendicular, if and only if 1

(a) ae + cg+1=0 (b) ae + bf+cg+1=0 (c) ae + bf+ cg =0 (d)( a+e)(b+f) + (c+g) =0

12 Direction ratio of 2x-1=y=3z-2 is 1


1 1 1 1 1 1 1 1
(a) ( , , ) (b) ( , 1, ) (c) ( , 1, −2) (d) ( , 0, )
2 2 3 2 3 2 2 3

13 Find the equation of a line parallel to the vector 3iˆ − ˆj − 3kˆ and passing through the 1
point ( -1,1,1).

𝑥+3 𝑦+1 𝑧+3 𝑥−3 𝑦+1 𝑧+3 𝑥+1 𝑦−1 𝑧−1


(𝑎 ) = = (b) = = (c) = =
−1 1 1 −1 1 1 1 −1 −3
𝑥+1 𝑦−1 𝑧−1
(d) = =
3 −1 −3

148
14 8. Find the equation of a line passing through (-4,2,-3) and is parallel to the line 1
2x − 1 4 − y z + 1
= =
2 7 2
𝑥+4 𝑦−2 𝑧+3 𝑥+4 𝑦−2 𝑧+3 𝑥−4 𝑦−2 𝑧+3 𝑥−4
(𝑎 ) = = (b) = = (c) = = (d) =
1 −7 2 1 7 2 2 1 2 1
𝑦+2 𝑧−3
=
−7 2

15 The co-ordinate of the foot of the perpendicular drawn from the point (2,-3,4) on the y- 1
axis is

(a) (2,3,4) (b) (-2,-3,-4) (c) (0,-3,0) (d) (2,0,4)

16 Distance of the point (a,b,c) from y-axis 1

(a) b (b) |𝑏| (c) |𝑏| + |𝑐| (d) √𝑎2 + 𝑐 2

17 The equation of x-axis in space are 1

(a) x=0, y=0 (b) x=0 , z=0 (c) x=0 (d) y=0,z=0

18 A line makes equal angles with co-ordinate axis . Direction cosine of this line are 1
1 1 1 1 1 1 1 −1 −1
(𝑎) ± (1,1,1) (b) ±( , , )(c) ±(3 , 3 , 3) (d) ±( , , )
√3 √3 √3 √3 √3 √3

𝑥+1 𝒚+𝟐 𝒛+𝟑


19 If the point P (a,b,0 ) lies on the line = = , then (a,b) is 1
2 𝟑 𝟒

1 2 1 1
(a) (1,2) (b) (2 , 3 ) (c) (2 , 4 ) (d) (0 ,0 )

𝜋 𝜋
20 If a line makes angle and with x-axis and y-axis respectively , then the angle 1
3 4
made by the line with z-axis is
𝜋 𝜋 𝜋 𝜋
(a) 2 (b) 3 (c) 4 (d) 6

SECTION B

149
Q. NO QUESTION MARK
𝒙 𝒚−𝟏 𝒛+𝟏
1 Find the coordinate of the point on the line 𝟏 = = which are at a distance of 2
𝟐 𝟐
√𝟏𝟏𝒖𝒏𝒊𝒕𝒔 𝒇𝒓𝒐𝒎 𝒕𝒉𝒆 𝒐𝒓𝒊𝒈𝒊𝒏.

2 Check whether the lines given by the equations x=2k+2, y=7k+1 z=-3k+3 and x=-q-2 2
,y=2q+8 z= 4q+5 are perpendicular or not

𝒙−𝟏 𝒚−𝟒 𝒛−𝟑 𝒙−𝟐 𝒚−𝟓 𝟏−𝒛


3 Find the value of p so that the lines line = = and 𝟒𝒑 = 𝟐 = are 2
−𝟐 𝟑𝒑 𝟒 𝟕
perpendicular to each other.

4 Find direction cosines and direction ratio of a line parallel to the lines whose 2
equations are 6x-12=3y+4= 2z-2

5 Find the vector equation of a line which passes through the point (-2,4,-5) and are 2
𝒙+𝟑 𝒚−𝟒 𝟖−𝒛
parallel to the line given by = =
𝟑 𝟓 −𝟔

SECTION C

Q. NO QUESTION MARK
𝒙−𝟒 𝒚+𝟑 𝒛+𝟏 𝒙−𝟏 𝒚+𝟏 𝒛+𝟏𝟎
1 Find the acute angle between the two lines by = = by = −𝟑 = 3
𝟑 𝟒 𝟓 𝟒 𝟓

2 ̂ ) + 𝒔 (2𝒊̂-𝒋̂ + 𝒌
⃗⃗ = (𝒊̂ +𝒋̂ − 𝒌
Find the shortest distance between the lines 𝒓 ̂) 3
̂ ) + 𝒕 (4𝒊̂ +2𝒋̂ + 𝟐𝒌
⃗⃗ = (𝒊̂ +𝒋̂ + 𝟐𝒌
and 𝒓 ̂)

3 Find the coordinate of the foot of the perpendicular from the point P(0,2,3) to the line 3
𝒙+𝟑 𝒚−𝟏 𝒛+𝟒
= =
𝟓 𝟐 𝟑

4 ̂ ) + 𝒔(𝒊̂
⃗⃗ = (𝒋̂ + 𝟐𝒌
Find the coordinate of the image of the pointQ (1,6,3) w.r.t the line𝒓 3
̂)
+2𝒋̂ + 𝟑𝒌

150
𝒙−𝟏 𝒚−𝟐 𝒛−𝟑 𝒙−𝟐 𝒚−𝟒 𝒛−𝟓
5 Find the shortest distance between the lines = 𝟑 = and = 𝟒 = 3
𝟐 𝟒 𝟑 𝟓

6 Find the direction cosines of the line passing through the two points (– 3
2, 4, – 5) and (1, 2, 3).

7 Show that the points A (2, 3, – 4), B (1, – 2, 3) and C (3, 8, – 11) are 3
collinear

8 If a line makes angles 90°, 135°, 45° with the x, y and z-axes 3
respectively, find its direction cosines

9 Show that the lines (x – 5)/7 = (y + 2)/-5 = z/1 and x/1 = y/2 = z/3 are 3
perpendicular to each other

10 What are the direction cosines of a line which makes equal angles with the 3
coordinate axes?

SECTION D

Q. NO QUESTION MARK

1 Find the vector and cartesian equations of the line through the point (1, 2, – 4) 5
and perpendicular to the two lines r⃗ = (8î – 19ĵ + 10k̂) + λ(3î – 16ĵ + 7k̂) and
r⃗ = (15î + 29ĵ + 5k̂) + µ (3î + 8ĵ – 5k̂).

2 Find the coordinates of the foot of perpendicular drawn from the point A (- 1, 8, 5
4) to the line joining the points B(0, – 1, 3) and C(2, – 3, – 1). Hence, find the
image of the point A in the line.

3 Prove that the line through A (0, – 1, – 1) and B(4, 5, 1) intersects the line 5
through C (3, 9, 4) and D (- 4, 4, 4).

4 Show that the linesr⃗ = (î + ĵ – k̂) + λ(3î – ĵ)andr⃗ = (4î – k̂) + µ(2î + 3k̂) intersect. Also, 5
find their point of intersection

5 The cartesian equation of a line is 6x – 2 = 3y + 1 = 2z – 2. Find the direction 5


cosines of the line. Write down the cartesian and vector equations of a line
passing through (2, – 1, – 1) which are parallel to the given line.

151
SECTION E

Q. QUESTION MARK
NO
1 The equation of motion of a rocket are x = 4t , y = -4t , z = t, where the time t is 4
given in the seconds and the distance is measured in kilometers.

(i)Find the points lie on the path of the rocket at t = 5 s.


(ii) Find the distance of the rocket from the starting point (0,0,0) in 5 seconds
2 4

Read the following text and answer the question on the basis of the same.
A cycle race was organized in a town , where the maximum speed limit was set by
the organizers . No participant are allowed to cross the specified speed limit, but
two cycles A and B are running at the speed more than allowed speed on the road
along the lines
𝑟⃗ = 𝑖̂ + 𝑗̂ – 𝑘̂ + 𝜆 (𝑖̂ +2 𝑗̂ – 2 𝑘̂ )
and𝑟⃗ = 𝑖̂ +2 𝑗̂ + 2k + 𝜇 (2𝑖̂ + 𝑗̂ + 𝑘̂)
(i) Write the equations in vector form.
(ii) Find the angle between two lines.

3 4

The equation of motion of an airplane are x= 4t, y = -4t , z = - 2t, where the time t
is given in minutes and the co-ordinates of a moving point in km. What is the path
of the airplane ? At what distances will the rocket be from the starting point
O(0,0,0) and from the following line in 10 minutes ?
152
𝑟⃗ =40𝑖̂ -10 𝑗̂ -20 𝑘̂ + 𝜆 (10𝑖̂ -20 𝑗̂ +10 𝑘̂ )

4 4

A snake is crawling along the line 𝑟⃗ =3𝑖̂ +2 𝑗̂ +3 𝑘̂ + 𝜆 (𝑖̂ -2 𝑗̂ +2 𝑘̂ ) and another


snake is crawling along the line 𝑟⃗ = -4𝑖̂ -2 𝑘̂ + 𝜇 (3𝑖̂ -2 𝑗̂ -2 𝑘̂ ). At what points on
the lines should they reach so that the distance between them is the shortest ?
Find the shortest possible distance between them

5 Read the following text and answer the question on the basis of the same. 4
A motor cycle race was organized in a town , where the maximum speed limit was
set by the organizers . No participant are allowed to cross the specified speed limit,
but two motorcycles A and B are running at the speed more than allowed speed on
the road along the lines
𝑟⃗ = 𝑖̂ +2 𝑗̂ – 𝑘̂ + 𝜆 (𝑖̂ +2 𝑗̂ – 𝑘̂ )
and 𝑟⃗ = 3𝑖̂ +3 𝑗̂ + 2k + 𝜇 (2𝑖̂ + 𝑗̂ + 𝑘̂)

(i) Find the cartesian equation of the line along which motorcycle B is
running.
(ii) Find the shortest distance between the lines.

ANSWER

SECTION A

Q. ANSWER MARK
NO
1 (d) 1

153
2 (d) 1
3 (c) 1
4 (b) 1
5 (a) 1
6 (c) 1
7 (d) 1
8 (b) 1
9 (a) 1
10 (d) 1
11 (a) 1
12 (b) 1
13 (b) 1
14 (a) 1
15 (c) 1
16 (d) 1
17 (d) 1
18 (b) 1
19 (c) 1
20 (b) 1

SECTION -B

Q. ANSWER MARK
NO
𝒙 𝒚−𝟏 𝒛+𝟏
1 = = = 𝝀 2
𝟏 𝟐 𝟐

Coordinate of any point on the line (𝝀,2𝝀 + 𝟏, 𝟐𝝀 − 𝟏)


A/q √(𝝀 − 𝟎)𝟐 + (𝟐𝝀 + 𝟏 − 𝟎)𝟐 + (𝟐𝝀 − 𝟏 − 𝟎)𝟐 =√𝟏𝟏
Squaring and solving 𝜆 = ±1
Hence the required points are (1,3,1) and (-1,-1,-3)

2 The given equations of the lines are 2


𝒙−𝟏 𝒚−𝟒 𝒛−𝟑 𝒙−𝟐 𝒚−𝟓 𝟏−𝒛
= = and = =
−𝟐 𝟑𝒑 𝟒 𝟒𝒑 𝟐 𝟕
The d,ratios of the lines are (-2,3p,4) and (4p,2,-7)
A/q -2x4p+3px2 + 4x(-7)=0
Solving p = -14
3 x=2k+2, y=7k+1 z=-3k+3 and x=-q-2 ,y=2q+8 z= 4q+5 2
The given equations can be written as
𝒙−𝟐 𝒚−𝟏 𝒛−𝟑 𝒙+𝟐 𝒚−𝟖 𝒛−𝟓
𝟐
= 𝟕 = −𝟑 = 𝒌 and −𝟏
= 𝟐 = 𝟒 =q
Now, 2x(-1) +7x2+(-3)x4= 0
Hence the given lines are are perpendicular

4 6x-12=3y+9= 2z-2 2
𝒙−𝟐 𝒚+𝟑 𝒛−𝟏
The given equations can be written as and 𝟏 = 𝟏 = 𝟏
𝟔 𝟑 𝟐

154
𝟏 𝟏 𝟏
Hence the direction ratios parallel to the lines are ( , , ) or (1,2,3)
𝟔 𝟑, 𝟐
𝟏 𝟐 𝟑 1 2 3
Direction cosines are ( , , ) =(√√14 √14 √14)
√𝟏+𝟗+𝟒 √𝟏+𝟗+𝟒 √𝟏+𝟗+𝟒

𝒙+𝟑 𝒚−𝟒 𝟖−𝒛 𝒙+𝟑 𝒚−𝟒 𝒛−𝟖


5 = 𝟓 = or = 𝟓 = and the point is (-2,4,-5) 2
𝟑 −𝟔 𝟑 𝟔

̂ ) + 𝝀( (3𝒊̂ +5𝒋̂ + 𝟔𝒌
⃗⃗ = (-2𝒊̂ +4𝒋̂ − 𝟓𝒌
Hence the vector equation of a line is 𝒓 ̂)

SECTION C

Q. ANSWER MARK
NO
1 ̂
Vector in the direction of first line 𝑏⃗⃗ = 3𝒊̂ +4𝒋̂ + 𝟓𝒌 3
̂
Vector in the direction of first line 𝑑⃗ = 4𝒊̂ -3𝒋̂ + 𝟓𝒌
1
Using appropriate formula we get cos 𝜃 =2 =>𝜃 = 𝜋/3

2 ̂)
𝑎1 (𝒊̂ +𝒋̂ − 𝒌
⃗⃗⃗⃗⃗= ̂)
𝑎2 =(𝒊̂ +𝒋̂ + 𝟐𝒌
and ⃗⃗⃗⃗⃗ 3

𝑏 ̂)
⃗⃗⃗⃗1 =(2𝒊̂ -𝒋̂ + 𝒌 and ̂
⃗⃗⃗⃗⃗2 =(4𝒊̂ +2𝒋̂ + 𝟐𝒌
𝑏
3√5
Using appropriate formula we get d= √6
units

3 Coordinate of any point on the line (5𝝀 − 𝟑, 2𝝀 + 𝟏, 𝟑𝝀 − 𝟒)------------(1) 3


D Ratio of the perpendicular line (5𝝀 − 𝟑,2𝝀 − 𝟏, 𝟑𝝀 − 𝟕)
D Ratio of the given line (5,2,3)
Now 5(5𝝀 − 𝟑) + 2(2𝝀 − 𝟏 )+3 (𝟑𝝀 − 𝟕)=0 =>𝝀= 1
(1)=> foot of the perpendicular is (2,3,-1)

4 Let Q (1,6,3) be the given point and P (x,y,z) be a point on the line and R (𝛼, 𝛽, 𝛾) 3
Be the image of Q on the given line
Coordinate of any point on the line (𝝀, 2𝝀 + 𝟏, 𝟑𝝀 + 𝟐)------------(1)
Ratio of the perpendicular line PQ (𝝀 − 𝟏, 2𝝀 − 𝟓, 𝟑𝝀 − 𝟏)
D Ratio of the given line (1,2,3)
a/q 1(𝝀 − 𝟏)+ 2(2𝝀 − 𝟓 )+3(𝟑𝝀 − 𝟏) =0 =>𝝀= 1
Coordinate of P is (1,3,5)
Using midpointformula we get coordinate of the image is (1,0,7)

5 ̂) + 𝒔(𝟐𝒊
⃗⃗ = (𝒊̂ +2𝒋̂ + 𝟑𝒌
Converting the equations in vector form 𝒓 ̂)
̂ +3𝒋̂ + 𝟒𝒌 3
⃗⃗ = (𝟐𝒊
𝒓 ̂) + 𝒔(𝟑𝒊
̂ +4𝒋̂ + 𝟓𝒌 ̂)
̂ +4𝒋̂ + 𝟓𝒌
Proceeding as qno 7 we get d= 1/√6 units

6 P(x1, y1, z1) = (-2, 4, -5) and Q(x2, y2, z2) = (1, 2, 3) 3

PQ=√77
Hence, the direction cosines of the line joining the given two points are

3 2 8
, ,
√77 √77 √77
155
7 We know that the direction ratios of the line passing through two points 3
P(x1, y1, z1) and Q(x2, y2, z2) are given by:

(x2 – x1, y2 – y1, z2 – z1 )

Given points are A (2, 3, – 4), B (1, – 2, 3) and C (3, 8, – 11).

Direction ratios of the line joining A and B are:

(1 – 2, – 2 – 3, 3 + 4)

i.e. (– 1, – 5, 7.)

The direction ratios of the line joining B and C are:

(3 –1, 8 + 2, – 11 – 3)

i.e.,( 2, 10, – 14.)

From the above, it is clear that direction ratios of AB and BC are


proportional.

That means AB is parallel to BC. But point B is common to both AB and


BC.

Hence, A, B, C are collinear points.

8 Let the direction cosines of the line be l, m, and n. 3

l = cos 90° = 0

m = cos 135° = -1/√2

n = cos 45° = 1/√2

Hence, the direction cosines of the line are 0, -1/√2, and 1/√2.

9 Given lines are: 3

(x – 5)/7 = (y + 2)/-5 = z/1 and x/1 = y/2 = z/3

The direction ratios of the given lines are 7, -5, 1 and 1, 2, 3, respectively.

156
We know that,

Two lines with direction ratios a1, b1, c1 and a2, b2, c2 are perpendicular to each other if
a1a2 + b1b2 + c1c2 = 0

Therefore, 7(1) + (-5) (2) + 1 (3)

= 7 – 10 + 3

=0

Hence, the given lines are perpendicular to each other.

10 Let α, β and γ be the angles made by the line with coordinate axes. 3
Then, α = β = γ ⇒cosα = cosβ = cos γ
⇒ l = m = n …(i)
[∵ l = cosα, m = cosβ, n = cosγ]
We know that, l2 + m2 + n2 = 1
∴ l2 + l2 + l2 = 1 [from Eq. (i)]
⇒ 3l2 = 1 ⇒ l2 = 1/3 ⇒ l = ± √1/3
From Eq. (i), direction cosines of a line are
(√1/3,√1/3,√1/3) or (-√1/3,-√1/3,-√1/3)

SECTION D

Q. ANSWER MARK
NO
1 Let the equation of line passing through (1, 2, -4) is 5

157
=> b1 = 2k, b2 = 3k and b3 = 6k. for some constant k.
Thus, the required vector equation of line is
r⃗ = (î + 2ĵ – 4k̂) + λ(2î + 3ĵ + 6k̂).
where λ = λ1k is any constant.
Now, xî + yĵ + zk̂ = (1 + 2λ)Î + (2+ 3λ)ĵ + (- 4 + 6λ)k̂
On comparing the coefficients of î, ĵ and k̂, we get
x = 1 + 2λ, y = 2 + 3λ and z = – 4 + 6λ

which is the required cartesian equation of a line .

2 The equation of a line joining the points B (0, – 1, 3) and C (2, – 3, – 1) is 5


r⃗ = (0î – ĵ + 3k̂) + λ[(2 – 0)î + (- 3 + 1)ĵ + (- 1 – 3)k̂]
⇒ r⃗ = (- ĵ + 3k̂) + λ(2î – 2ĵ – 4k̂)
⇒ r⃗ = (2λ)î + (- 2λ – 1)ĵ + (- 4λ + 3)k̂
So. any point on line BC is to the form
(2λ, – 2λ – 1, – 4λ + 3)
Let foot of the perpendicular drawn from point A to the line BC be T(2λ, – 2λ – 1, –
4λ + 3).

Now, DR’s of line AT is (2λ + 1, – 2λ – 1 – 8, – 4λ + 3 – 4) or (2λ + 1, 2λ – 9, –


4λ – 1).
Since, AT is perpendicular to BC.
∴ 2 × (2λ + 1) + (- 2) × (- 2λ– 9) + (- 4) (- 4λ– 1) = 0
[∵ a1a2 + b1b2 + c1c2 = 0]
⇒ 4λ + 2 + 4λ + 18 + 16λ + 4 = 0
⇒ 24λ + 24 = 0
∴ Coordinates of foot of perpendicular is
T (2 × (- 1)), – 2 × (- 1) – 1, – 4 × (- 1) + 3) or T(- 2, 1, 7)
Let P(x, y, z) be the image of a point A with respect to the line BC. So, point
T is the mid-point of AP.
∴ Coordinates of T = Coordinates of mid-point of AP
⇒ (- 2, 1, 7) = [(x−1)/2,(y+8)/2,(z+4)/2]
On equating the corresponding coordinates, we get
– 2 = (x−1)/2, 1 = (y+8)/2 and 7 = (z+4)/2
⇒ x = -3, y = – 6 and z = 10

3 The equation of line through A(0, -1, -1) and B(4, 5, 5) is 5

158
= 3(0 + 10) – 10 (0 + 14) + 5 (- 20 + 42)
= 30 – 140 + 110 = 0
Hence, the given lines intersect

4 Given lines can be rewritten as 5


r⃗ = (3λ + 1)î + (1 – λ)ĵ – k̂
and r⃗ = (4 + 2µ)î + 0ĵ + (3µ – 1)k̂ ……(ii)
Clearly, any point on line (j) is of the form P (3λ + 1, 1 – λ, – 1)and any point
on tine (ii) is of the form Q (4 + 2µ, 0, 3µ – 1)
If lines (i) and (ii) intersect, then these points must coincide for some λ and
µ.
Consider, 3λ + 1 = 4 + 2µ
⇒ 3λ– 2µ = 3 …….. (iii)
1 – λ = 0 ……. (iv)
and 3µ – 1 = – 1 ………. (v)
From Eq. (iv), we get λ = 1 and put the value of λ in Eq. (iii), we get
3(1) – 2µ = 3
⇒– 2µ = 3 – 3 ⇒µ = 0
On putting the value of µ in Eq. (V), we get
3(0) – 1 = – 1 ⇒ 0 – 1 = – 1
⇒– 1 = – 1,
which is true
Hence, both lines intersect each other.
The point of intersection of both lines can be obtained by putting λ =
1 in coordinates of P. So, the point of intersection is (3 + 1, 1 – 1, –
1). i.e. (4, 0, – 1)

5 Given equation of line is


6x – 2 = 3y + 1 = 2z – 2

159
Here, DR’s of the line are (1, 2, 3).
∴ Direction cosines of the line are

The equation of a line passing through (2, – 1, – 1) and parallel o the


given line is
(x−2)/1=(y+1)/2=(z+1)/3 = λ (say)
⇒ x = 2 + λ, y = – 1 + 2λ and z = – 1 + 3λ
Now, xî + yĵ + zk̂ = (2 + λ)î + (- 1 + 2λ)ĵ + (- 1 + 3λ)k̂
∴ r⃗ = (2î + ĵ + k̂) + λ (î + 2ĵ + 3k̂)
which is the required equation of line in vector form.

SECTION E

Q. ANSWER MARK
NO
1 (i) The equation of motion of a rocket are x = 4t , y = -4t , z = t, 4
at t = 5
x= 20, y = -20, z = 5
so, points lie on the path = (20,-20,5)
(ii) points lie on the path after 5 seconds = (20,-20,5)
Distance from starting point (0,0,0)
= I √400 + 400 + 25 I
= I √825 I
=5 √33

2 b1= (𝑖̂ +2 𝑗̂ – 2 𝑘̂ ) 4
and b2= (2𝑖̂ + 𝑗̂ + 𝑘̂)
b1 . b2 = (𝑖̂ +2 𝑗̂ – 2 𝑘̂ ). (2𝑖̂ + 𝑗̂ + 𝑘̂)
= 2+2-2
=2
I b 1I = √ 1 + 4 + 4 = 3
I b2 I = √4 + 1 + 1 = √6
the angle between two lines cos𝜃 = 2/3√6
So, 𝜃 = cos-1 (2/3√6 )

160
3 x= 4t, y = -4t , z = -2t, 4
Or, x/4 = t, -y/4 = t , z/-2 = t
So, x/4 = y/-4 = z/-2
Direction Ratios are 4,-4,-2
When t = 10 seconds, the airplane will be at the points (40,-40,-20)
Distance from the origin in 10 minutes = √1600 + 1600 + 400
= √3600 = 60 km
Distance of the point (40,-40,-20) from the given line
= I (a2- a1)X𝑏⃖⃗I/I𝑏⃖⃗I
= I -30𝑗̂ X ((10𝑖̂ -20 𝑗̂ +10 𝑘̂ )I/I(10𝑖̂ -20 𝑗̂ +10 𝑘̂ )I
= I -300 𝑖̂ + 300 𝑘̂ I/ I(10𝑖̂ -20 𝑗̂ +10 𝑘̂ )I
=300 √2 /10 √6 = 10 √3 km

4 The given lines are non-parallel lines. There is a unique line segment PQ 4
which is at right angles to both the lines.
Hence, shortest distance between the snakes = PQ
The position vector of P lying on the line 𝑟⃗ =3𝑖̂ +2 𝑗̂ +3 𝑘̂ + 𝜆 (𝑖̂ -2 𝑗̂ +2 𝑘̂ )
is (3+ λ)𝑖̂+(2-2𝜆)𝑗̂ +(3 + 2𝜆)𝑘̂ for some λ
The position vector of Q lying on the line 𝑟⃗ =-4𝑖̂ -2 𝑘̂ + 𝜇 (3𝑖̂ -2 𝑗̂ -2 𝑘̂ )
is (-4+ 3𝜇)𝑖̂ - 2𝜇𝑗̂ +(-2 - 2𝜇)𝑘̂ for some 𝜇

𝑃𝑄 = (-7+ 3𝜇 − λ)𝑖̂ – (-2-2𝜇 + 2 𝜆)𝑗̂ +(-2 - 2𝜇 − 3 − 2 𝜆 )𝑘̂


⃗⃗⃗⃗⃗⃗
Since, Pq is perpendicular to both the lines .
So, (-7+ 3𝜇 − λ)- (-2-2𝜇 + 2 𝜆)(-2) +(-2 - 2𝜇 − 3 − 2 𝜆 )2 = 0
-7+ 3𝜇 − λ -4 -4𝜇 +4 λ -10 -4 𝜇 -4 λ = 0
- λ - 5 𝜇 =21 (i)
(-7+ 3𝜇 − λ)3- (-2-2𝜇 + 2 𝜆)(-2) +(-2 - 2𝜇 − 3 − 2 𝜆 )(-2 )= 0
-21+ 9𝜇 − 3λ -4 -4𝜇 +4 λ +10 +4 𝜇 +4λ = 0
5λ +9 𝜇 =15 (ii)
Solving equn. (i) and (ii)
λ =33/2 and 𝜇 = -15/2
The position vector of the points at which they should be so that the
distance between them is the shortest are
(39𝑖̂ -62 𝑗̂ +70 𝑘̂)/2 and (-53𝑖̂ +30 𝑗̂ +26 𝑘̂ )/2
𝑃𝑄 =(-92𝑖̂ +92 𝑗̂ +44 𝑘̂)/2 =(-46𝑖̂ +46 𝑗̂ +22 𝑘̂)
⃗⃗⃗⃗⃗⃗
The shortest distance = I 𝑃𝑄⃗⃗⃗⃗⃗⃗ 𝐼 = √2116 + 2116 + 484 = √4716
= 2√1179 Unit

5 (i)The line along which motorcycle B is running 4


𝑟⃗ = 3𝑖̂ +3 𝑗̂ + 2k + 𝜇 (2𝑖̂ + 𝑗̂ + 𝑘̂)
(𝑥𝑖̂ +y 𝑗̂ + 𝑧𝑘̂) = (3+2𝜇)𝑖̂ +(3 +𝜇)𝑗̂ +( 2+𝜇) k
x = (3+2𝜇) , y =(3 +𝜇) , z = ( 2+𝜇)
Or, (x-3)/2 = , y-3 = 𝜇 , z-2 = 𝜇
The required cartesian equation =
(x-3)/2 = y-3 = z-2
ii) a1 = 𝑖̂ +2 𝑗̂ – 𝑘̂ , a2 = 3𝑖̂ +3 𝑗̂ + 2k
161
b1 = 𝑖̂ +2 𝑗̂ – 𝑘̂ , b2 = 2𝑖̂ + 𝑗̂ + 𝑘̂
a2 – a1 = 3𝑖̂ +3 𝑗̂ +2 𝑘̂ - 𝑖̂ -2 𝑗̂ + 𝑘̂ = 2𝑖̂ + 𝑗̂ + 3 𝑘̂

𝑖 𝑗 𝑘
b1 X b 2 = [1 2 −1] = 3𝑖̂ +3 𝑗̂ – 3 𝑘̂
2 1 1
(a2 – a1).( b1 X b 2) = (2𝑖̂ + 𝑗̂ + 3 𝑘̂) . (3𝑖̂ +3 𝑗̂ – 3 𝑘̂ )
=6 +3 – 9
=0
So, Shortest distance between given lines = 0.

CHAPTER- 12- LPP

Q. QUESTION MARK
NO
1 All the decision variables of any LPP are – 1

(a) integers (b) real numbers (c) rational numbers (d) non-negative real
numbers

2 Which of the following problems gives a bounded feasible region? 1

(a) x + y ≥ 7, x ≥ 0, y ≥ 0 (b) x + y ≤ 7, x ≥ 0, y ≥ 0

(c) x + y ≥ 5, x ≤ 0, y ≤ 0 (d) x + y ≥ 0, x ≤ 0, y ≤ 0

3 Feasible region indicated by the given inequalities 1


y ≤ 4,
2x + y ≤ 9,
x ≥ 0, y ≥ 0 is –

(a) a bounded region in first quadrant

(b) a bounded region in first and second quadrant

(c) an unbounded region in first quadrant

(d) none of these


4 Which of the following statements is correct? 1

(a) Every LPP has an optimal solution

162
(b) Every LPP has only one optimal solution

(c) If a LPP have two optimal solutions, then it will have infinitely many
optimal solutions

(d) None of these

5 Let Z = 60x + 15y. The value of Max Z – Min Z, if the feasible region for an 1

LPP is shown in attached Fig. is –


(a) 1350 (b) 1650 (c) 1800 (d) 3000

6 If the constraints of a LPP be changed, then – 1

(a) objective function must be changed

(b) optimal solution will be changed

(c) optimal solution will be unchanged

(d) the problem cannot be solved

7 The corner points of the feasible region determined by the system of linear 1
constraints are (0, 5), (0,0), (3,4), (5, 0). Let Z = m.x + n.y , where m , n > 0.
Condition on m and n so that the maximum of Z occurs at both the points
(0, 5) and (3, 4) is –

(a) m = n (b) m = 2n (c) n = 2m (d) n = 3m

8 Which of the following relations is correct for a LPP? 1

(a) Max Z = – Min Z (b) Max Z = – Min (–Z) (c) Max Z = Min (–Z) (d)
None of these

9 The LPP minimize Z = x + y ; 1


subject to the constraints
3x + 4y ≤ 9 ,
3x + 4y ≥ 12
and x , y ≥ 0 –

163
(a) has unique feasible solution

(b) have two feasible solutions

(c) have infinite number of feasible solutions

(d) has no feasible solution

10 Feasible region (shaded) for a LPP is shown in the attached Fig. Minimum of 1
Z = 3x + 9y occurs at the point –

(a) (5,5) (b) (15,15) (c) (0,10) (d) (10,10)

11 Minimize Z = 3x + y , 3
subject to the constraints
2x + y ≥ 14 ,
x–y≥4
and x, y ≥ 0.
Solving graphically we get x= p , y = 2 and Z = 20. Find the value of p.
12 Find the redundant constraints, in solving the following LPP: Minimize 3
f = 6x + 10y
subject to the constraints
x ≥ 6,
y ≥2,
2x + y ≥ 10,
x≥ 0, y ≥ 0.
13 A wholesale merchant wants to start the business of cereal with Rs. 24000. 3
Wheat is Rs. 400 per quintal and rice is Rs. 600 per quintal. He has capacity
to store 200 quintal cereal. He earns the profit Rs. 25 per quintal on wheat
and Rs. 40 per quintal on rice. Then find the linear constraints.

164
14 Find the linear constraints for which the shaded area (i) ∆BDE and (ii) ∆ACE 3
in the figure is the solution set.

15 Solve graphically the following L.P.P. : 3


Maximize Z = 5x1 - 10x2 , subject to the constraints :
2x1 - x2 ≥ 0 ;
–x1 + 5x2 ≥ 5 ;
x1 , x2 ≥ 0

ANSWERS:

Q. ANSWER MARKS
NO
1 (d) 1
2 (b) 1
3 (a) 1
4 (c) 1
5 (c) 1
6 (b) 1
7 (d) 1
8 (b) 1
9 (d) 1
10 (a) 1
11 Given objective function is Z = 3x + y , which we have to minimize. 2
Now, x=p,y=2 and Z = 20, satisfies the objective function ⇒20 = 3p+2. Hence,
p = 6.
12 When x ≥ 6 and y ≥ 2, then 2x + y ≥ 2×6+2, 2
i.e., 2x + y ≥ 14
Hence, x ≥ 0, y ≥ 0 and 2x + y ≥ 10 are automatically satisfied by every point of
the region {(x, y): x ≥ 6 }∩{(x, y):y ≥ 2}
Hence the required redundant constraints are x ≥ 0, y ≥ 0 and 2x + y ≥ 10.
13 Let x quintal rice and y quintal wheat be stored. 2

165
Then obviously x ≥ 0, y ≥ 0 and x + y ≤ 200, since he has capacity to store 200
quintal cereal
Again, total cost = Rs. (600x + 400y).
So, 600x + 400y ≤ 24000
Hence, the required linear constraints are
x + y ≤ 200, 3x + 2y ≤ 120, x ≥ 0, y ≥ 0.
14 Equation of the line AB is x + 2y = 8 and equation of the line CD is 3x + 2y = 12 3
and these lines meet at E(2,3) .
Hence, (i) For the shaded portion ∆BDE, the linear constraints are:
3x + 2y ≤ 12, x + 2y ≥ 8, x ≥ 0.
(ii) For the shaded portion ∆ACE, the linear constraints are:
3x + 2y ≥ 12, x + 2y ≤ 8, y ≥ 0.

15 Considering a set of rectangular Cartesian coordinate axes OX1, OX2 in the 3


plane we draw the graph of the given constraints in the first quadrant.
Here the shaded feasible region is unbounded. The only vertex of the feasible

region is B (5/9, 10/9).


Assuming Z = 10, the profit line 5x1 - 10x2 = 10 is given in the figure by the bold
line. Since this is a maximization problem, we move the profit line parallel
away from the origin. This line will never leave out the feasible region and the
given L.P.P. has unbounded solution.

166
CHAPTER-13: PROBABILITY

Q. QUESTION M
NO A
R
K

1 4 7 𝐵 2
If for any two events A and B, P(A) =5 and P(A∩ 𝐵) = 10, then find 𝑃 (𝐴)

2 A bag contains 3 white, 4 black and 2 red balls. If 2 balls are drawn at random (without 2
replacement), then find the probability that both the balls are white.

3 A die, whose faces are marked 1, 2, 3 in red and 4, 5, 6 in green, is tossed. Let A be the event 2
“number obtained is even” and B be the event “number obtained is red”. Find if A and B are
independent events.

4 𝐴 2
Let A and B be two events. If 𝑃(𝐴) = 0.2, 𝑃(𝐵) = 0.4, 𝑃(𝐴 ∪ 𝐵) = 0.6 then find 𝑃 (𝐵)

5 A black and a red die are rolled together. Find the conditional probability of obtaining the sum 8, 2
given that the red die resulted in a number less than 4.

6 To test the quality of electric bulbs produced in a factory, two bulbs are randomly selected from a 2
large sample without replacement. If either bulb is defective, the entire lot is rejected. Suppose a
sample of 200 bulbs contains 5 defective bulbs. Find the probability that the sample will be rejected.

7 A fair die is rolled. Consider the events A={1, 3, 5}, B={2, 3} and C={2, 3, 4, 5}. 2
Find (i) 𝑃(𝐴/𝐵) (ii) 𝑃(𝐵/𝐴)

8 A fair coin and an unbiased die are tossed. Let A be the event ‘head appears on the coin’ and B be 2
the event ‘3 on the die’. Check whether A and B are independent events or not .

167
9 10% of the bulbs produced in a factory are red colour and 2 % are red and defective. If one bulb is 2
picked at random, determine the probability of its being defective if it is red.
2
10 The probability that a person will get an electric contract is and the probability that he will not get 2
5
4 2
plumbing contract is 7. If the probability of getting at least one contract is 3 , what is the probability
that he will get both ?

11 A refrigerator box contains 2 milk chocolates and 4 dark chocolates. Two chocolates are drawn at 3
random. Find the probability distribution of the number of milk chocolates. What is the most likely
outcome ?

12 Bag I contains 3 red and 4 black balls while another bag II contains 5 red and 6 black balls. One ball is 3
drawn at random from one of the bags and it is found to be red. Find the probability that it was
drawn from bag II .

13 The probability distribution of a random variable X is given below : 3

X 1 2 3

P(X) 𝑘 𝑘 𝑘
2 3 6
(i) Find the value of k
(ii) Find P(1≤ 𝑋 < 3)
(iii) Find E(X), the mean of X

14 Suppose 5% of men and 0.25% of women have grey hair. A grey haired person is selected at 3
random. What is the probability of this person being male ? Assume that there are equal number of
males and females.

15 A random variable X has the following probability distribution : 3

X 0 1 2 3 4 5 6 7

P(X) 0 𝑘 2𝑘 2𝑘 3𝑘 𝑘2 2𝑘 2 7𝑘 2 + 𝑘

Determine :
(i) 𝑘 (ii) P(𝑋 < 3) (iii) P(𝑋 > 6)

16 A bag contains 4 red and 3 black balls. A second bag contains 2 red and 4 black balls. One bag is 3
selected at random. From the selected bag, one ball is drawn. Find the probability that the ball
drawn is red.

17 Suppose that 6% of the people with blood group O are left handed and 10% of those with other 3
blood groups are left handed. 30% of the people have blood group O. If a left handed person is
selected at random, what is the probability that he/she will have blood group O ?

168
18 An electric assembly consists of two sub-systems say A and B. From previous testing procedures, the 3
following probabilities are assumed to be known
P(A fails)=0.2, P(B fails alone)=0.15, P( A and B fail)=0.15
Evaluate the following probabilities
(i) P(A fails/B has failed)
(ii) P(A fails alone)

19 A machine operates if all of its three components function. The probability that the first component 3
fails during the year is 0.14, the second component fails is 0.10 and the third component fails is
0.05. What is the probability that the machine will fail during the year ?

20 Two thirds of the students in a class are boys and the rest are girls. It is known that the probability 3
of a girl getting a first class is 0.25 and that of a boy getting a first class is 0.28. Find the probability
that a student chosen at random will get first class marks in the subject.

ANSWERS:

Q. ANSWER MA
NO RKS

1 𝐵 𝑃(𝐴 ∩ 𝐵) 1
𝑃( ) =
𝐴 𝑃(𝐴)
7
10 35 7
= 4 = =
40 8
5 1

2 𝐶23 1
𝑃(𝐵𝑜𝑡ℎ 𝑏𝑎𝑙𝑙𝑠 𝑎𝑟𝑒 𝑤ℎ𝑖𝑡𝑒) = 9
𝐶2
3!
1
= 2!1!
9! = 12
2!7!
1

169
3 Here, A= Event that “number obtained is even”
B= Event that “number obtained is red”

3 1 3 1 1
𝑃(𝐴) = = ; 𝑃(𝐵) = = ;
6 2 6 2 2
1 1 1 1
𝑃(𝐴 ∩ 𝐵) = ; 𝑃(𝐴) × 𝑃(𝐵) = × =
6 2 2 4
1
i.e., 𝑃(𝐴 ∩ 𝐵) ≠ 𝑃(𝐴) . 𝑃(𝐵)
Hence, A and B are not independent events.
1
2
4 Since, 𝑃(𝐴 ∪ 𝐵) = 𝑃(𝐴) + 𝑃(𝐵) − 𝑃(𝐴 ∩ 𝐵)
0.6 = 0.2 + 0.4 − 𝑃(𝐴 ∩ 𝐵)
𝑃(𝐴 ∩ 𝐵) = 0.6 − 0.6 = 0 1
𝐴 𝑃(𝐴 ∩ 𝐵) 0
∴ 𝑃( ) = = =0
𝐵 𝑃(𝐵) 0.4
1

5 Let E = “Event of obtaining the sum 8”


= {(2,6), (6,2), (3,5), (5,3), (4,4)}
And F = “Event that red die resulted in a number less than 4”
= {(1,1), (2,1), (3,1), (4,1), (5,1), (6,1), (1,2), (2,2), (3,2), (4,2), (5,2), (6,2), (1,3), (2,3), (3,3),
(4,3), (5,3), (6,3)}
𝐸 ∩ 𝐹 = {(6,2), (5,3)}
1
2
𝐸 𝑃(𝐸∩𝐹) 36 1
𝑃 (𝐹 ) = = 18 =9
𝑃(𝐹)
36
1

6 Clearly, the sample will be rejected if at least one of the two bulbs is defective. Therefore, if we
consider the following events :
A = First bulb is defective, B= Second bulb is defective
Then,

Required probability =𝑃(𝐴 ∪ 𝐵) = 1 − 𝑃(𝐴 ∪ 𝐵) = 1 − 𝑃(𝐴 ∩ 𝐵)


1
𝐵 195 194 197
1 − 𝑃(𝐴). 𝑃 ( ) = 1 − × =
𝐴 200 199 3980
1

170
1
7 𝐴 𝑃(𝐴 ∩ 𝐵) 1
6
𝑃( ) = = 1 =
𝐵 𝑃(𝐵) 2 1
3
1
𝐵 𝑃(𝐴 ∩ 𝐵) 6 1
𝑃( ) = = 1 =
𝐴 𝑃(𝐴) 3
2
1

8 The sample space related to the experiment is given by


S={(H,1), (H,2), (H,3), (H,4), (H,5), (H,6), (T,1), (T,2), (T,3), (T,4), (T,5), (T,6)}
We have
A={(H,1), (H,2), (H,3), (H,4), (H,5), (H,6)}, B={(H,3), (T,3)} and so A∩ 𝐵 ={(H,3)} 1
6 1 2 1 1
∴ 𝑃(𝐴) = = , 𝑃(𝐵) = = 𝑎𝑛𝑑 𝑃(𝐴 ∩ 𝐵) =
12 2 12 6 12
1

9 Consider the events


A= the bulb produced is red, B= the bulb produced is defective
10 1 2 1
𝑃(𝐴) = 100 = 10 and 𝑃(𝐴 ∩ 𝐵) = 100 = 50

∴ The required probability = 1


1
𝐵 𝑃(𝐴 ∩ 𝐵) 50 1
𝑃( ) = = 1 =
𝐴 𝑃(𝐴) 5
10

10 Consider the events


A= Person gets an electric contract, B= Person gets plumbing contract
2 4 2
𝑃(𝐴) = 5 , 𝑃(𝐵) = 7 and 𝑃(𝐴 ∪ 𝐵) = 3

Now, 𝑃(𝐴 ∪ 𝐵) = 𝑃(𝐴) + 𝑃(𝐵) − 𝑃(𝐴 ∩ 𝐵) 1

2 2 4
⇒3 = 5 + (1 − 7) − 𝑃(𝐴 ∩ 𝐵)
2 3 2 17
⇒𝑃(𝐴 ∩ 𝐵) = 5 + 7 − 3 = 105
1

171
11 Let X denote the number of milk chocolate drawn.

X P(X)

0 4 3 12
× =
6 5 30 1
2 4 16 2
1 (6 × 5) × 2 = 2
30

2 2 1 2
× =
6 5 30
Most likely outcome is getting one chocolate of each type. 1
2
12 Let 𝐸1 be the event of choosing the bag I, 𝐸2 the event of choosing the bag II and A be the event
of drawing a red ball.
1
Then 𝑃(𝐸1 ) = 𝑃(𝐸2 ) = 2
𝐴 3
Also, 𝑃 (𝐸 ) = P (drawing a red ball from bag I)=7
1

𝐴 5
And 𝑃 (𝐸 ) = P (drawing a red ball from bag II)=11
2
1
𝐸2
Now, the probability of drawing a ball from bag II, being given that it is red, is 𝑃 ( 𝐴 ). By using
Bayes’ theorem, we have
𝐴 1 5
𝐸2 𝑃(𝐸2 )𝑃 (𝐸 ) × 11 35
2 2
𝑃( ) = 𝐴 𝐴
= 1 3 1 5 =
𝐴 𝑃(𝐸1 )𝑃 (𝐸 ) + 𝑃(𝐸2 )𝑃 (𝐸 ) 2 × 7 + 2 × 11 68
1 2

13 𝑘 𝑘 𝑘 1
(i) ∑ 𝑃(𝑋𝑖 ) = 1 ⇒ 2
+3+6 =1 ⇒𝑘=1
𝑘 𝑘 5𝑘 5×1 5
(ii) P(1≤ 𝑋 < 3) = 𝑃(1) + 𝑃(2) = 2 + 3 = = =6
6 6
1
𝑘 𝑘 𝑘 5𝑘 5×1 5
(iii) 𝐸(𝑋) = ∑ 𝑝𝑖 . 𝑥𝑖 = 1 × 2 + 2 × 3 + 3 × 6 = = =3
3 3 1

172
14 Let 𝐸1 , 𝐸2 𝑎𝑛𝑑 𝐴 be the event such that

𝐸1 = Selecting male person


𝐸2 =Selecting female person
𝐴 = Selecting grey haired person
1 1 𝐴 5 𝐴 0.25 1
Then 𝑃(𝐸1 ) = 2, 𝑃(𝐸2 ) = 2, 𝑃 (𝐸 ) = 100 , 𝑃 (𝐸 ) = = 400
1 2 100
1
Hence, required probability is
𝐴 1 5
𝐸1 𝑃(𝐸1 )𝑃 (𝐸 ) × 100 20
1 2
𝑃( ) = 𝐴 𝐴
= 1 5 1 1 =
𝐴 𝑃(𝐸1 )𝑃 ( ) + 𝑃(𝐸2 )𝑃 ( ) 2 × 100 + 2 × 400 21
𝐸1 𝐸2
2

15 (i) ∑𝑛𝑖=1 𝑝𝑖 = 1 ⇒ 0 + 𝑘 + 2𝑘 + 2𝑘 + 3𝑘+𝑘 2 + 2𝑘 2 + 7𝑘 2 + 𝑘 = 1


⇒10𝑘 2 + 9𝑘 − 1 = 0
1
⇒ 𝑘 = −1(𝑛𝑒𝑔𝑙𝑒𝑐𝑡𝑒𝑑 𝑎𝑠 𝑘 𝑐𝑎𝑛 𝑛𝑒𝑣𝑒𝑟 𝑏𝑒 𝑛𝑒𝑔𝑎𝑡𝑖𝑣𝑒) 𝑎𝑛𝑑 𝑘 =
10
1
1
⇒ 𝑘=
10
3
(ii) P(𝑋 < 3) = 𝑃(𝑋 = 0) + 𝑃(𝑋 = 1) + 𝑃(𝑋 = 2) = 0 + 𝑘 + 2𝑘 = 3𝑘 = 10 1

1 1 17
(iii) P(𝑋 > 6) = 𝑃(𝑋 = 7) = 7𝑘 2 + 𝑘 = 7 × 100 + 10 = 100
1

16 Let 𝐸1 , 𝐸2 𝑎𝑛𝑑 𝐴 be the event such that

𝐸1 = Selecting bag I
𝐸2 =Selecting bag II
𝐴 = Drawing a red ball
1 1 𝐴 4 𝐴 2
Then 𝑃(𝐸1 ) = 2, 𝑃(𝐸2 ) = 2, 𝑃 (𝐸 ) = 7 , 𝑃 (𝐸 ) = 6
1 2
1
Using the law of total probability, we have
𝐴 𝐴 1 4 1 2 19
Required probability= P(A)=𝑃(𝐸1 )𝑃 (𝐸 ) + 𝑃(𝐸2 )𝑃 (𝐸 ) = 2 × 7 + 2 × 6 = 42
1 2
2

173
17 Let 𝐸1 , 𝐸2 𝑎𝑛𝑑 𝐴 be the event such that

𝐸1 = A person with blood group O is selected.


𝐸2 = A person with other blood groups is selected
𝐴 = A left handed person is selected
30 3 70 7 𝐴 6 𝐴 10 1
Then 𝑃(𝐸1 ) = 100 = 10, 𝑃(𝐸2 ) = 100 = 10, 𝑃 (𝐸 ) = 100 , 𝑃 (𝐸 ) = 100
1 2

Hence, required probability is


𝐴 3 6
𝐸1 𝑃(𝐸1 )𝑃 (𝐸 ) × 100 9
1 10
𝑃( ) = 𝐴 𝐴
= 3 6 7 10 =
𝐴 𝑃(𝐸1 )𝑃 ( ) + 𝑃(𝐸2 )𝑃 ( ) 10 × 100 + 10 × 100 44
𝐸1 𝐸2
2

18 Consider the events


E= A fails, F=B fails

𝑃(𝐸) = 0.2 , 𝑃(𝐸 ∩ 𝐹) = 0.15 and 𝑃(𝐸 ∩ 𝐹) = 0.15

Now, 𝑃(𝐸 ∩ 𝐹) = 0.15 ⇒ 𝑃(𝐹) − 𝑃(𝐸 ∩ 𝐹) = 0.15

⇒𝑃(𝐹) = 0.15 + 0.15 = 0.30 1


𝐸 𝑃(𝐸∩𝐹) 0.15 1 1
(i) P(A fails/B has failed)=𝑃 (𝐹 ) = 𝑃(𝐹)
= 0.30 = 2
1
(ii) P(A fails alone)=𝑃(𝐸 ∩ 𝐹) = 𝑃(𝐸) − 𝑃(𝐸 ∩ 𝐹) = 0.2 − 0.15 = 0.05

19 Consider the events


A=First component of the machine fails during the year,
B=Second component of the machine fails during the year
C=Third component of the machine fails during the year
We have 𝑃(𝐴) = 0.14, 𝑃(𝐵) = 0.10 and 𝑃(𝐶) = 0.05 1
Clearly, the machine will fail if at least one of its three components fail during the year

∴ Required probability= 𝑃(𝐴 ∪ 𝐵 ∪ 𝐶) = 1 − 𝑃(𝐴 ∪ 𝐵 ∪ 𝐶) = 1 − 𝑃(𝐴 ∩ 𝐵 ∩ 𝐶)

= 1 − 𝑃(𝐴). 𝑃(𝐵). 𝑃(𝐶) = 1 − (1 − 0.14)(1 − 0.10)(1 − 0.05)

= 1 − (0.86)(0.90)(0.95) = 0.2647
2

174
20 Consider the events
𝐸1 =A boy is chosen from the class,
𝐸2 =A girl is chosen from the class
A=the students get first class marks
Then, 𝑃(𝐸1 ) = 2/3 , 𝑃(𝐸2 ) = 1/3, 𝑃(𝐴/𝐸1 ) =0.28 and𝑃(𝐴/𝐸2 ) =0.25 1
Using the law of total probability, we obtain
𝐴 𝐴 2 1 2
P(A)=𝑃(𝐸1 )𝑃 (𝐸 ) + 𝑃(𝐸2 )𝑃 (𝐸 ) = 3 × 0.28 + 3 × 0.25 = 0.27
1 2

𝐴 1−𝑃(𝐵)
1 Two events A and B are such that 𝑃(𝐵) ≠ 0, then 𝑃 (𝐵) . 𝐴 = 1
1−𝑃( )
𝐵

a) 𝑃(𝐵)
b) 𝑃(𝐴)
c) 𝑃(𝐴 ∩ 𝐵)
d) None of these

2 If two events are A and B, then the probability of happening only 1


one event of them exactly
a) 𝑃(𝐴) + 𝑃(𝐵) − 𝑃(𝐴 ∩ 𝐵)

b) 𝑃(𝐴) + 𝑃(𝐵) − 2𝑃(𝐴 ∩ 𝐵)

c) 𝑃(𝐴) + 𝑃(𝐵) − 𝑃(𝐴 ∪ 𝐵)


𝐴 𝐵
d) 𝑃 (𝐵) + 𝑃 (𝐴)

3 Given P (A) = ½, P (B) = p and P (AB) = 3/5. The value of p for 1


which A and B are mutually exclusive.
1
a) 5
1
b) 15
1
c) 10
1
d) 7
4 The probability of having 53 Mondays in a leap year chosen 1
randomly
1 3 4 2
a) b) 7 c)7 d) 7
7

175
5 If the probability for A to fail in an examination is 0.2 and that for 1
B is 0.3. Then the probability that neither fails.
a) 0.57 b) 0.58 c) 0.56 d) 0.65

6 If A and B are independent events, and P(A)= ½, P(B) =1/3, then 1


P (𝐴 ∪ 𝐵 )=
2 1 1
a) b) 6 c) 3 d)
3
4
3

7 A random variable X is specified by the following distribution. 1

X 2 3 4
P (X = r) 0.3 0.4 0.3
the mean of this distribution is
a) 2 b) 2.7 c) 3.5 d) 3

8 Two dice are thrown. It is known that the sum of numbers on 1


the dice is less than 6, the probability of getting a sum 3 is
(a) 1/18 (b) 1/10 (c) 2/5 (d) 1/5

9 Events A and B are independent if 1


(a) P(A∩B) =P(A/B) P(B)
(b) P(A∩B) =P(B/A) P(A)
(c) P(A∩B) =P(A)+P(B)
(d) P(A∩B) =P(A)P(B)
10 If two events are 1
independent, then
(a)they must be mutually
exclusive

(b) the sum of their probabilities must be equal to 1


(c) both(a) and (b) are correct
(d) none of the above is correct
11 If P(A∩B) = 0.15, P(B’) = 0.10, 1
then P(A/B) =
(a) 1/3 (b) ¼ (c) 1/6 (d) 1/5

176
12 A bag contains 5 red and 3 blue balls. If 3 balls are drawn 1
at random without replacement, the probability of
getting exactly one red ball is
(a) 45/196 (b) 135/392 (c) 15/56 (d)
15/29

13 If P(A) = 1/2, P(B) = 0, then P(A|B) is 1


(a) 0
(b) 1/2
(c) not defined
(d) 1
14 If A and B are any two events such that P(A) + P(B) – P (A and 1
B) =P(A), then

(a) P(B|A) = 1
(b) P(A|B) = 1
(c) P(B|A) = 0
(d) P(A|B) = 0
15 If A and B are two events such that P(A) ≠ 0 and P (B | A) = 1, 1
then

(a) A ⊂ B
(b) B ⊂ A
(c) B = φ
(d) A = φ
16 The probability of obtaining an even prime number on each die, 1
when a pair of dice is rolled is
(a) 0 (b) 1/3 (c) 1/12 (d) 1/36

17 Two events A and B will be independent, if 1


(a) A and B are mutually
exclusive

(b) P(A′∩B′) = [1 – P(A)] [1 –


P(B)]

(c) P(A) = P(B)

(d) P(A) + P(B) = 1

18 If P(A|B) > P(A), then which of the following is correct: 1


(a) P(B|A) < P(B)
(b) P(A ∩ B) < P(A) . P(B)
(c) P(B|A) > P(B)
(d) P(B|A) = P(B)

177
19 Events A and B are said to be mutually exclusive iff 1
(a) P (A ∩ B) =P(A) +P(B)
(b) P (A ∩ B) =P(A)P(B)
(c) A ∩ B = φ
(d) None of these
20 If A and B are independent events, then which of the following is 1
not true
(a) P(A|B) = P(A)
(b) P(B|A) = P(B)
(c) P(B|A) = P(A|B)
(d) None of these
Long Questions (4 Marks)
1 Two cards are drawn simultaneously (or successively without 4
replacement) from a well shuffled pack of 52 cards. Find the
mean of the number of aces.

2 Colored balls are distributed in three bags as shown in the 4


following table

Colour of the ball


Bag
Red White Black

I 1 2 3

II 2 4 1

III 4 5 3

A bag is selected at random and two balls are randomly


drawn from the selected bag. They happen to be black and red.
What is the probability that they came from bag I.

3 . A factory has three machine X, Y, Z producing 1000, 2000, 4


3000 bolts per day respectively. The machine X produced
1% defective bolts, Y produce 1.5% and Z produce 2%
defective bolts. At the end of a day, a bolt is drawn at
random and is found defective. What is the probability
that the defective bolt is produced by the machine X?

178
4 Two cards are drawn in succession from a well shuffled deck of 4
52 cards, the first card being replaced, before the second is
drawn. Let X denote the number of spades drawn. Find the
probability distribution of X?

5 A man is known to speak truth 3 out of 4 times. He throws a die 4


and reports that it is six. Find the probability that it is
actually a six.

ANSWERS:
Q. NO ANSWER MARKS
1 d)𝑃(𝐴) 1
Explanation: (𝐴 ∩ 𝐵) ∪ (𝐴 ∩ 𝐵) = 𝐴 ∩ (𝐵 ∪ 𝐵) = 𝐴 ∩ 𝑆 = 𝐴

𝑃(𝐴 ∩ 𝐵) + 𝑃(𝐴 ∩ 𝐵) = 𝑃(𝐴)

𝐴 𝐴
𝑃( )+𝑃( ) = 1
𝐵 𝐵
𝐵 𝐴 𝐴
𝐴 𝑃(𝐴 ∩ 𝐵) 𝑃(𝐴). 𝑃 (𝐴) 𝑃(𝐴). [1 − 𝑃 (𝐵)] 𝑃 (𝐵) . 1 − 𝑃(𝐵)
𝑃( ) = = = = 𝐴
= 𝑃(𝐴)
𝐵 𝑃(𝐵) 1 − 𝑃(𝐵) 1 − 𝑃(𝐵) [1 − 𝑃 ( )] 𝐵

2 b) 𝑃(𝐴) + 𝑃(𝐵) − 2𝑃(𝐴 ∩ 𝐵) 1


Explanation: 𝑃(𝐸) = 𝑃(𝐴 ∩ 𝐵) + 𝑃(𝐴 ∩ 𝐵) = 𝑃(𝐴) − 𝑃(𝐴 ∩ 𝐵) + 𝑃(𝐵) −
𝑃(𝐴 ∩ 𝐵) = 𝑃(𝐴 ∪ 𝐵) − 𝑃(𝐴 ∩ 𝐵) = 1 − 𝑃(𝐴 ∪ 𝐵) − 1 + 𝑃(𝐴 ∩ 𝐵)

= 𝑃(𝐴 ∪ 𝐵) − 𝑃(𝐴 ∩ 𝐵) = 𝑃(𝐴) + 𝑃(𝐵) − 2𝑃(𝐴 ∩ 𝐵)


1
3 c)10 1
Explanation: if A and B are mutually exclusive
𝑃(𝐴 ∪ 𝐵) = 𝑃(𝐴) + 𝑃(𝐵)
3 1
= +𝑝
5 2
3 1 1
𝑝= − =
5 2 10

179
2
4 d)7 1
Explanation: a leap year has 52 weeks and 2 odd days. These 2 days may be first and
last day of a week. Such pair of days may be of 7 kinds where there will be 2
2
Mondays. So, the required probability is 7
1
5 c)3 1

Explanation: 𝑃(𝐴) = 0.2 , 𝑃(𝐵) = 0.3

So, 𝑃(𝐴) = 1 − 0.2 = 0.8, 𝑃(𝐵) = 1 − 0.3 = 0.7


𝑃(𝑛𝑒𝑖𝑡ℎ𝑒𝑟 𝑓𝑎𝑖𝑙𝑠) = 𝑃(𝐴 ∩ 𝐵) = 𝑃(𝐴). 𝑃(𝐵) = 0.8 × 0.7 = 0.56
As A and B are independent event.
1
6 c)
3
1
1 1 1 2 1
Explanation: P (𝐴 ∪ 𝐵)=𝑃(𝐴)𝑃(𝐵) = (1 − 2) . (1 − 3) = 2 × 3 = 3

7 d)3 1
Explanation: Req. mean= 𝐸(𝑋) = 𝑝1 𝑥1 + 𝑝2 𝑥2 + 𝑝3 𝑥3 = (2 × .3) + (3 × .4) +
(4 × .3) = 3
1
8 d)5 1
Explanation:

𝐴 = 𝑠𝑢𝑚 < 6 = 2, 3, 4 ,5
= {(1,1), (1, 2), (2,1), (3,1), (1, 3), (2, 2), (2, 3), (3, 2), (4, 1), (1, 4)}
𝐵 = 𝑠𝑢𝑚 3 = {(1, 2), (2, 1)}
𝐴 ∩ 𝐵 = {(1, 2), (2, 1)}
𝐵 𝑃(𝐴 ∩ 𝐵) 2 1
𝑃( ) = = =
𝐴 𝑃(𝐴) 10 5

9 d)The condition of being independent events is 𝑃(𝐴 ∩ 𝐵) = 𝑃(𝐴). 𝑃(𝐵) 1


10 d)none of above option is correct 1
1
11 c) 1
6

Explanation: P(A∩B) = 0.15, P(B’) = 0.10


So, P(B)=1-0.10=0.90
𝐴 𝑃(𝐴 ∩ 𝐵) 0.15 1
𝑃( ) = = =
𝐵 𝑃(𝐵) 0.90 6

180
15
12 c)56 1
Explanation: P (getting exactly one red ball in three drawn)
𝑃𝑅 . 𝑃𝑅 𝑃𝑅 + 𝑃𝑅 𝑃𝑅 𝑃𝑅 + 𝑃𝑅 𝑃𝑅 𝑃𝑅

5 3 2 3 5 2 3 2 5 5 5 5 15
=( × × )+( × × )+( × × ) = × × =
8 7 6 8 7 6 8 7 6 56 56 56 56
13 c)not defined 1
𝐴 𝑃(𝐴∩𝐵) 𝑃(𝐴∩𝐵)
Explanation: 𝑃 (𝐵) = = = 𝑛𝑜𝑡 𝑑𝑒𝑓𝑖𝑛𝑒𝑑.
𝑃(𝐵) 0

14 d)1 1
Explanation: P(A) + P(B) – P (A and B) =P(A)
𝑃(𝐴 ∩ 𝐵) = 𝑃(𝐵)
𝑃(𝐴 ∩ 𝐵)
=1
𝑃(𝐵)
𝐴
𝑃( ) = 1
𝐵
15 a) A ⊂ B 1
𝐵
Explanation: 𝑃 (𝐴) = 1

𝑃(𝐴 ∩ 𝐵)
=1
𝑃(𝐴)
𝑃(𝐴 ∩ 𝐵) = 𝑃(𝐴)
𝐴∩𝐵 =𝐴
So, A ⊂ B

1
16 d) 1
36

Explanation: 𝐴 =
{(1, 2), (2,1 ), (2, 2), (2, 3), (2, 4), (2, 5), (2, 6), (3, 2), (4, 2), (5,2), (6, 2)}
1
𝑃(2, 2) =
36
17 b)(A′∩B′) = [1 – P(A)] [1 – P(B)] as A and B are 1
independent then 𝐴 and 𝐵 are also independent.

181
𝐵
18 c)𝑃 (𝐴) > 𝑃(𝐵) 1
𝐵
Explanation: 𝑃 (𝐴) > 𝑃(𝐴)

𝑃(𝐴 ∩ 𝐵)
> 𝑃(𝐴)
𝑃(𝐵)

𝑃(𝐴 ∩ 𝐵)
> 𝑃(𝐴)
𝑃(𝐵)
𝐵
𝑃 ( ) > 𝑃(𝐵)
𝐴
19 c)𝐴 ∩ 𝐵 = ∅ 1
20 (a) c) P(B|A) = P(A|B) 1

Long questions (4)-Answer

182
1 Solution: 4

If X is the number of aces drawn

C0 48 C2 248  47 564 188


4
P( X = 0) = = = =
52
C2 52  51 663 221

C1 48 C1 4  48  2 96
4
32
P( X = 1) = 52 = = =
C2 52  51 663 221

C1 48 C0
4
4  3 2 3 1
P( X = 2) = 52 = = =
C2 2  51 51 663 221

 The Probability Distribution of X is given by

X 0 1 2

188 32 1
P(X)
221 221 221

188 32 1 34
 Pi xi = 0  + 1 + 2 =
 221 221 221 221

183
2 Solution: 4
Let E1 : Bag I is selected; E2: Bag II is selected

Let E3 : Bag III is selected and A = A black ball and a red ball is drawn

1
𝑃(𝐸1 ) = 𝑃(𝐸2 ) = 𝑃(𝐸3 ) = 3

𝐴 1×3 3 1
𝑃 (𝐸 ) = = 15 = 5
1 6𝐶2

𝐴 2×1 2
𝑃 (𝐸 ) = = 21
2 7𝐶2

𝐴 4×3 4×3 2
𝑃 (𝐸 ) = 12𝐶 = = 11
3 2 66

Using Baye’s Theorem


1 1 1
𝐸 ×
𝑃 (𝐴 1 ) = 1 3 5
1 1 2 1 2 = 5
1 2 2
× + × + × + +
3 5 3 21 3 11 5 21 11

231
= 551

184
3 Solution: 4
Let E1: Bolt is manufactured by machine ‘X’, E2: Bolt is manufactured by machine
Y and E3: Bolt is manufactured by machine Z,
Total number of bolts manufactured by machine X, Y, Z in one day
= 1,000 +2,000 +3,000 = 6,000
Therefore P(E/E1)=1/6 P(E/E2)=1/3 P(E/E3)=1/2
Let E: Bolt manufactured is defective

P(E/E1) = 1/100, P(E/E2) = 1.5/100 = 3/200 and P(E/E3) = 2/100

Required probability = P (E3/E) = P(E3) P (E/E3)


P (E1) P (E/E1) + P (E2) P (E/E2) + P(E3)
P (E/E3)
1/6× 1/100 1/6
= _________________________________ = _________________ = 1/ 10
1/6× 1/100+ 1/3× 3/200+ ½ × 2/100 1/6 + ½ + 1

4 Solution: 4
The probability of getting a spade when a single card is drawn = 13/52 = ¼
or P (not getting a spade) = 1- ¼= ¾
P (X =0) = P (no spade is drawn) = P ( not a spade ) P (not a spade)
= ¾ × ¾= 9/16
P(X=1) = P (drawing a spade only at one draw)
= P (a spade and not a spade) + P (not a spade and a spade)
= ¼×¾+¾× ¼=⅜
P(X=2) = P (drawing spades at both the draws)
= ¼ × ¼ = 1/16 Hence, the probability distribution of x is

X 0 1 2
P(X) 9/16 3/8 1/16

185
5 Solution: 4
Let E= the event that the man reports that six occurs in the throwing of a dice.

𝑆1 = event of getting six, 𝑆2 = event of getting no six


1 55
𝑃(𝑆1 ) = 6, 𝑃(𝑆2 ) = 6

P(E/S_1 )= probability that the man speaks the truth=5/6

P(E/S_2 )= probability that the man does not speak the truth=3/4

To find P(S_1/E)=probability that report of the man that six has occurred is actually a
six

=P(S_1 )P(E/S_1 )/(P(S_1 )P(E/S_1 )+𝑃(𝑆2 ) P(E/S_2 )=3/8

Q. NO QUESTION MARK

1 A coach is training three players .He observed that player A hits 4 times the shot
in 5 tries, player B hits 3 times the shot in 4 and player C is able to hit twice the
shot in 3 tries.

Based on above information, answer following questions

i)A,B,C all hit. It is a shot. What is the probability that the shot is hit by B only?
1
ii)If A,B,C all try , what is the probability that it was hit by none?
1
iii)Find the probability that the shot was hit by exactly 2 players.
2

2 Read the following passage and answer the questions given below:

In an Office three employees Jayant, Sonia and Olivia process a calculation in an


excel form. Probability that Jayant, Sonia, Olivia process the calculation

186
respectively is 50%, 20% and 30% . Jayant has a probability of making a mistake
as 0.06 , Sonia has probability 0.04 to make a mistake and Olivia has a
probability 0.03 . Based on the above information, answer the following
questions.

(i)Find the probability that Sonia processed the calculation and committed a
mistake.
1
(ii)Find the total probability of committing a mistake in processing the
calculation.

(iii)The boss wants to do a good check. During check, he selects a calculation 1


form at random from all the days. If the form selected at random has a mistake,
find the probability that the form is not processed by Jayant.

3 Tiki has started late for college.She is running towards Laboni bus-stop. To reach
college she has to change buses from either Hidco Crossing or Dharmatala.For
that she would take either bus A or bus B.Probability of getting into bus A, B are
3/7 , 4/7 . If she gets on bus A coming from Karunamoyee , she would get bus 1
or 2 from Hidco crossing. Probability of getting bus 1 from Hidco crossing is ⅖ ,
probability of getting bus 2 from Hidco crossing is ⅗ .If she gets on bus B from
Quality crossing and gets bus 1 or bus 3 from Dharmatala. Probability of getting
bus 1 from Dharmatala is ⅓, probability of getting bus 3 from Dharmatala is ⅔ .

i)Tiki reaches college by bus 1 .What is the probability that she caught bus B? 2
ii)What is the probability that she reaches college by bus 2? 2

4 Rubiya, Thaksh, Shanteri, and Lilly entered a spinning zone for a fun game, but
there is a twist: they don't know which spinner will appear on their screens until
it is their turn to play. They may encounter one of the following spinners, or
perhaps even both. Spinners have numbers 1 to 9 on those: Different
combinations of numbers will lead to exciting prizes. Below are some of the
rewards they can win:

187
● Get the number '5', from Spinner A and '8' from Spinner B, and you'll win
a music player!
● You win a photo frame if Spinner A lands on a value greater than 4.
● You win an earplug if you get even in spinner A or odd in spinner B.
i)Thaksh spun both the spinners, A and B in one of his turns. What is the
probability that Thaksh wins a music player in that turn?
1
ii) Lilly spun spinner A in one of her turns.What is the probability that the
number she got is even given that it is a multiple of 3?

iii) Rubiya spun both the spinners.What is the probability that she wins a photo 1
frame only?

Or 2
As Shanteri steps up to the screen, the game administrator reveals that she
would see either Spinner A or Spinner B for her turn, the probability of seeing
Spinner A on the screen is 65%, while that of Spinner B is 35%. What is the
probability that Shanteri wins an earplug?

5 A doctor is to visit a patient. From past experience, it is known that the


probabilities that he will come by cab, metro, bike or by other means of
transport are respectively 0.3, 0.2, 0.1 and 0.4. The probabilities that he will be
late are 0.25, 0.3, 0.35 and 0.1 if he comes by cab, metro, bike and other means
of transport respectively.

Based on the above information, answer the following questions.


1
(i) When the doctor arrives late, what is the probability that he comes by metro?
1
(ii) When the doctor arrives late, what is the probability that he comes by cab?
1
(iii) When the doctor arrives late, what is the probability that he comes by bike?
1
(iv)When the doctor arrives late, what is the probability that he comes by other
means of transport?

188
6 Two boxes containing candies are placed on a table. The boxes are labeled𝐵1 5
and 𝐵2. Box 𝐵1 contains 7 cinnamon candies and 4 ginger candies. Box
𝐵2contains 3 cinnamon candies and 10 pepper candies. The boxes are arranged
so that the probability of selecting box 𝐵1 is 1⁄3 and the probability of selecting
box 𝐵2 is 2⁄3. Suresh is blindfolded and asked to select a candy. He will win a
color TV if he selects a cinnamon candy. What is the probability that Suresh will
win the TV (that is, he will select a cinnamon candy)?

7 Three companies A, B and C supply 25%, 35% and 40% of the notebooks to a 5
school. Past experience shows that 5%, 4% and 2% of the notebooks produced
by these companies are defective. If a notebook was found to be defective, what
is the probability that the notebook was supplied by A?

8 If a machine is correctly set up, it produces 90% acceptable items. If it is 5


incorrectly set up, it produces only 40% acceptable items. Past experience shows
that 80% of the set ups are correctly done. If after a certain set up, the machine
produces acceptable items, find the probability that the machine is correctly set
up.

9 Of the students in a college, it is known that 60% reside in hostels and 40% are 5
day scholars (not residing in hostels). Previous year results report that 30% of all
students who reside in hostels attain A grade and 20% of day scholars attain A
grade in their annual examination. At the end of the year, one student is chosen
at random from the college and he has an A grade. What is the probability that
the student is a hostler?

10 An urn contains 5 red and 5 black balls. A ball is drawn at random, its color is 5
noted and is returned to the urn. Moreover, 2 additional balls of the color drawn
are put in the urn and then a ball is drawn at random. What is the probability
that the second ball is red?

ANSWERS:

Q. NO ANSWER MARKS

1 Let A, B and C be the events that players A, B and C hit respectively.

Let D be the event that it is a shot.


4 3 2
Then, P(A) = 5, P(B) = 4, P(C) = 3

i) A,B,C all hit. It is a shot.

The probability that the shot is hit by B only

189
4 3 2
=(1-5) 4 (1-3) ½

3
=60

1
=20
½
ii)If A,B,C all try , the probability that it was hit by none is
4 3 2 1
(1-5)(1- 4) (1-3)=60
½
iii) The probability that the shot was hit by exactly 2 players ½
4 3 2 4 3 2 4 3 2
=(5)(4) (1-3) + (1-5)( 4) (3) +(5)(1- 4) (3)

26 1
=
60

13 ½
=30
½

2 Let A, B and C be the events that calculation is done byJayant, Sonia and Olivia
respectively.

Let D be the event that there is an error in the calculation.


50 20 30
Then, P(A) = 100, P(B) = 100, P(C) = 100 and

P(D/A) = 0.06 ,P(D/B) =0.04 ,P(D/C) =0.03

(i)The probability that Sonia processed the calculation and committed a mistake
𝑃(𝐷/𝐵)𝑃(𝐵)
= 𝑃(𝐷/𝐴)𝑃(𝐴)+ 𝑃(𝐷/𝐵)𝑃(𝐵)+ 𝑃(𝐷/𝐶)𝑃(𝐶)

20
0.04 𝑋 ½
100
= 50 20 30
0.06 𝑋 +0.04 𝑋 +0.03 𝑋
100 100 100

8
=47 ½

(ii)The total probability of committing a mistake in processing the calculation


𝐷 𝐷 𝐷
= 𝑃 (𝐴 ) 𝑃(𝐴) + 𝑃 (𝐵 ) 𝑃(𝐵) + 𝑃 (𝐶 ) 𝑃(𝐶)

50 20 30 ½
=0.06 x 100+ 0.04 x100+ 0.03 x 100
½
=.047

(iii) If the form selected at random has a mistake, the probability that the form is not
processed by Jayant is

190
=1-probability that the form has a mistake and is processed by Jayant ½
𝑃(𝐷/𝐴)𝑃(𝐴) 1
=1-𝑃(𝐷/𝐴)𝑃(𝐴)+ 𝑃(𝐷/𝐵)𝑃(𝐵)+ 𝑃(𝐷/𝐶)𝑃(𝐶))

50 ½
0.06 𝑋
100
=1- 50 20 30
0.06 𝑋 +0.04 𝑋 +0.03 𝑋
100 100 100

17
=47

3 i)If Tiki reaches college by bus 1 , the probability that she caught bus B is
4 1
𝑋
7 3
1
=3 2 4 1
𝑋 + 𝑋
7 5 7 3

20
=38
½
10
=19 ½

ii)The probability that she reaches college by bus 2


4 3 1
=7 x 5

12
=35
1

4 i)If Thaksh spun both the spinners, the probability that Thaksh wins a music player in
that turn is

no of way to get the number '5', from Spinner A and '8' from Spinner B /total count of
1
pairs of values on both Spinner
1𝑋 1 1
=9 𝑋 9=81

ii)Lilly spun spinner A in one of her turns. She could get any of 1,2,3,4,5,6,7,8,9. Out
of these even and multiple of 3 is 6 only.
1
So the probability that the number she got is even given that it is a multiple of 3 is 9. 1

iii) Rubiya spun both the spinners.The probability that she wins a photo frame only is
5𝑋 9 5
=9
9𝑋9
2
Or
65 4 35 5 535 107
The probability that Shanteri wins an earplug =100 X 9 + 100 X 9 =900=180

5 he will come by cab,metro, bike or by other means of transport are respectively 0.3,
0.2, 0.1 and 0.4. The probabilities that he will be late are 0.25, 0.3, 0.35 and 0.1 if he
comes by cab, metro, bike and other means of transport

191
i)When the doctor arrives late, the probability that he comes by metro is
0.2 𝑋 0.3 0.06 6 2
= = = 1
0.3 𝑋 0.25 + 0.2 𝑋 0.3 + 0.1 𝑋 0.35 + 0.4 𝑋 0.1 0.21 21 7

(ii) When the doctor arrives late, the probability that he comes by cab is
0.3 𝑋 0.25 0.075 75 5 1
= 0.21 =210=14
0.3 𝑋 0.25 + 0.2 𝑋 0.3 + 0.1 𝑋 0.35 + 0.4 𝑋 0.1

(iii) When the doctor arrives late, the probability that he comes by bike is
0.1 𝑋 0.35 0.035 35 1
= 0.21 =210=6
0.3 𝑋 0.25 + 0.2 𝑋0.3 + 0.1 𝑋 0.35 + 0.4 𝑋 0.1
1
(iv)When the doctor arrives late, the probability that he comes by other means of
transport is
0.4 𝑋 0.1 0.04 4
= =
0.3 𝑋0.25 + 0.2 𝑋 0.3 + 0.1 𝑋 0.35 + 0.4 𝑋 0.1 0.21 21
1

6 Let A be the event of drawing a cinnamon candy. 1

Let 𝐵1 be the event of selecting box 𝐵1.

Let 𝐵2 be the event of selecting box 𝐵2 . 1


1 2
Then, P(𝐵1) =3 and P(𝐵2 ) = 3 ½

P(A) = P(A ∩ 𝐵1) + P(A ∩ 𝐵2) ½

= P(A|𝐵1) P(𝐵1) + P(A|𝐵2) P(𝐵2) 1

7 1 3 2
= (11) (3) + (11) (3)
1
13
= 33.

7 Let A, B and C be the events that notebooks are provided by A, B and C respectively.

Let D be the event that notebooks are defective

Then,

P(A) = 0.25, P(B) = 0.35, P(C) = 0.4 1

P(D|A) = 0.05, P(D|B) = 0.04, P(D|C) = 0.02 ½


𝑃(𝐷|𝐴)𝑃(𝐴)
P(A│D) =𝑃(𝐷|𝐴)𝑃(𝐴)+ 𝑃(𝐷|𝐵)𝑃(𝐵)+ 𝑃(𝐷|𝐶)𝑃(𝐶)

0.05∗0.25 1
=0.05∗0.25 + 0.04∗0.35 + 0.02∗0.4
1
125
= 345
1

192
25 ½
=69.

8 Let A be the event that the machine produces 2 acceptable items.

Also let 𝐵1 represent the event of correct set up and 𝐵2represent the event of

incorrect setup. Now

P(𝐵1) = 0.8, P(𝐵2) = 0.2 ½

P(A|𝐵1) = 0.9 and P(A|𝐵2) = 0.4 1

Therefore
𝑃(𝐵1 ) 𝑃(𝐴|𝐵1 )
P(𝐵1|A) =𝑃(𝐵
1 ) 𝑃(𝐴|𝐵1 ) + 𝑃(𝐵2 )𝑃(𝐴|𝐵2 )

0.8× 0.9 1
=0.8× 0.9 + 0.2 × 0.4
1
72
=80
1
9
= 10. ½

9 Let event 𝐸1 : a student is residing in hostel

𝐸2 : a student is a day scholar (i.e., not residing in hostel)

then𝐸1 and 𝐸2 are mutually exclusive and exhaustive events.

Given: 60% students reside in hostel and 40% don’t reside in

hostel (i.e., are day scholars) 1


60 40
∴P(𝐸1 ) = 100=0.6 and P(𝐸2 ) = 100=0.4

Let event E: a student attains A grade

Given: 30% of hostlers get A grade and 20% day scholars get A

grade.
30 20 1
∴P(E / 𝐸1 ) i.e., probability that a hostler gets A grade= =0.3 and P(E / 𝐸2 ) = =
100 100
0.2

We have to find P(𝐸1 / E). i.e., P(a student getting A grade resides in hostel)
𝑃(𝐸 / 𝐸1 ) 𝑃(𝐸1 )
We know that P(𝐸1 / E) =𝑃(𝐸 / 𝐸
1 ) 𝑃(𝐸1 )+𝑃(𝐸 / 𝐸2 )𝑃(𝐸2 )

0.6 𝑋 0.3
1
=0.6 𝑋 0.3 +0.4 𝑋 0.2

18
=26

193
9
=13 1

10 Urn contains 5 red and 5 black balls (So Total balls = 5 + 5 = 10)

Let 𝐸1 : first draw gives a red ball, 𝐸2 : first draw gives a black ball
5 1 5 1 1
∴P(𝐸1 ) =10=2 and P𝐸2 ) = 10=2

When the first draw gives a red ball, two additional red balls are put in the urn so that
its contents are 7 (= 5 + 2) red and 5 black balls. When the first draw gives a black
ball, two additional black balls are put in the urn so that its contents are 5 red and 7
(= 5 + 2) black balls.

Let A: The second draw gives a red ball.


7 5
P(A/𝐸1 )=12 ,P(A/𝐸2 )=12
1
Required probability = P(A)=

Probability that the first ball is red and then the second ball drawn after two reds are
added in the urn is also red + Probability that the first ball is black and second is red.

=P(A ∩ 𝐸1 ) + P(A ∩ 𝐸2 ) 1

=P(A/𝐸1 )P(𝐸1 )+P(A/𝐸2 )P(𝐸2 ) ½


7 1 5 1
= 12 x 2 + 12 x 2 ½

1 ½
=2
½

ADDITIONAL QUESTIONS ON PROBABILITY

Q. QUESTION MARK
NO
1−𝑃(𝐵)
1 Two events A and B are such that 𝑃(𝐵) ≠ 0,then 𝑃 (𝐴⁄ ̅ ) . 1−𝑃(𝐴⁄ ) = 1
𝐵 𝐵

a) 𝑃(𝐵)
b) 𝑃(𝐴)
c) 𝑃(𝐴 ∩ 𝐵)
d) None of these
2 If two events are A and B, then the probability of happening only one event of 1
them exactly

194
a) 𝑃(𝐴) + 𝑃(𝐵) − 𝑃(𝐴 ∩ 𝐵)
b) 𝑃(𝐴̅) + 𝑃(𝐵̅ ) − 2𝑃(𝐴̅ ∩ 𝐵̅ )
c) 𝑃(𝐴) + 𝑃(𝐵) − 𝑃(𝐴 ∪ 𝐵)
d) 𝑃 (𝐴⁄ ̅ ) + 𝑃 (𝐵⁄ ̅)
𝐵 𝐴

3 Given P (A) = ½, P (B) = p and P (AB) = 3/5. The value of p for which A and B are 1
mutually exclusive.
1
a) 5
1
b) 15
1
c) 10
1
d) 7
4 The probability of having 53 Mondays in a leap year chosen randomly 1
1 3 4 2
a) b) 7 c)7 d) 7
7
5 If the probability for A to fail in an examination is 0.2 and that for B is 0.3. Then the 1
probability that neither fails.

a) 0.57 b) 0.58 c) 0.56 d) 0.65

6 If A and B are independent events, and P(A)= ½, P(B) =1/3, then 1


P (𝐴̅ ∪ 𝐵̅ )=
2 1 1 4
a) b) 6 c) 3 d) 3
3

7 A random variable X is specified by the following distribution. 1

X 2 3 4

P (X = r) 0.3 0.4 0.3

the mean of this distribution is


a) 2 b) 2.7 c) 3.5 d) 3
8 Two dice are thrown. It is known that the sum of numbers on the dice is 1
lessthan6,the probabilityofgettingasum3 is
(a) 1/18 (b) 1/10 (c) 2/5 (d) 1/5

9 EventsAandBareindependentif 1
(a) P(A∩B)=P(A/B)P(B)

(b) P(A∩B)=P(B/A)P(A)
(c) P(A∩B)=P(A)+P(B)

195
(d) P(A∩B)=P(A)P(B)
10 Iftwoeventsare independent,then 1
(a)theymustbemutuallyexclusive

(b) The sum of their probabilities must be equal to1


(c) both(a)and(b)are correct
(d) none of the above is correct
11 IfP(A∩B)=0.15,P(B’)=0.10,thenP(A/B)= 1
(a) 1/3 (b) ¼ (c) 1/6 (d) 1/5

12 A bag contains 5 red and 3 blue balls. If 3 balls are drawn at random 1
without replacement,the probability of getting exactly one red ball is
(a) 45/196 (b) 135/392 (c) 15/56 (d) 15/29

13 IfP(A)=1/2,P(B)=0,thenP(A|B)is 1
(a) 0
(b) 1/2
(c) Not defined
(d) 1
14 If A and B are any two events such that P(A) + P(B) – P (A and B) =P(A),then 1
(a) P(B|A)=1
(b) P(A|B)=1
(c) P(B|A)=0
(d) P(A|B)=0
15 IfA and B are two events such that P(A)≠0 andP(B |A)=1,then 1
(a) A ⊂B
(b) B ⊂A
(c) B =φ
(d) A=φ
16 The probability of obtaining an even prime number on each die, when a pair of 1
dice is rolled is
(a) 0 (b) 1/3 (c) 1/12 (d) 1/36
17 Two events A and B will be independent,if 1
(a) A and B are mutually exclusive
(b)P(A′∩B′)=[1–P(A)][1 –P(B)]
(c)P(A)=P(B)
(d)P(A)+P(B)=1
18 IfP(A|B)>P(A),then which of the following is correct: 1
(a) P(B|A)<P(B)
(b) P(A∩B)<P(A).P(B)
(c) P(B|A)>P(B)
(d) P(B|A)=P(B)
19 Events A and B are said to be mutually exclusive iff 1
(a) P(A∩B)=P(A)+P(B)
(b) P(A∩B)=P(A)P(B)
(c) A∩B=φ
(d) None of these
196
20 If A and B are independent events, then which of the following is not true 1
(a) P(A|B)= P(A)
(b) P(B|A)= P(B)
(c) P(B|A)=P(A|B)
(d) None of these
CASE STUDY BASED- (4 Marks)
1 An insurance company believes that people can be divided into two 4
classes: those who are accident prone and those who are not. The
company’s statistics show that an accident-prone person will have an
accident at some time within a fixed one-year period with probability
0.6,where as this probability is 0.2 for a person who is not accident prone.

The company knows that 20 percent of the population is accident prone.


Based on the given information answer the following questions:

(i) What is the probability that a new policy holder will have an
accident within a year of purchasing a policy?

(ii) Suppose that a new policy holder has an accident within a year of
Purchasing a policy. What is the probability that he or she is accident
prone?

2 A shopkeeper sells three types off lower seeds A1,A2 and A3.They are 4
soldas a mixture where the proportions are 4:4:2 respectively. The
germination rates of three types of seeds are 45%,60%,35%.

Based on the given information, answer the following questions:

(i) Find the probability of a randomly chosen seed to germinate

(ii) Find the probability that seed will not germinate given that it is of the
type A3
(iii) that is of the type A2, given that a randomly chosen seed does not
germinate.

3 An item is manufactured by three machines A, B and C. Out of the total 4


numbers of items manufactured during a specified period,50% are
manufactured on A, 30% are manufactured on B, 20% are manufactured
on C. 2% of items produced on A, 2% of items produced on B and
3%produced on Care defective. All the items are stored at one storeroom.

(i) One item is drawn at random and is found to be defective. What is the
probability that it is manufactured on machine A?

ii) One item is drawn at random and is found to be defective. What is the
probability that it is manufactured on machine B?

197
4 The reliability of a COVID PCR test is specified as follows: Of people 4
having COVID, 90% of the test detects the disease but 10% goes
undetected. Of people free of COVID,99% of the test is judged COVID

Negative but 1% are diagnosed as showing COVID positive. From a large


population of which only 0.1% have COVID, one person is selected at
random, given the COVID PCR test, and the pathologist reports him/her
as COVID positive.

(i) A person is selected at random and tested. What is the probability that
he is tested positive?

(ii) What is the probability that the person is actually having COVID given
that he is tested as COVID positive’?

5 An electronic assembly consists of two sub-systems A and B. From previous 4


testing procedures, the following probabilities are assumed to be known:

P(A fails)= 0.2

P (B fails alone) = 0.15

P (A and B fail) = 0.15

Based on the above information, answer the following:

(i) P (A fails or B fails)


(ii) P (A fails/B has failed)
(iii) P (A fails alone)

ANSWERS:
Q. NO ANSWER MARKS
1 d)𝑃(𝐴) 1
Explanation: (𝐴 ∩ 𝐵) ∪ (𝐴 ∩ 𝐵̅ ) = 𝐴 ∩ (𝐵 ∪ 𝐵̅ ) = 𝐴 ∩ 𝑆 = 𝐴
𝑃(𝐴 ∩ 𝐵) + 𝑃(𝐴 ∩ 𝐵̅ ) = 𝑃(𝐴)
𝐴 𝐴
𝑃( )+𝑃( ) = 1
𝐵 𝐵̅

198
𝐵̅ 𝐴 𝐴
𝐴 𝑃(𝐴 ∩ 𝐵̅ ) 𝑃(𝐴). 𝑃 (𝐴) 𝑃(𝐴). [1 − 𝑃 (𝐵)] 𝑃 (𝐵̅) . 1 − 𝑃(𝐵)
𝑃( ) = = = = = 𝑃(𝐴)
𝐵̅ 𝑃(𝐵̅ ) 1 − 𝑃(𝐵) 1 − 𝑃(𝐵) 𝐴
[1 − 𝑃 ( )] 𝐵

2 b) 𝑃(𝐴̅) + 𝑃(𝐵̅ ) − 2𝑃(𝐴̅ ∩ 𝐵̅ ) 1


Explanation: 𝑃(𝐸) = 𝑃(𝐴 ∩ 𝐵̅ ) + 𝑃(𝐴̅ ∩ 𝐵) = 𝑃(𝐴) − 𝑃(𝐴 ∩ 𝐵) + 𝑃(𝐵) −
𝑃(𝐴 ∩ 𝐵) = 𝑃(𝐴 ∪ 𝐵) − 𝑃(𝐴 ∩ 𝐵) = 1 − 𝑃(𝐴 ̅̅̅̅̅̅̅ ̅̅̅̅̅̅̅
∪ 𝐵 ) − 1 + 𝑃(𝐴 ∩ 𝐵)
= 𝑃(𝐴̅ ∪ 𝐵̅ ) − 𝑃(𝐴̅ ∩ 𝐵̅ ) = 𝑃(𝐴̅) + 𝑃(𝐵̅ ) − 2𝑃(𝐴̅ ∩ 𝐵̅ )

3 1
c)10 1
Explanation: if A and B are mutually exclusive
𝑃(𝐴 ∪ 𝐵) = 𝑃(𝐴) + 𝑃(𝐵)
3 1
= +𝑝
5 2
3 1 1
𝑝= − =
5 2 10
4 2
d)7 1
Explanation: a leap year has 52 weeks and 2 odd days. These 2 days may be first
and last day of a week. Such pair of days may be of 7 kinds where there will be 2
2
Mondays. So, the required probability is 7

5 1
c)3 1
Explanation: 𝑃(𝐴̅) = 0.2 , 𝑃(𝐵̅ ) = 0.3
So, 𝑃(𝐴) = 1 − 0.2 = 0.8, 𝑃(𝐵) = 1 − 0.3 = 0.7
𝑃(𝑛𝑒𝑖𝑡ℎ𝑒𝑟 𝑓𝑎𝑖𝑙𝑠) = 𝑃(𝐴 ∩ 𝐵) = 𝑃(𝐴). 𝑃(𝐵) = 0.8 × 0.7 = 0.56
As A and B are independent event.

6 1
c)3 1
1 1 1 2 1
Explanation: P(𝐴̅ ∪ 𝐵̅)=𝑃(𝐴̅)𝑃(𝐵̅ ) = (1 − 2) . (1 − 3) = 2 × 3 = 3

7 d)3 1
Explanation: Req. mean=𝐸(𝑋) = 𝑝1 𝑥1 + 𝑝2 𝑥2 + 𝑝3 𝑥3 = (2 × .3) + (3 × .4) +
(4 × .3) = 3

8 1
d)5 1
Explanation:
𝐴 = 𝑠𝑢𝑚 < 6 = 2, 3, 4 ,5
= {(1,1), (1, 2), (2,1), (3,1), (1, 3), (2, 2), (2, 3), (3, 2), (4, 1), (1, 4)}

199
𝐵 = 𝑠𝑢𝑚 3 = {(1, 2), (2, 1)}
𝐴 ∩ 𝐵 = {(1, 2), (2, 1)}
𝐵 𝑃(𝐴 ∩ 𝐵) 2 1
𝑃( ) = = =
𝐴 𝑃(𝐴) 10 5

9 d)The condition of being independent events is 𝑃(𝐴 ∩ 𝐵) = 𝑃(𝐴). 𝑃(𝐵) 1


10 d)none of above option is correct 1
11 c)
1
1
6

Explanation: P(A∩B)=0.15,P(B’)=0.10
So, P(B)=1-0.10=0.90
𝐴 𝑃(𝐴 ∩ 𝐵 ) 0.15 1
𝑃( ) = = =
𝐵 𝑃(𝐵) 0.90 6
15
12 c)56 1
Explanation: P (getting exactly one red ball in three drawn)
𝑃𝑅 . 𝑃𝑅̅ 𝑃𝑅̅ + 𝑃𝑅̅ 𝑃𝑅 𝑃𝑅̅ + 𝑃𝑅̅ 𝑃𝑅̅ 𝑃𝑅
5 3 2 3 5 2 3 2 5 5 5 5 15
=( × × )+( × × )+( × × ) = × × =
8 7 6 8 7 6 8 7 6 56 56 56 56
13 c)not defined 1
𝐴 𝑃(𝐴∩𝐵) 𝑃(𝐴∩𝐵)
Explanation: 𝑃 (𝐵) = = = 𝑛𝑜𝑡 𝑑𝑒𝑓𝑖𝑛𝑒𝑑.
𝑃(𝐵) 0

14 d)1 1
Explanation: P(A) + P(B) – P (A and B) =P(A)
𝑃(𝐴 ∩ 𝐵) = 𝑃(𝐵)
𝑃(𝐴 ∩ 𝐵)
=1
𝑃(𝐵)
𝐴
𝑃( ) = 1
𝐵
15 a) A ⊂B 1
𝐵
Explanation: 𝑃 (𝐴) = 1

𝑃(𝐴 ∩ 𝐵)
=1
𝑃(𝐴)
𝑃(𝐴 ∩ 𝐵) = 𝑃(𝐴)
𝐴∩𝐵 =𝐴
So, A ⊂B

200
1
16 d)36 1
Explanation: 𝐴 =
{(1, 2), (2,1 ), (2, 2), (2, 3), (2, 4), (2, 5), (2, 6), (3, 2), (4, 2), (5,2), (6, 2)}
1
𝑃(2, 2) =
36
17 b) (A′∩B′)=[1–P(A)][1 –P(B)] as A and B are 1
independent then 𝐴̅ and 𝐵̅ are also
independent.

𝐵
18 c)𝑃 (𝐴) > 𝑃(𝐵) 1
𝐵
Explanation: 𝑃 (𝐴) > 𝑃(𝐴)

𝑃(𝐴 ∩ 𝐵)
> 𝑃(𝐴)
𝑃(𝐵)

𝑃(𝐴 ∩ 𝐵)
> 𝑃(𝐴)
𝑃(𝐵)
𝐵
𝑃 ( ) > 𝑃(𝐵)
𝐴
19 c)𝐴 ∩ 𝐵 = ∅ 1
20 c) P(B|A)=P(A|B) 1

CASE STUDY BASED (4)-Answer


1 Solution: 4

i)Let A: policy holder who is accident prone

B: Policy holder who is not accident prone

And E: Person will have an accident within 1 year of purchasing policy.

P(A)= 20%= 2/10, P(B) = 80%= 8/10

P(E/A) = prob of accident-prone person within 1 year of policy= 0.6

P(E/B) = Prob of not accident prone within 1 year of policy = 0.2

201
𝐸
P(new policy holder will have an accident) = total prob = 𝑃(𝐴) × 𝑃 (𝐴) +
𝐸
𝑃(𝐵) × 𝑃 (𝐵) = 0.2 × 0.6 + 0.8 × 0.2 = 0.28

𝐸
𝑃(𝐴).𝑃( ) 12 3
𝐴
ii) P(accident prone/accident) = P(A/E) = 𝐸 𝐸 = 28 = 7 by Baye’s
𝑃(𝐴)×𝑃( )+𝑃(𝐵)×𝑃( )
𝐴 𝐵

theorem.

2 Solution: 4

We have, 𝐴1 : 𝐴2 : 𝐴3 = 4: 4: 2 where 𝐴1 , 𝐴2 , 𝐴3 denote the three types of flower


seeds.

P (𝐴1 )= 4/10, P (𝐴2 ) = 4/10, P (𝐴3 ) = 2/10

E: event that that a seed germinates

𝐸̅ : event that a seed not germinates

P(E/𝐴1 )= 45/100, P(E/𝐴2 ) = 60/100, P(E/𝐴3 ) = 35/100

𝐸̅ 𝐸̅ 𝐸̅
P (𝐴 ) = 55/100, P (𝐴 ) = 40/100, P (𝐴 ) = 65/100
1 2 3

i)P(E)= total prob = P (𝐴1 ). P(E/𝐴1 )+ P (𝐴2 ). P(E/𝐴2 )+ P (𝐴3 ). P(E/𝐴3 ) =


490/1000 = 0.49

𝐸̅
ii) P (𝐴 ) = 1- P(E/𝐴2 ) = 1-35/100 = 65/100
2

E
𝐴 P (𝐴2 ).P( )
𝐴2
iii) P ( 𝐸2 ) = E E E = 16/51 = 0.314 by Baye’s theorem.
P (𝐴1 ).P( )+ P (𝐴2 ).P( )+ P (𝐴3 ).P( )
𝐴1 𝐴2 𝐴3

3 Solution: 4

𝐸1 = item manufactured on A

𝐸2 = item manufactured on B

𝐸3 =item manufactured on C
202
E= event of being defective
P (𝐸1 ) = 50/100, P (𝐸2 ) = 30/100, P (𝐸3 )= 20/100
P (E/𝐸1 )= 2/100= 1/50, P (E/𝐸2 )= 2/100= 1/50, P (E/𝐸3 ) = 3/100
P (𝐸1 /𝐸) = prob of being defective from A =5/11 by Baye’s theorem
P (𝐸2 /𝐸) = prob of being defective fromB = 3/11

4 Solution: 4

E: person selected has COVID

F: does not have COVID

G- Test judge COVID positive

P (E) = prob that the person selected has COVID = 0.1% =0.1/100 = 0.001

P (F) = prob that the person selected has not COVID = 1- P (E) = 1-0.001= 0.999

P (G/E) = prob that the test judges COVID +, if the person has actually COVID = 90% =
90/100 = 0.9

P (G/F) = prob that the test judges COVID +, if the person does not haveCOVID = 1%
=1/100 = 0.01

(i)P (person tested positive) = P (E). P (G/E)+ P (F). P (G/F) = 0.001× 0.9 +
0.999 × 0.01 = 0.01089
𝑃(𝐸).𝑃(𝐺/𝐸)
(ii) By Baye’s theorem, P (E/G) =P (E).P (G/E) + P (F).P (G/F) = 9/108.9 = 90/1089 = 0.083

5 Solution: 4
Let, 𝐸𝐴 = event in which A fails
𝐸𝐵 = event in which B fails
P (𝐸𝐴 ) = 0.2, P (𝐸𝐴 ∩ 𝐸𝐵 ) = 0.15
P (B fails alone) = P (𝐸𝐵 ) - P (𝐸𝐴 ∩ 𝐸𝐵 )→ 0.15 = P (𝐸𝐵 ) – 0.15 → P (𝐸𝐵 ) = 0.30
(i) P (𝐸𝐴 ∪ 𝐸𝐵 ) = P (𝐸𝐴 ) + P (𝐸𝐵 ) - P (𝐸𝐴 ∩ 𝐸𝐵 ) = 0.2 + 0.3 – 0.15 = 0.35
(ii) P (A fails/ B has failed) = P (𝐸𝐴 ∩ 𝐸𝐵 )/ P (B) = 0.15/0.30 = ½ = 0,5
(iii) P (A fails only) = P (𝐸𝐴 ) - P (𝐸𝐴 ∩ 𝐸𝐵 ) = 0.2 – 0.15 = 0.05

203
204

You might also like